Feat(1ST): DM1

This commit is contained in:
Bertrand Benjamin 2023-03-15 08:58:29 +01:00
parent e3ffe1e626
commit 285ac83539
101 changed files with 7785 additions and 0 deletions

Binary file not shown.

View File

@ -0,0 +1,158 @@
\documentclass[a4paper,12pt]{article}
\usepackage{myXsim}
\usepackage{pgfplots}
\usetikzlibrary{decorations.markings}
\pgfplotsset{compat=1.18}
\title{ DM1 \hfill ANEX DIT CHENAUD Lou}
\tribe{1ST}
\date{A rendre pour le lundi 27 mars 2023}
\duree{}
\xsimsetup{
solution/print = false
}
\pagestyle{empty}
\begin{document}
\maketitle
Le barème est donné à titre indicatif, il pourra être modifié.
\begin{exercise}[subtitle={Polynôme de degré 2}]
Développer les expressions suivantes pour vérifier que ce sont des polynômes de degré 2. Vous préciserez les valeurs de $a$, $b$ et $c$.
\begin{multicols}{2}
\begin{enumerate}
\item $f(x) = (- 6x + 3)(- 7x + 3)$
\item $g(x) = (10x - 3)^{2}$
\item $h(x) = - 4 + x(- 7x - 10)$
\item $i(x) = - 4x^{2} + x(- 8x - 1)$
\item $j(x) = - 3(x + 6)(x + 8)$
\item $k(x) = 6(x + 5)(x - 10)$
\end{enumerate}
\end{multicols}
\end{exercise}
\begin{solution}
\begin{enumerate}
\item
\begin{align*}
f(x) &= (- 6x + 3)(- 7x + 3)\\&= - 6x \times - 7x - 6x \times 3 + 3 \times - 7x + 3 \times 3\\&= - 6(- 7) \times x^{1 + 1} + 3(- 6) \times x + 3(- 7) \times x + 9\\&= - 18x - 21x + 42x^{2} + 9\\&= (- 18 - 21) \times x + 42x^{2} + 9\\&= 42x^{2} - 39x + 9
\end{align*}
C'est un polynôme de degré 2 avec $a = 42$, $b = - 39$ et $c = 9$.
\item
\begin{align*}
g(x) &= (10x - 3)^{2}\\&= (10x - 3)(10x - 3)\\&= 10x \times 10x + 10x(- 3) - 3 \times 10x - 3(- 3)\\&= 10 \times 10 \times x^{1 + 1} - 3 \times 10 \times x - 3 \times 10 \times x + 9\\&= - 30x - 30x + 100x^{2} + 9\\&= (- 30 - 30) \times x + 100x^{2} + 9\\&= 100x^{2} - 60x + 9
\end{align*}
C'est un polynôme de degré 2 avec $a = 100$, $b = - 60$ et $c = 9$.
\item
\begin{align*}
h(x) &= - 4 + x(- 7x - 10)\\&= - 4 + x \times - 7x + x(- 10)\\&= - 7x^{2} - 10x - 4
\end{align*}
C'est un polynôme de degré 2 avec $a = - 7$, $b = - 10$ et $c = - 4$.
\item
\begin{align*}
i(x) &= - 4x^{2} + x(- 8x - 1)\\&= - 4x^{2} + x \times - 8x + x(- 1)\\&= - 4x^{2} - 8x^{2} - x\\&= - 4x^{2} - 8x^{2} - x\\&= (- 4 - 8) \times x^{2} - x\\&= - 12x^{2} - x
\end{align*}
C'est un polynôme de degré 2 avec $a = - 12$, $b = - 1$ et $c = 0$.
\item
\begin{align*}
j(x) &= - 3(x + 6)(x + 8)\\&= (- 3x - 3 \times 6)(x + 8)\\&= (- 3x - 18)(x + 8)\\&= - 3x \times x - 3x \times 8 - 18x - 18 \times 8\\&= 8(- 3) \times x - 144 - 3x^{2} - 18x\\&= - 24x - 144 - 3x^{2} - 18x\\&= - 3x^{2} - 24x - 18x - 144\\&= - 3x^{2} + (- 24 - 18) \times x - 144\\&= - 3x^{2} - 42x - 144
\end{align*}
C'est un polynôme de degré 2 avec $a = - 3$, $b = - 42$ et $c = - 144$.
\item
\begin{align*}
k(x) &= 6(x + 5)(x - 10)\\&= (6x + 6 \times 5)(x - 10)\\&= (6x + 30)(x - 10)\\&= 6x \times x + 6x(- 10) + 30x + 30(- 10)\\&= - 10 \times 6 \times x - 300 + 6x^{2} + 30x\\&= - 60x - 300 + 6x^{2} + 30x\\&= 6x^{2} - 60x + 30x - 300\\&= 6x^{2} + (- 60 + 30) \times x - 300\\&= 6x^{2} - 30x - 300
\end{align*}
C'est un polynôme de degré 2 avec $a = 6$, $b = - 30$ et $c = - 300$.
\end{enumerate}
\end{solution}
\begin{exercise}[subtitle={Étude de fonctions}]
Soit $f(x) = - 2x^{2} + 32x - 120$ une fonction définie sur $\R$.
\begin{enumerate}
\item Calculer les valeurs suivantes
\[
f(1) \qquad f(-2)
\]
\item Dériver la fonction $f$
\item Étudier le signe de $f'$ puis en déduire les variations de $f$.
\item Est-ce que $f$ admet un maximum? un minimum? Calculer sa valeur.
\end{enumerate}
\end{exercise}
\begin{solution}
\begin{enumerate}
\item On remplace $x$ par les valeurs demandées
\[
f(1) = - 2 \times 1^{2} + 32 \times 1 - 120=- 2 \times 1 + 32 - 120=- 2 - 88=- 90
\]
\[
f(-1) = - 2 \times - 1^{2} + 32(- 1) - 120=- 2 \times 1 - 32 - 120=- 2 - 152=- 154
\]
\item Dérivation
\[
f'(x) = - 4x + 32
\]
\item Pas de solutions automatiques.
\item Pas de solutions automatiques.
\end{enumerate}
\end{solution}
\begin{exercise}[subtitle={Enclos}]
Dans son garage, Jean a trouvé 24m de grillage. \\
Il décide de l'utiliser pour faire un enclos rectangulaire. Afin d'obtenir un enclos encore plus grand, il veut utiliser le mur du jardin qui formera un côté. Le grillage formera les 3 autres.
\begin{center}
\includegraphics[scale=0.8]{./fig/enclos}
\end{center}
Comment placer les poteaux pour avoir un enclos le plus grand possible ?
\textit{Cet exercice est un exercice de recherche. Vous êtes invité à utiliser les outils que vous voulez. Une part importante de la note sera dédiée à la rédaction et aux explications de ce que vous faites. Vous êtes encouragé à faire des schémas.}
\textit{Si vous utilisez le tableur, vous devrez le mentionner et m'envoyer le tableur à l'adresse \url{benjamin.bertrand@ac-lyon.fr}}
\end{exercise}
\begin{solution}
Formule de l'aire (en notant $x$ la longueur du côté de l'enclos à côté du mur)
\[
A(x) = x(24 - 2x) = - 2x^{2} + 24x
\]
On va donc étudier les variations de la fonction $A(x) = - 2x^{2} + 24x$
\begin{itemize}
\item Fonction dérivée : $A'(x) = - 4x + 24$
\item On résout l'inéquation $A'(x) \geq 0$ pour déterminer quand la fonction $A'$ est positive.
\begin{align*}
A(x) & \geq 0 \\
- 4x + 24 & \geq 0 \\
- 4x + 24 + - 24 &\geq 0 + - 24 \\
- 4x &\geq - 24 \\
\frac{- 4x}{- 4} &\leq \frac{- 24}{- 4} \\
x &\leq 6 \\
\end{align*}
Donc $A(x)$ est positif quand $x$ est plus \textbf{petit} que $6$
\item
\begin{center}
\begin{tikzpicture}
\tkzTabInit[lgt=3,espcl=4]{$x$/1,Signe de $f'(x)$/2, Variations de $f(x)$/2}{, $6$ ,}%
\tkzTabLine{, +, z, -, }
\tkzTabVar{-/ ,+/$f(6) = 72$ , -/}%
\end{tikzpicture}
\end{center}
\end{itemize}
Pour avoir le plus grand enclos, il faut placer les poteaux à 6m du mur et on aura alors un enclos de $72m^2$.
\end{solution}
\end{document}
%%% Local Variables:
%%% mode: latex
%%% TeX-master: "master"
%%% End:

Binary file not shown.

View File

@ -0,0 +1,158 @@
\documentclass[a4paper,12pt]{article}
\usepackage{myXsim}
\usepackage{pgfplots}
\usetikzlibrary{decorations.markings}
\pgfplotsset{compat=1.18}
\title{ DM1 \hfill BADEL Melinda}
\tribe{1ST}
\date{A rendre pour le lundi 27 mars 2023}
\duree{}
\xsimsetup{
solution/print = false
}
\pagestyle{empty}
\begin{document}
\maketitle
Le barème est donné à titre indicatif, il pourra être modifié.
\begin{exercise}[subtitle={Polynôme de degré 2}]
Développer les expressions suivantes pour vérifier que ce sont des polynômes de degré 2. Vous préciserez les valeurs de $a$, $b$ et $c$.
\begin{multicols}{2}
\begin{enumerate}
\item $f(x) = (4x - 9)(5x - 9)$
\item $g(x) = (- 4x - 2)^{2}$
\item $h(x) = 5 + x(- 8x - 6)$
\item $i(x) = - 9x^{2} + x(8x - 2)$
\item $j(x) = - 5(x - 3)(x - 4)$
\item $k(x) = 3(x - 2)(x + 10)$
\end{enumerate}
\end{multicols}
\end{exercise}
\begin{solution}
\begin{enumerate}
\item
\begin{align*}
f(x) &= (4x - 9)(5x - 9)\\&= 4x \times 5x + 4x(- 9) - 9 \times 5x - 9(- 9)\\&= 4 \times 5 \times x^{1 + 1} - 9 \times 4 \times x - 9 \times 5 \times x + 81\\&= - 36x - 45x + 20x^{2} + 81\\&= (- 36 - 45) \times x + 20x^{2} + 81\\&= 20x^{2} - 81x + 81
\end{align*}
C'est un polynôme de degré 2 avec $a = 20$, $b = - 81$ et $c = 81$.
\item
\begin{align*}
g(x) &= (- 4x - 2)^{2}\\&= (- 4x - 2)(- 4x - 2)\\&= - 4x \times - 4x - 4x(- 2) - 2 \times - 4x - 2(- 2)\\&= - 4(- 4) \times x^{1 + 1} - 2(- 4) \times x - 2(- 4) \times x + 4\\&= 8x + 8x + 16x^{2} + 4\\&= (8 + 8) \times x + 16x^{2} + 4\\&= 16x^{2} + 16x + 4
\end{align*}
C'est un polynôme de degré 2 avec $a = 16$, $b = 16$ et $c = 4$.
\item
\begin{align*}
h(x) &= 5 + x(- 8x - 6)\\&= 5 + x \times - 8x + x(- 6)\\&= - 8x^{2} - 6x + 5
\end{align*}
C'est un polynôme de degré 2 avec $a = - 8$, $b = - 6$ et $c = 5$.
\item
\begin{align*}
i(x) &= - 9x^{2} + x(8x - 2)\\&= - 9x^{2} + x \times 8x + x(- 2)\\&= - 9x^{2} + 8x^{2} - 2x\\&= - 9x^{2} + 8x^{2} - 2x\\&= (- 9 + 8) \times x^{2} - 2x\\&= - x^{2} - 2x
\end{align*}
C'est un polynôme de degré 2 avec $a = - 1$, $b = - 2$ et $c = 0$.
\item
\begin{align*}
j(x) &= - 5(x - 3)(x - 4)\\&= (- 5x - 5(- 3))(x - 4)\\&= (- 5x + 15)(x - 4)\\&= - 5x \times x - 5x(- 4) + 15x + 15(- 4)\\&= - 4(- 5) \times x - 60 - 5x^{2} + 15x\\&= 20x - 60 - 5x^{2} + 15x\\&= - 5x^{2} + 20x + 15x - 60\\&= - 5x^{2} + (20 + 15) \times x - 60\\&= - 5x^{2} + 35x - 60
\end{align*}
C'est un polynôme de degré 2 avec $a = - 5$, $b = 35$ et $c = - 60$.
\item
\begin{align*}
k(x) &= 3(x - 2)(x + 10)\\&= (3x + 3(- 2))(x + 10)\\&= (3x - 6)(x + 10)\\&= 3x \times x + 3x \times 10 - 6x - 6 \times 10\\&= 10 \times 3 \times x - 60 + 3x^{2} - 6x\\&= 30x - 60 + 3x^{2} - 6x\\&= 3x^{2} + 30x - 6x - 60\\&= 3x^{2} + (30 - 6) \times x - 60\\&= 3x^{2} + 24x - 60
\end{align*}
C'est un polynôme de degré 2 avec $a = 3$, $b = 24$ et $c = - 60$.
\end{enumerate}
\end{solution}
\begin{exercise}[subtitle={Étude de fonctions}]
Soit $f(x) = 4x^{2} + 20x - 56$ une fonction définie sur $\R$.
\begin{enumerate}
\item Calculer les valeurs suivantes
\[
f(1) \qquad f(-2)
\]
\item Dériver la fonction $f$
\item Étudier le signe de $f'$ puis en déduire les variations de $f$.
\item Est-ce que $f$ admet un maximum? un minimum? Calculer sa valeur.
\end{enumerate}
\end{exercise}
\begin{solution}
\begin{enumerate}
\item On remplace $x$ par les valeurs demandées
\[
f(1) = 4 \times 1^{2} + 20 \times 1 - 56=4 \times 1 + 20 - 56=4 - 36=- 32
\]
\[
f(-1) = 4 \times - 1^{2} + 20(- 1) - 56=4 \times 1 - 20 - 56=4 - 76=- 72
\]
\item Dérivation
\[
f'(x) = 8x + 20
\]
\item Pas de solutions automatiques.
\item Pas de solutions automatiques.
\end{enumerate}
\end{solution}
\begin{exercise}[subtitle={Enclos}]
Dans son garage, Jean a trouvé 37m de grillage. \\
Il décide de l'utiliser pour faire un enclos rectangulaire. Afin d'obtenir un enclos encore plus grand, il veut utiliser le mur du jardin qui formera un côté. Le grillage formera les 3 autres.
\begin{center}
\includegraphics[scale=0.8]{./fig/enclos}
\end{center}
Comment placer les poteaux pour avoir un enclos le plus grand possible ?
\textit{Cet exercice est un exercice de recherche. Vous êtes invité à utiliser les outils que vous voulez. Une part importante de la note sera dédiée à la rédaction et aux explications de ce que vous faites. Vous êtes encouragé à faire des schémas.}
\textit{Si vous utilisez le tableur, vous devrez le mentionner et m'envoyer le tableur à l'adresse \url{benjamin.bertrand@ac-lyon.fr}}
\end{exercise}
\begin{solution}
Formule de l'aire (en notant $x$ la longueur du côté de l'enclos à côté du mur)
\[
A(x) = x(37 - 2x) = - 2x^{2} + 37x
\]
On va donc étudier les variations de la fonction $A(x) = - 2x^{2} + 37x$
\begin{itemize}
\item Fonction dérivée : $A'(x) = - 4x + 37$
\item On résout l'inéquation $A'(x) \geq 0$ pour déterminer quand la fonction $A'$ est positive.
\begin{align*}
A(x) & \geq 0 \\
- 4x + 37 & \geq 0 \\
- 4x + 37 + - 37 &\geq 0 + - 37 \\
- 4x &\geq - 37 \\
\frac{- 4x}{- 4} &\leq \frac{- 37}{- 4} \\
x &\leq \dfrac{37}{4} \\
\end{align*}
Donc $A(x)$ est positif quand $x$ est plus \textbf{petit} que $\dfrac{37}{4}$
\item
\begin{center}
\begin{tikzpicture}
\tkzTabInit[lgt=3,espcl=4]{$x$/1,Signe de $f'(x)$/2, Variations de $f(x)$/2}{, $\dfrac{37}{4}$ ,}%
\tkzTabLine{, +, z, -, }
\tkzTabVar{-/ ,+/$f(\dfrac{37}{4}) = \dfrac{2738}{16}$ , -/}%
\end{tikzpicture}
\end{center}
\end{itemize}
Pour avoir le plus grand enclos, il faut placer les poteaux à 6m du mur et on aura alors un enclos de $72m^2$.
\end{solution}
\end{document}
%%% Local Variables:
%%% mode: latex
%%% TeX-master: "master"
%%% End:

Binary file not shown.

View File

@ -0,0 +1,158 @@
\documentclass[a4paper,12pt]{article}
\usepackage{myXsim}
\usepackage{pgfplots}
\usetikzlibrary{decorations.markings}
\pgfplotsset{compat=1.18}
\title{ DM1 \hfill BALTA Zina}
\tribe{1ST}
\date{A rendre pour le lundi 27 mars 2023}
\duree{}
\xsimsetup{
solution/print = false
}
\pagestyle{empty}
\begin{document}
\maketitle
Le barème est donné à titre indicatif, il pourra être modifié.
\begin{exercise}[subtitle={Polynôme de degré 2}]
Développer les expressions suivantes pour vérifier que ce sont des polynômes de degré 2. Vous préciserez les valeurs de $a$, $b$ et $c$.
\begin{multicols}{2}
\begin{enumerate}
\item $f(x) = (- 4x - 9)(2x - 9)$
\item $g(x) = (9x - 9)^{2}$
\item $h(x) = 4 + x(8x - 1)$
\item $i(x) = 9x^{2} + x(9x + 6)$
\item $j(x) = - 5(x + 4)(x - 2)$
\item $k(x) = - 9(x - 3)(x + 4)$
\end{enumerate}
\end{multicols}
\end{exercise}
\begin{solution}
\begin{enumerate}
\item
\begin{align*}
f(x) &= (- 4x - 9)(2x - 9)\\&= - 4x \times 2x - 4x(- 9) - 9 \times 2x - 9(- 9)\\&= - 4 \times 2 \times x^{1 + 1} - 9(- 4) \times x - 9 \times 2 \times x + 81\\&= 36x - 18x - 8x^{2} + 81\\&= (36 - 18) \times x - 8x^{2} + 81\\&= - 8x^{2} + 18x + 81
\end{align*}
C'est un polynôme de degré 2 avec $a = - 8$, $b = 18$ et $c = 81$.
\item
\begin{align*}
g(x) &= (9x - 9)^{2}\\&= (9x - 9)(9x - 9)\\&= 9x \times 9x + 9x(- 9) - 9 \times 9x - 9(- 9)\\&= 9 \times 9 \times x^{1 + 1} - 9 \times 9 \times x - 9 \times 9 \times x + 81\\&= - 81x - 81x + 81x^{2} + 81\\&= (- 81 - 81) \times x + 81x^{2} + 81\\&= 81x^{2} - 162x + 81
\end{align*}
C'est un polynôme de degré 2 avec $a = 81$, $b = - 162$ et $c = 81$.
\item
\begin{align*}
h(x) &= 4 + x(8x - 1)\\&= 4 + x \times 8x + x(- 1)\\&= 8x^{2} - x + 4
\end{align*}
C'est un polynôme de degré 2 avec $a = 8$, $b = - 1$ et $c = 4$.
\item
\begin{align*}
i(x) &= 9x^{2} + x(9x + 6)\\&= 9x^{2} + x \times 9x + x \times 6\\&= 9x^{2} + 9x^{2} + 6x\\&= 9x^{2} + 9x^{2} + 6x\\&= (9 + 9) \times x^{2} + 6x\\&= 18x^{2} + 6x
\end{align*}
C'est un polynôme de degré 2 avec $a = 18$, $b = 6$ et $c = 0$.
\item
\begin{align*}
j(x) &= - 5(x + 4)(x - 2)\\&= (- 5x - 5 \times 4)(x - 2)\\&= (- 5x - 20)(x - 2)\\&= - 5x \times x - 5x(- 2) - 20x - 20(- 2)\\&= - 2(- 5) \times x + 40 - 5x^{2} - 20x\\&= 10x + 40 - 5x^{2} - 20x\\&= - 5x^{2} + 10x - 20x + 40\\&= - 5x^{2} + (10 - 20) \times x + 40\\&= - 5x^{2} - 10x + 40
\end{align*}
C'est un polynôme de degré 2 avec $a = - 5$, $b = - 10$ et $c = 40$.
\item
\begin{align*}
k(x) &= - 9(x - 3)(x + 4)\\&= (- 9x - 9(- 3))(x + 4)\\&= (- 9x + 27)(x + 4)\\&= - 9x \times x - 9x \times 4 + 27x + 27 \times 4\\&= 4(- 9) \times x + 108 - 9x^{2} + 27x\\&= - 36x + 108 - 9x^{2} + 27x\\&= - 9x^{2} - 36x + 27x + 108\\&= - 9x^{2} + (- 36 + 27) \times x + 108\\&= - 9x^{2} - 9x + 108
\end{align*}
C'est un polynôme de degré 2 avec $a = - 9$, $b = - 9$ et $c = 108$.
\end{enumerate}
\end{solution}
\begin{exercise}[subtitle={Étude de fonctions}]
Soit $f(x) = 5x^{2} + 70x + 200$ une fonction définie sur $\R$.
\begin{enumerate}
\item Calculer les valeurs suivantes
\[
f(1) \qquad f(-2)
\]
\item Dériver la fonction $f$
\item Étudier le signe de $f'$ puis en déduire les variations de $f$.
\item Est-ce que $f$ admet un maximum? un minimum? Calculer sa valeur.
\end{enumerate}
\end{exercise}
\begin{solution}
\begin{enumerate}
\item On remplace $x$ par les valeurs demandées
\[
f(1) = 5 \times 1^{2} + 70 \times 1 + 200=5 \times 1 + 70 + 200=5 + 270=275
\]
\[
f(-1) = 5 \times - 1^{2} + 70(- 1) + 200=5 \times 1 - 70 + 200=5 + 130=135
\]
\item Dérivation
\[
f'(x) = 10x + 70
\]
\item Pas de solutions automatiques.
\item Pas de solutions automatiques.
\end{enumerate}
\end{solution}
\begin{exercise}[subtitle={Enclos}]
Dans son garage, Jean a trouvé 32m de grillage. \\
Il décide de l'utiliser pour faire un enclos rectangulaire. Afin d'obtenir un enclos encore plus grand, il veut utiliser le mur du jardin qui formera un côté. Le grillage formera les 3 autres.
\begin{center}
\includegraphics[scale=0.8]{./fig/enclos}
\end{center}
Comment placer les poteaux pour avoir un enclos le plus grand possible ?
\textit{Cet exercice est un exercice de recherche. Vous êtes invité à utiliser les outils que vous voulez. Une part importante de la note sera dédiée à la rédaction et aux explications de ce que vous faites. Vous êtes encouragé à faire des schémas.}
\textit{Si vous utilisez le tableur, vous devrez le mentionner et m'envoyer le tableur à l'adresse \url{benjamin.bertrand@ac-lyon.fr}}
\end{exercise}
\begin{solution}
Formule de l'aire (en notant $x$ la longueur du côté de l'enclos à côté du mur)
\[
A(x) = x(32 - 2x) = - 2x^{2} + 32x
\]
On va donc étudier les variations de la fonction $A(x) = - 2x^{2} + 32x$
\begin{itemize}
\item Fonction dérivée : $A'(x) = - 4x + 32$
\item On résout l'inéquation $A'(x) \geq 0$ pour déterminer quand la fonction $A'$ est positive.
\begin{align*}
A(x) & \geq 0 \\
- 4x + 32 & \geq 0 \\
- 4x + 32 + - 32 &\geq 0 + - 32 \\
- 4x &\geq - 32 \\
\frac{- 4x}{- 4} &\leq \frac{- 32}{- 4} \\
x &\leq 8 \\
\end{align*}
Donc $A(x)$ est positif quand $x$ est plus \textbf{petit} que $8$
\item
\begin{center}
\begin{tikzpicture}
\tkzTabInit[lgt=3,espcl=4]{$x$/1,Signe de $f'(x)$/2, Variations de $f(x)$/2}{, $8$ ,}%
\tkzTabLine{, +, z, -, }
\tkzTabVar{-/ ,+/$f(8) = 128$ , -/}%
\end{tikzpicture}
\end{center}
\end{itemize}
Pour avoir le plus grand enclos, il faut placer les poteaux à 6m du mur et on aura alors un enclos de $72m^2$.
\end{solution}
\end{document}
%%% Local Variables:
%%% mode: latex
%%% TeX-master: "master"
%%% End:

Binary file not shown.

View File

@ -0,0 +1,158 @@
\documentclass[a4paper,12pt]{article}
\usepackage{myXsim}
\usepackage{pgfplots}
\usetikzlibrary{decorations.markings}
\pgfplotsset{compat=1.18}
\title{ DM1 \hfill BARDOUSSE Yanis}
\tribe{1ST}
\date{A rendre pour le lundi 27 mars 2023}
\duree{}
\xsimsetup{
solution/print = false
}
\pagestyle{empty}
\begin{document}
\maketitle
Le barème est donné à titre indicatif, il pourra être modifié.
\begin{exercise}[subtitle={Polynôme de degré 2}]
Développer les expressions suivantes pour vérifier que ce sont des polynômes de degré 2. Vous préciserez les valeurs de $a$, $b$ et $c$.
\begin{multicols}{2}
\begin{enumerate}
\item $f(x) = (5x - 2)(- 10x - 2)$
\item $g(x) = (- 9x - 2)^{2}$
\item $h(x) = 8 + x(- 2x - 1)$
\item $i(x) = - 10x^{2} + x(- 5x + 4)$
\item $j(x) = - 9(x - 1)(x - 7)$
\item $k(x) = - 10(x + 7)(x + 9)$
\end{enumerate}
\end{multicols}
\end{exercise}
\begin{solution}
\begin{enumerate}
\item
\begin{align*}
f(x) &= (5x - 2)(- 10x - 2)\\&= 5x \times - 10x + 5x(- 2) - 2 \times - 10x - 2(- 2)\\&= 5(- 10) \times x^{1 + 1} - 2 \times 5 \times x - 2(- 10) \times x + 4\\&= - 10x + 20x - 50x^{2} + 4\\&= (- 10 + 20) \times x - 50x^{2} + 4\\&= - 50x^{2} + 10x + 4
\end{align*}
C'est un polynôme de degré 2 avec $a = - 50$, $b = 10$ et $c = 4$.
\item
\begin{align*}
g(x) &= (- 9x - 2)^{2}\\&= (- 9x - 2)(- 9x - 2)\\&= - 9x \times - 9x - 9x(- 2) - 2 \times - 9x - 2(- 2)\\&= - 9(- 9) \times x^{1 + 1} - 2(- 9) \times x - 2(- 9) \times x + 4\\&= 18x + 18x + 81x^{2} + 4\\&= (18 + 18) \times x + 81x^{2} + 4\\&= 81x^{2} + 36x + 4
\end{align*}
C'est un polynôme de degré 2 avec $a = 81$, $b = 36$ et $c = 4$.
\item
\begin{align*}
h(x) &= 8 + x(- 2x - 1)\\&= 8 + x \times - 2x + x(- 1)\\&= - 2x^{2} - x + 8
\end{align*}
C'est un polynôme de degré 2 avec $a = - 2$, $b = - 1$ et $c = 8$.
\item
\begin{align*}
i(x) &= - 10x^{2} + x(- 5x + 4)\\&= - 10x^{2} + x \times - 5x + x \times 4\\&= - 10x^{2} - 5x^{2} + 4x\\&= - 10x^{2} - 5x^{2} + 4x\\&= (- 10 - 5) \times x^{2} + 4x\\&= - 15x^{2} + 4x
\end{align*}
C'est un polynôme de degré 2 avec $a = - 15$, $b = 4$ et $c = 0$.
\item
\begin{align*}
j(x) &= - 9(x - 1)(x - 7)\\&= (- 9x - 9(- 1))(x - 7)\\&= (- 9x + 9)(x - 7)\\&= - 9x \times x - 9x(- 7) + 9x + 9(- 7)\\&= - 7(- 9) \times x - 63 - 9x^{2} + 9x\\&= 63x - 63 - 9x^{2} + 9x\\&= - 9x^{2} + 63x + 9x - 63\\&= - 9x^{2} + (63 + 9) \times x - 63\\&= - 9x^{2} + 72x - 63
\end{align*}
C'est un polynôme de degré 2 avec $a = - 9$, $b = 72$ et $c = - 63$.
\item
\begin{align*}
k(x) &= - 10(x + 7)(x + 9)\\&= (- 10x - 10 \times 7)(x + 9)\\&= (- 10x - 70)(x + 9)\\&= - 10x \times x - 10x \times 9 - 70x - 70 \times 9\\&= 9(- 10) \times x - 630 - 10x^{2} - 70x\\&= - 90x - 630 - 10x^{2} - 70x\\&= - 10x^{2} - 90x - 70x - 630\\&= - 10x^{2} + (- 90 - 70) \times x - 630\\&= - 10x^{2} - 160x - 630
\end{align*}
C'est un polynôme de degré 2 avec $a = - 10$, $b = - 160$ et $c = - 630$.
\end{enumerate}
\end{solution}
\begin{exercise}[subtitle={Étude de fonctions}]
Soit $f(x) = 7x^{2} - 77x + 168$ une fonction définie sur $\R$.
\begin{enumerate}
\item Calculer les valeurs suivantes
\[
f(1) \qquad f(-2)
\]
\item Dériver la fonction $f$
\item Étudier le signe de $f'$ puis en déduire les variations de $f$.
\item Est-ce que $f$ admet un maximum? un minimum? Calculer sa valeur.
\end{enumerate}
\end{exercise}
\begin{solution}
\begin{enumerate}
\item On remplace $x$ par les valeurs demandées
\[
f(1) = 7 \times 1^{2} - 77 \times 1 + 168=7 \times 1 - 77 + 168=7 + 91=98
\]
\[
f(-1) = 7 \times - 1^{2} - 77(- 1) + 168=7 \times 1 + 77 + 168=7 + 245=252
\]
\item Dérivation
\[
f'(x) = 14x - 77
\]
\item Pas de solutions automatiques.
\item Pas de solutions automatiques.
\end{enumerate}
\end{solution}
\begin{exercise}[subtitle={Enclos}]
Dans son garage, Jean a trouvé 34m de grillage. \\
Il décide de l'utiliser pour faire un enclos rectangulaire. Afin d'obtenir un enclos encore plus grand, il veut utiliser le mur du jardin qui formera un côté. Le grillage formera les 3 autres.
\begin{center}
\includegraphics[scale=0.8]{./fig/enclos}
\end{center}
Comment placer les poteaux pour avoir un enclos le plus grand possible ?
\textit{Cet exercice est un exercice de recherche. Vous êtes invité à utiliser les outils que vous voulez. Une part importante de la note sera dédiée à la rédaction et aux explications de ce que vous faites. Vous êtes encouragé à faire des schémas.}
\textit{Si vous utilisez le tableur, vous devrez le mentionner et m'envoyer le tableur à l'adresse \url{benjamin.bertrand@ac-lyon.fr}}
\end{exercise}
\begin{solution}
Formule de l'aire (en notant $x$ la longueur du côté de l'enclos à côté du mur)
\[
A(x) = x(34 - 2x) = - 2x^{2} + 34x
\]
On va donc étudier les variations de la fonction $A(x) = - 2x^{2} + 34x$
\begin{itemize}
\item Fonction dérivée : $A'(x) = - 4x + 34$
\item On résout l'inéquation $A'(x) \geq 0$ pour déterminer quand la fonction $A'$ est positive.
\begin{align*}
A(x) & \geq 0 \\
- 4x + 34 & \geq 0 \\
- 4x + 34 + - 34 &\geq 0 + - 34 \\
- 4x &\geq - 34 \\
\frac{- 4x}{- 4} &\leq \frac{- 34}{- 4} \\
x &\leq \dfrac{17}{2} \\
\end{align*}
Donc $A(x)$ est positif quand $x$ est plus \textbf{petit} que $\dfrac{17}{2}$
\item
\begin{center}
\begin{tikzpicture}
\tkzTabInit[lgt=3,espcl=4]{$x$/1,Signe de $f'(x)$/2, Variations de $f(x)$/2}{, $\dfrac{17}{2}$ ,}%
\tkzTabLine{, +, z, -, }
\tkzTabVar{-/ ,+/$f(\dfrac{17}{2}) = \dfrac{578}{4}$ , -/}%
\end{tikzpicture}
\end{center}
\end{itemize}
Pour avoir le plus grand enclos, il faut placer les poteaux à 6m du mur et on aura alors un enclos de $72m^2$.
\end{solution}
\end{document}
%%% Local Variables:
%%% mode: latex
%%% TeX-master: "master"
%%% End:

Binary file not shown.

View File

@ -0,0 +1,158 @@
\documentclass[a4paper,12pt]{article}
\usepackage{myXsim}
\usepackage{pgfplots}
\usetikzlibrary{decorations.markings}
\pgfplotsset{compat=1.18}
\title{ DM1 \hfill BLONDIN Damien}
\tribe{1ST}
\date{A rendre pour le lundi 27 mars 2023}
\duree{}
\xsimsetup{
solution/print = false
}
\pagestyle{empty}
\begin{document}
\maketitle
Le barème est donné à titre indicatif, il pourra être modifié.
\begin{exercise}[subtitle={Polynôme de degré 2}]
Développer les expressions suivantes pour vérifier que ce sont des polynômes de degré 2. Vous préciserez les valeurs de $a$, $b$ et $c$.
\begin{multicols}{2}
\begin{enumerate}
\item $f(x) = (- 4x + 4)(10x + 4)$
\item $g(x) = (4x + 6)^{2}$
\item $h(x) = 6 + x(3x + 4)$
\item $i(x) = - 2x^{2} + x(- 7x - 3)$
\item $j(x) = - 3(x + 9)(x + 10)$
\item $k(x) = - 1(x - 6)(x + 8)$
\end{enumerate}
\end{multicols}
\end{exercise}
\begin{solution}
\begin{enumerate}
\item
\begin{align*}
f(x) &= (- 4x + 4)(10x + 4)\\&= - 4x \times 10x - 4x \times 4 + 4 \times 10x + 4 \times 4\\&= - 4 \times 10 \times x^{1 + 1} + 4(- 4) \times x + 4 \times 10 \times x + 16\\&= - 16x + 40x - 40x^{2} + 16\\&= (- 16 + 40) \times x - 40x^{2} + 16\\&= - 40x^{2} + 24x + 16
\end{align*}
C'est un polynôme de degré 2 avec $a = - 40$, $b = 24$ et $c = 16$.
\item
\begin{align*}
g(x) &= (4x + 6)^{2}\\&= (4x + 6)(4x + 6)\\&= 4x \times 4x + 4x \times 6 + 6 \times 4x + 6 \times 6\\&= 4 \times 4 \times x^{1 + 1} + 6 \times 4 \times x + 6 \times 4 \times x + 36\\&= 24x + 24x + 16x^{2} + 36\\&= (24 + 24) \times x + 16x^{2} + 36\\&= 16x^{2} + 48x + 36
\end{align*}
C'est un polynôme de degré 2 avec $a = 16$, $b = 48$ et $c = 36$.
\item
\begin{align*}
h(x) &= 6 + x(3x + 4)\\&= 6 + x \times 3x + x \times 4\\&= 3x^{2} + 4x + 6
\end{align*}
C'est un polynôme de degré 2 avec $a = 3$, $b = 4$ et $c = 6$.
\item
\begin{align*}
i(x) &= - 2x^{2} + x(- 7x - 3)\\&= - 2x^{2} + x \times - 7x + x(- 3)\\&= - 2x^{2} - 7x^{2} - 3x\\&= - 2x^{2} - 7x^{2} - 3x\\&= (- 2 - 7) \times x^{2} - 3x\\&= - 9x^{2} - 3x
\end{align*}
C'est un polynôme de degré 2 avec $a = - 9$, $b = - 3$ et $c = 0$.
\item
\begin{align*}
j(x) &= - 3(x + 9)(x + 10)\\&= (- 3x - 3 \times 9)(x + 10)\\&= (- 3x - 27)(x + 10)\\&= - 3x \times x - 3x \times 10 - 27x - 27 \times 10\\&= 10(- 3) \times x - 270 - 3x^{2} - 27x\\&= - 30x - 270 - 3x^{2} - 27x\\&= - 3x^{2} - 30x - 27x - 270\\&= - 3x^{2} + (- 30 - 27) \times x - 270\\&= - 3x^{2} - 57x - 270
\end{align*}
C'est un polynôme de degré 2 avec $a = - 3$, $b = - 57$ et $c = - 270$.
\item
\begin{align*}
k(x) &= - 1(x - 6)(x + 8)\\&= (- 1x - 1(- 6))(x + 8)\\&= (- x + 6)(x + 8)\\&= (- x) \times x + (- x) \times 8 + 6x + 6 \times 8\\&= 8(- 1) \times x + 48 - x^{2} + 6x\\&= - 8x + 48 - x^{2} + 6x\\&= - x^{2} - 8x + 6x + 48\\&= - x^{2} + (- 8 + 6) \times x + 48\\&= - x^{2} - 2x + 48
\end{align*}
C'est un polynôme de degré 2 avec $a = - 1$, $b = - 2$ et $c = 48$.
\end{enumerate}
\end{solution}
\begin{exercise}[subtitle={Étude de fonctions}]
Soit $f(x) = 10x^{2} + 60x + 50$ une fonction définie sur $\R$.
\begin{enumerate}
\item Calculer les valeurs suivantes
\[
f(1) \qquad f(-2)
\]
\item Dériver la fonction $f$
\item Étudier le signe de $f'$ puis en déduire les variations de $f$.
\item Est-ce que $f$ admet un maximum? un minimum? Calculer sa valeur.
\end{enumerate}
\end{exercise}
\begin{solution}
\begin{enumerate}
\item On remplace $x$ par les valeurs demandées
\[
f(1) = 10 \times 1^{2} + 60 \times 1 + 50=10 \times 1 + 60 + 50=10 + 110=120
\]
\[
f(-1) = 10 \times - 1^{2} + 60(- 1) + 50=10 \times 1 - 60 + 50=10 - 10=0
\]
\item Dérivation
\[
f'(x) = 20x + 60
\]
\item Pas de solutions automatiques.
\item Pas de solutions automatiques.
\end{enumerate}
\end{solution}
\begin{exercise}[subtitle={Enclos}]
Dans son garage, Jean a trouvé 30m de grillage. \\
Il décide de l'utiliser pour faire un enclos rectangulaire. Afin d'obtenir un enclos encore plus grand, il veut utiliser le mur du jardin qui formera un côté. Le grillage formera les 3 autres.
\begin{center}
\includegraphics[scale=0.8]{./fig/enclos}
\end{center}
Comment placer les poteaux pour avoir un enclos le plus grand possible ?
\textit{Cet exercice est un exercice de recherche. Vous êtes invité à utiliser les outils que vous voulez. Une part importante de la note sera dédiée à la rédaction et aux explications de ce que vous faites. Vous êtes encouragé à faire des schémas.}
\textit{Si vous utilisez le tableur, vous devrez le mentionner et m'envoyer le tableur à l'adresse \url{benjamin.bertrand@ac-lyon.fr}}
\end{exercise}
\begin{solution}
Formule de l'aire (en notant $x$ la longueur du côté de l'enclos à côté du mur)
\[
A(x) = x(30 - 2x) = - 2x^{2} + 30x
\]
On va donc étudier les variations de la fonction $A(x) = - 2x^{2} + 30x$
\begin{itemize}
\item Fonction dérivée : $A'(x) = - 4x + 30$
\item On résout l'inéquation $A'(x) \geq 0$ pour déterminer quand la fonction $A'$ est positive.
\begin{align*}
A(x) & \geq 0 \\
- 4x + 30 & \geq 0 \\
- 4x + 30 + - 30 &\geq 0 + - 30 \\
- 4x &\geq - 30 \\
\frac{- 4x}{- 4} &\leq \frac{- 30}{- 4} \\
x &\leq \dfrac{15}{2} \\
\end{align*}
Donc $A(x)$ est positif quand $x$ est plus \textbf{petit} que $\dfrac{15}{2}$
\item
\begin{center}
\begin{tikzpicture}
\tkzTabInit[lgt=3,espcl=4]{$x$/1,Signe de $f'(x)$/2, Variations de $f(x)$/2}{, $\dfrac{15}{2}$ ,}%
\tkzTabLine{, +, z, -, }
\tkzTabVar{-/ ,+/$f(\dfrac{15}{2}) = \dfrac{450}{4}$ , -/}%
\end{tikzpicture}
\end{center}
\end{itemize}
Pour avoir le plus grand enclos, il faut placer les poteaux à 6m du mur et on aura alors un enclos de $72m^2$.
\end{solution}
\end{document}
%%% Local Variables:
%%% mode: latex
%%% TeX-master: "master"
%%% End:

Binary file not shown.

View File

@ -0,0 +1,158 @@
\documentclass[a4paper,12pt]{article}
\usepackage{myXsim}
\usepackage{pgfplots}
\usetikzlibrary{decorations.markings}
\pgfplotsset{compat=1.18}
\title{ DM1 \hfill BOUAFIA Lina}
\tribe{1ST}
\date{A rendre pour le lundi 27 mars 2023}
\duree{}
\xsimsetup{
solution/print = false
}
\pagestyle{empty}
\begin{document}
\maketitle
Le barème est donné à titre indicatif, il pourra être modifié.
\begin{exercise}[subtitle={Polynôme de degré 2}]
Développer les expressions suivantes pour vérifier que ce sont des polynômes de degré 2. Vous préciserez les valeurs de $a$, $b$ et $c$.
\begin{multicols}{2}
\begin{enumerate}
\item $f(x) = (4x - 9)(- 8x - 9)$
\item $g(x) = (5x - 4)^{2}$
\item $h(x) = - 1 + x(- 9x + 8)$
\item $i(x) = 9x^{2} + x(2x + 10)$
\item $j(x) = 9(x - 8)(x + 5)$
\item $k(x) = - 9(x - 6)(x + 7)$
\end{enumerate}
\end{multicols}
\end{exercise}
\begin{solution}
\begin{enumerate}
\item
\begin{align*}
f(x) &= (4x - 9)(- 8x - 9)\\&= 4x \times - 8x + 4x(- 9) - 9 \times - 8x - 9(- 9)\\&= 4(- 8) \times x^{1 + 1} - 9 \times 4 \times x - 9(- 8) \times x + 81\\&= - 36x + 72x - 32x^{2} + 81\\&= (- 36 + 72) \times x - 32x^{2} + 81\\&= - 32x^{2} + 36x + 81
\end{align*}
C'est un polynôme de degré 2 avec $a = - 32$, $b = 36$ et $c = 81$.
\item
\begin{align*}
g(x) &= (5x - 4)^{2}\\&= (5x - 4)(5x - 4)\\&= 5x \times 5x + 5x(- 4) - 4 \times 5x - 4(- 4)\\&= 5 \times 5 \times x^{1 + 1} - 4 \times 5 \times x - 4 \times 5 \times x + 16\\&= - 20x - 20x + 25x^{2} + 16\\&= (- 20 - 20) \times x + 25x^{2} + 16\\&= 25x^{2} - 40x + 16
\end{align*}
C'est un polynôme de degré 2 avec $a = 25$, $b = - 40$ et $c = 16$.
\item
\begin{align*}
h(x) &= - 1 + x(- 9x + 8)\\&= - 1 + x \times - 9x + x \times 8\\&= - 9x^{2} + 8x - 1
\end{align*}
C'est un polynôme de degré 2 avec $a = - 9$, $b = 8$ et $c = - 1$.
\item
\begin{align*}
i(x) &= 9x^{2} + x(2x + 10)\\&= 9x^{2} + x \times 2x + x \times 10\\&= 9x^{2} + 2x^{2} + 10x\\&= 9x^{2} + 2x^{2} + 10x\\&= (9 + 2) \times x^{2} + 10x\\&= 11x^{2} + 10x
\end{align*}
C'est un polynôme de degré 2 avec $a = 11$, $b = 10$ et $c = 0$.
\item
\begin{align*}
j(x) &= 9(x - 8)(x + 5)\\&= (9x + 9(- 8))(x + 5)\\&= (9x - 72)(x + 5)\\&= 9x \times x + 9x \times 5 - 72x - 72 \times 5\\&= 5 \times 9 \times x - 360 + 9x^{2} - 72x\\&= 45x - 360 + 9x^{2} - 72x\\&= 9x^{2} + 45x - 72x - 360\\&= 9x^{2} + (45 - 72) \times x - 360\\&= 9x^{2} - 27x - 360
\end{align*}
C'est un polynôme de degré 2 avec $a = 9$, $b = - 27$ et $c = - 360$.
\item
\begin{align*}
k(x) &= - 9(x - 6)(x + 7)\\&= (- 9x - 9(- 6))(x + 7)\\&= (- 9x + 54)(x + 7)\\&= - 9x \times x - 9x \times 7 + 54x + 54 \times 7\\&= 7(- 9) \times x + 378 - 9x^{2} + 54x\\&= - 63x + 378 - 9x^{2} + 54x\\&= - 9x^{2} - 63x + 54x + 378\\&= - 9x^{2} + (- 63 + 54) \times x + 378\\&= - 9x^{2} - 9x + 378
\end{align*}
C'est un polynôme de degré 2 avec $a = - 9$, $b = - 9$ et $c = 378$.
\end{enumerate}
\end{solution}
\begin{exercise}[subtitle={Étude de fonctions}]
Soit $f(x) = 5x^{2} - 45x + 90$ une fonction définie sur $\R$.
\begin{enumerate}
\item Calculer les valeurs suivantes
\[
f(1) \qquad f(-2)
\]
\item Dériver la fonction $f$
\item Étudier le signe de $f'$ puis en déduire les variations de $f$.
\item Est-ce que $f$ admet un maximum? un minimum? Calculer sa valeur.
\end{enumerate}
\end{exercise}
\begin{solution}
\begin{enumerate}
\item On remplace $x$ par les valeurs demandées
\[
f(1) = 5 \times 1^{2} - 45 \times 1 + 90=5 \times 1 - 45 + 90=5 + 45=50
\]
\[
f(-1) = 5 \times - 1^{2} - 45(- 1) + 90=5 \times 1 + 45 + 90=5 + 135=140
\]
\item Dérivation
\[
f'(x) = 10x - 45
\]
\item Pas de solutions automatiques.
\item Pas de solutions automatiques.
\end{enumerate}
\end{solution}
\begin{exercise}[subtitle={Enclos}]
Dans son garage, Jean a trouvé 37m de grillage. \\
Il décide de l'utiliser pour faire un enclos rectangulaire. Afin d'obtenir un enclos encore plus grand, il veut utiliser le mur du jardin qui formera un côté. Le grillage formera les 3 autres.
\begin{center}
\includegraphics[scale=0.8]{./fig/enclos}
\end{center}
Comment placer les poteaux pour avoir un enclos le plus grand possible ?
\textit{Cet exercice est un exercice de recherche. Vous êtes invité à utiliser les outils que vous voulez. Une part importante de la note sera dédiée à la rédaction et aux explications de ce que vous faites. Vous êtes encouragé à faire des schémas.}
\textit{Si vous utilisez le tableur, vous devrez le mentionner et m'envoyer le tableur à l'adresse \url{benjamin.bertrand@ac-lyon.fr}}
\end{exercise}
\begin{solution}
Formule de l'aire (en notant $x$ la longueur du côté de l'enclos à côté du mur)
\[
A(x) = x(37 - 2x) = - 2x^{2} + 37x
\]
On va donc étudier les variations de la fonction $A(x) = - 2x^{2} + 37x$
\begin{itemize}
\item Fonction dérivée : $A'(x) = - 4x + 37$
\item On résout l'inéquation $A'(x) \geq 0$ pour déterminer quand la fonction $A'$ est positive.
\begin{align*}
A(x) & \geq 0 \\
- 4x + 37 & \geq 0 \\
- 4x + 37 + - 37 &\geq 0 + - 37 \\
- 4x &\geq - 37 \\
\frac{- 4x}{- 4} &\leq \frac{- 37}{- 4} \\
x &\leq \dfrac{37}{4} \\
\end{align*}
Donc $A(x)$ est positif quand $x$ est plus \textbf{petit} que $\dfrac{37}{4}$
\item
\begin{center}
\begin{tikzpicture}
\tkzTabInit[lgt=3,espcl=4]{$x$/1,Signe de $f'(x)$/2, Variations de $f(x)$/2}{, $\dfrac{37}{4}$ ,}%
\tkzTabLine{, +, z, -, }
\tkzTabVar{-/ ,+/$f(\dfrac{37}{4}) = \dfrac{2738}{16}$ , -/}%
\end{tikzpicture}
\end{center}
\end{itemize}
Pour avoir le plus grand enclos, il faut placer les poteaux à 6m du mur et on aura alors un enclos de $72m^2$.
\end{solution}
\end{document}
%%% Local Variables:
%%% mode: latex
%%% TeX-master: "master"
%%% End:

Binary file not shown.

View File

@ -0,0 +1,158 @@
\documentclass[a4paper,12pt]{article}
\usepackage{myXsim}
\usepackage{pgfplots}
\usetikzlibrary{decorations.markings}
\pgfplotsset{compat=1.18}
\title{ DM1 \hfill BOUQUARD James}
\tribe{1ST}
\date{A rendre pour le lundi 27 mars 2023}
\duree{}
\xsimsetup{
solution/print = false
}
\pagestyle{empty}
\begin{document}
\maketitle
Le barème est donné à titre indicatif, il pourra être modifié.
\begin{exercise}[subtitle={Polynôme de degré 2}]
Développer les expressions suivantes pour vérifier que ce sont des polynômes de degré 2. Vous préciserez les valeurs de $a$, $b$ et $c$.
\begin{multicols}{2}
\begin{enumerate}
\item $f(x) = (3x - 1)(9x - 1)$
\item $g(x) = (- 2x + 4)^{2}$
\item $h(x) = 2 + x(5x + 8)$
\item $i(x) = - 1x^{2} + x(5x - 7)$
\item $j(x) = - 8(x + 5)(x + 4)$
\item $k(x) = 2(x + 8)(x + 3)$
\end{enumerate}
\end{multicols}
\end{exercise}
\begin{solution}
\begin{enumerate}
\item
\begin{align*}
f(x) &= (3x - 1)(9x - 1)\\&= 3x \times 9x + 3x(- 1) - 1 \times 9x - 1(- 1)\\&= 3 \times 9 \times x^{1 + 1} - 1 \times 3 \times x - 1 \times 9 \times x + 1\\&= - 3x - 9x + 27x^{2} + 1\\&= (- 3 - 9) \times x + 27x^{2} + 1\\&= 27x^{2} - 12x + 1
\end{align*}
C'est un polynôme de degré 2 avec $a = 27$, $b = - 12$ et $c = 1$.
\item
\begin{align*}
g(x) &= (- 2x + 4)^{2}\\&= (- 2x + 4)(- 2x + 4)\\&= - 2x \times - 2x - 2x \times 4 + 4 \times - 2x + 4 \times 4\\&= - 2(- 2) \times x^{1 + 1} + 4(- 2) \times x + 4(- 2) \times x + 16\\&= - 8x - 8x + 4x^{2} + 16\\&= (- 8 - 8) \times x + 4x^{2} + 16\\&= 4x^{2} - 16x + 16
\end{align*}
C'est un polynôme de degré 2 avec $a = 4$, $b = - 16$ et $c = 16$.
\item
\begin{align*}
h(x) &= 2 + x(5x + 8)\\&= 2 + x \times 5x + x \times 8\\&= 5x^{2} + 8x + 2
\end{align*}
C'est un polynôme de degré 2 avec $a = 5$, $b = 8$ et $c = 2$.
\item
\begin{align*}
i(x) &= - 1x^{2} + x(5x - 7)\\&= - x^{2} + x \times 5x + x(- 7)\\&= - x^{2} + 5x^{2} - 7x\\&= - x^{2} + 5x^{2} - 7x\\&= (- 1 + 5) \times x^{2} - 7x\\&= 4x^{2} - 7x
\end{align*}
C'est un polynôme de degré 2 avec $a = 4$, $b = - 7$ et $c = 0$.
\item
\begin{align*}
j(x) &= - 8(x + 5)(x + 4)\\&= (- 8x - 8 \times 5)(x + 4)\\&= (- 8x - 40)(x + 4)\\&= - 8x \times x - 8x \times 4 - 40x - 40 \times 4\\&= 4(- 8) \times x - 160 - 8x^{2} - 40x\\&= - 32x - 160 - 8x^{2} - 40x\\&= - 8x^{2} - 32x - 40x - 160\\&= - 8x^{2} + (- 32 - 40) \times x - 160\\&= - 8x^{2} - 72x - 160
\end{align*}
C'est un polynôme de degré 2 avec $a = - 8$, $b = - 72$ et $c = - 160$.
\item
\begin{align*}
k(x) &= 2(x + 8)(x + 3)\\&= (2x + 2 \times 8)(x + 3)\\&= (2x + 16)(x + 3)\\&= 2x \times x + 2x \times 3 + 16x + 16 \times 3\\&= 3 \times 2 \times x + 48 + 2x^{2} + 16x\\&= 6x + 48 + 2x^{2} + 16x\\&= 2x^{2} + 6x + 16x + 48\\&= 2x^{2} + (6 + 16) \times x + 48\\&= 2x^{2} + 22x + 48
\end{align*}
C'est un polynôme de degré 2 avec $a = 2$, $b = 22$ et $c = 48$.
\end{enumerate}
\end{solution}
\begin{exercise}[subtitle={Étude de fonctions}]
Soit $f(x) = 5x^{2} - 15x - 140$ une fonction définie sur $\R$.
\begin{enumerate}
\item Calculer les valeurs suivantes
\[
f(1) \qquad f(-2)
\]
\item Dériver la fonction $f$
\item Étudier le signe de $f'$ puis en déduire les variations de $f$.
\item Est-ce que $f$ admet un maximum? un minimum? Calculer sa valeur.
\end{enumerate}
\end{exercise}
\begin{solution}
\begin{enumerate}
\item On remplace $x$ par les valeurs demandées
\[
f(1) = 5 \times 1^{2} - 15 \times 1 - 140=5 \times 1 - 15 - 140=5 - 155=- 150
\]
\[
f(-1) = 5 \times - 1^{2} - 15(- 1) - 140=5 \times 1 + 15 - 140=5 - 125=- 120
\]
\item Dérivation
\[
f'(x) = 10x - 15
\]
\item Pas de solutions automatiques.
\item Pas de solutions automatiques.
\end{enumerate}
\end{solution}
\begin{exercise}[subtitle={Enclos}]
Dans son garage, Jean a trouvé 21m de grillage. \\
Il décide de l'utiliser pour faire un enclos rectangulaire. Afin d'obtenir un enclos encore plus grand, il veut utiliser le mur du jardin qui formera un côté. Le grillage formera les 3 autres.
\begin{center}
\includegraphics[scale=0.8]{./fig/enclos}
\end{center}
Comment placer les poteaux pour avoir un enclos le plus grand possible ?
\textit{Cet exercice est un exercice de recherche. Vous êtes invité à utiliser les outils que vous voulez. Une part importante de la note sera dédiée à la rédaction et aux explications de ce que vous faites. Vous êtes encouragé à faire des schémas.}
\textit{Si vous utilisez le tableur, vous devrez le mentionner et m'envoyer le tableur à l'adresse \url{benjamin.bertrand@ac-lyon.fr}}
\end{exercise}
\begin{solution}
Formule de l'aire (en notant $x$ la longueur du côté de l'enclos à côté du mur)
\[
A(x) = x(21 - 2x) = - 2x^{2} + 21x
\]
On va donc étudier les variations de la fonction $A(x) = - 2x^{2} + 21x$
\begin{itemize}
\item Fonction dérivée : $A'(x) = - 4x + 21$
\item On résout l'inéquation $A'(x) \geq 0$ pour déterminer quand la fonction $A'$ est positive.
\begin{align*}
A(x) & \geq 0 \\
- 4x + 21 & \geq 0 \\
- 4x + 21 + - 21 &\geq 0 + - 21 \\
- 4x &\geq - 21 \\
\frac{- 4x}{- 4} &\leq \frac{- 21}{- 4} \\
x &\leq \dfrac{21}{4} \\
\end{align*}
Donc $A(x)$ est positif quand $x$ est plus \textbf{petit} que $\dfrac{21}{4}$
\item
\begin{center}
\begin{tikzpicture}
\tkzTabInit[lgt=3,espcl=4]{$x$/1,Signe de $f'(x)$/2, Variations de $f(x)$/2}{, $\dfrac{21}{4}$ ,}%
\tkzTabLine{, +, z, -, }
\tkzTabVar{-/ ,+/$f(\dfrac{21}{4}) = \dfrac{882}{16}$ , -/}%
\end{tikzpicture}
\end{center}
\end{itemize}
Pour avoir le plus grand enclos, il faut placer les poteaux à 6m du mur et on aura alors un enclos de $72m^2$.
\end{solution}
\end{document}
%%% Local Variables:
%%% mode: latex
%%% TeX-master: "master"
%%% End:

Binary file not shown.

View File

@ -0,0 +1,158 @@
\documentclass[a4paper,12pt]{article}
\usepackage{myXsim}
\usepackage{pgfplots}
\usetikzlibrary{decorations.markings}
\pgfplotsset{compat=1.18}
\title{ DM1 \hfill DA COSTA QUEIROGA Délinda}
\tribe{1ST}
\date{A rendre pour le lundi 27 mars 2023}
\duree{}
\xsimsetup{
solution/print = false
}
\pagestyle{empty}
\begin{document}
\maketitle
Le barème est donné à titre indicatif, il pourra être modifié.
\begin{exercise}[subtitle={Polynôme de degré 2}]
Développer les expressions suivantes pour vérifier que ce sont des polynômes de degré 2. Vous préciserez les valeurs de $a$, $b$ et $c$.
\begin{multicols}{2}
\begin{enumerate}
\item $f(x) = (2x - 2)(3x - 2)$
\item $g(x) = (- 6x + 5)^{2}$
\item $h(x) = 10 + x(4x + 9)$
\item $i(x) = 4x^{2} + x(- 10x - 8)$
\item $j(x) = 7(x + 3)(x - 6)$
\item $k(x) = - 5(x + 3)(x - 3)$
\end{enumerate}
\end{multicols}
\end{exercise}
\begin{solution}
\begin{enumerate}
\item
\begin{align*}
f(x) &= (2x - 2)(3x - 2)\\&= 2x \times 3x + 2x(- 2) - 2 \times 3x - 2(- 2)\\&= 2 \times 3 \times x^{1 + 1} - 2 \times 2 \times x - 2 \times 3 \times x + 4\\&= - 4x - 6x + 6x^{2} + 4\\&= (- 4 - 6) \times x + 6x^{2} + 4\\&= 6x^{2} - 10x + 4
\end{align*}
C'est un polynôme de degré 2 avec $a = 6$, $b = - 10$ et $c = 4$.
\item
\begin{align*}
g(x) &= (- 6x + 5)^{2}\\&= (- 6x + 5)(- 6x + 5)\\&= - 6x \times - 6x - 6x \times 5 + 5 \times - 6x + 5 \times 5\\&= - 6(- 6) \times x^{1 + 1} + 5(- 6) \times x + 5(- 6) \times x + 25\\&= - 30x - 30x + 36x^{2} + 25\\&= (- 30 - 30) \times x + 36x^{2} + 25\\&= 36x^{2} - 60x + 25
\end{align*}
C'est un polynôme de degré 2 avec $a = 36$, $b = - 60$ et $c = 25$.
\item
\begin{align*}
h(x) &= 10 + x(4x + 9)\\&= 10 + x \times 4x + x \times 9\\&= 4x^{2} + 9x + 10
\end{align*}
C'est un polynôme de degré 2 avec $a = 4$, $b = 9$ et $c = 10$.
\item
\begin{align*}
i(x) &= 4x^{2} + x(- 10x - 8)\\&= 4x^{2} + x \times - 10x + x(- 8)\\&= 4x^{2} - 10x^{2} - 8x\\&= 4x^{2} - 10x^{2} - 8x\\&= (4 - 10) \times x^{2} - 8x\\&= - 6x^{2} - 8x
\end{align*}
C'est un polynôme de degré 2 avec $a = - 6$, $b = - 8$ et $c = 0$.
\item
\begin{align*}
j(x) &= 7(x + 3)(x - 6)\\&= (7x + 7 \times 3)(x - 6)\\&= (7x + 21)(x - 6)\\&= 7x \times x + 7x(- 6) + 21x + 21(- 6)\\&= - 6 \times 7 \times x - 126 + 7x^{2} + 21x\\&= - 42x - 126 + 7x^{2} + 21x\\&= 7x^{2} - 42x + 21x - 126\\&= 7x^{2} + (- 42 + 21) \times x - 126\\&= 7x^{2} - 21x - 126
\end{align*}
C'est un polynôme de degré 2 avec $a = 7$, $b = - 21$ et $c = - 126$.
\item
\begin{align*}
k(x) &= - 5(x + 3)(x - 3)\\&= (- 5x - 5 \times 3)(x - 3)\\&= (- 5x - 15)(x - 3)\\&= - 5x \times x - 5x(- 3) - 15x - 15(- 3)\\&= - 3(- 5) \times x + 45 - 5x^{2} - 15x\\&= 15x + 45 - 5x^{2} - 15x\\&= - 5x^{2} + 15x - 15x + 45\\&= - 5x^{2} + (15 - 15) \times x + 45\\&= - 5x^{2} + 45
\end{align*}
C'est un polynôme de degré 2 avec $a = - 5$, $b = 0$ et $c = 45$.
\end{enumerate}
\end{solution}
\begin{exercise}[subtitle={Étude de fonctions}]
Soit $f(x) = - 7x^{2} + 91x - 280$ une fonction définie sur $\R$.
\begin{enumerate}
\item Calculer les valeurs suivantes
\[
f(1) \qquad f(-2)
\]
\item Dériver la fonction $f$
\item Étudier le signe de $f'$ puis en déduire les variations de $f$.
\item Est-ce que $f$ admet un maximum? un minimum? Calculer sa valeur.
\end{enumerate}
\end{exercise}
\begin{solution}
\begin{enumerate}
\item On remplace $x$ par les valeurs demandées
\[
f(1) = - 7 \times 1^{2} + 91 \times 1 - 280=- 7 \times 1 + 91 - 280=- 7 - 189=- 196
\]
\[
f(-1) = - 7 \times - 1^{2} + 91(- 1) - 280=- 7 \times 1 - 91 - 280=- 7 - 371=- 378
\]
\item Dérivation
\[
f'(x) = - 14x + 91
\]
\item Pas de solutions automatiques.
\item Pas de solutions automatiques.
\end{enumerate}
\end{solution}
\begin{exercise}[subtitle={Enclos}]
Dans son garage, Jean a trouvé 23m de grillage. \\
Il décide de l'utiliser pour faire un enclos rectangulaire. Afin d'obtenir un enclos encore plus grand, il veut utiliser le mur du jardin qui formera un côté. Le grillage formera les 3 autres.
\begin{center}
\includegraphics[scale=0.8]{./fig/enclos}
\end{center}
Comment placer les poteaux pour avoir un enclos le plus grand possible ?
\textit{Cet exercice est un exercice de recherche. Vous êtes invité à utiliser les outils que vous voulez. Une part importante de la note sera dédiée à la rédaction et aux explications de ce que vous faites. Vous êtes encouragé à faire des schémas.}
\textit{Si vous utilisez le tableur, vous devrez le mentionner et m'envoyer le tableur à l'adresse \url{benjamin.bertrand@ac-lyon.fr}}
\end{exercise}
\begin{solution}
Formule de l'aire (en notant $x$ la longueur du côté de l'enclos à côté du mur)
\[
A(x) = x(23 - 2x) = - 2x^{2} + 23x
\]
On va donc étudier les variations de la fonction $A(x) = - 2x^{2} + 23x$
\begin{itemize}
\item Fonction dérivée : $A'(x) = - 4x + 23$
\item On résout l'inéquation $A'(x) \geq 0$ pour déterminer quand la fonction $A'$ est positive.
\begin{align*}
A(x) & \geq 0 \\
- 4x + 23 & \geq 0 \\
- 4x + 23 + - 23 &\geq 0 + - 23 \\
- 4x &\geq - 23 \\
\frac{- 4x}{- 4} &\leq \frac{- 23}{- 4} \\
x &\leq \dfrac{23}{4} \\
\end{align*}
Donc $A(x)$ est positif quand $x$ est plus \textbf{petit} que $\dfrac{23}{4}$
\item
\begin{center}
\begin{tikzpicture}
\tkzTabInit[lgt=3,espcl=4]{$x$/1,Signe de $f'(x)$/2, Variations de $f(x)$/2}{, $\dfrac{23}{4}$ ,}%
\tkzTabLine{, +, z, -, }
\tkzTabVar{-/ ,+/$f(\dfrac{23}{4}) = \dfrac{1058}{16}$ , -/}%
\end{tikzpicture}
\end{center}
\end{itemize}
Pour avoir le plus grand enclos, il faut placer les poteaux à 6m du mur et on aura alors un enclos de $72m^2$.
\end{solution}
\end{document}
%%% Local Variables:
%%% mode: latex
%%% TeX-master: "master"
%%% End:

Binary file not shown.

View File

@ -0,0 +1,158 @@
\documentclass[a4paper,12pt]{article}
\usepackage{myXsim}
\usepackage{pgfplots}
\usetikzlibrary{decorations.markings}
\pgfplotsset{compat=1.18}
\title{ DM1 \hfill GNUI Kadia}
\tribe{1ST}
\date{A rendre pour le lundi 27 mars 2023}
\duree{}
\xsimsetup{
solution/print = false
}
\pagestyle{empty}
\begin{document}
\maketitle
Le barème est donné à titre indicatif, il pourra être modifié.
\begin{exercise}[subtitle={Polynôme de degré 2}]
Développer les expressions suivantes pour vérifier que ce sont des polynômes de degré 2. Vous préciserez les valeurs de $a$, $b$ et $c$.
\begin{multicols}{2}
\begin{enumerate}
\item $f(x) = (- 4x - 1)(- 8x - 1)$
\item $g(x) = (7x + 8)^{2}$
\item $h(x) = 3 + x(7x + 7)$
\item $i(x) = - 9x^{2} + x(- 3x + 10)$
\item $j(x) = - 5(x + 7)(x + 5)$
\item $k(x) = - 4(x - 6)(x + 2)$
\end{enumerate}
\end{multicols}
\end{exercise}
\begin{solution}
\begin{enumerate}
\item
\begin{align*}
f(x) &= (- 4x - 1)(- 8x - 1)\\&= - 4x \times - 8x - 4x(- 1) - 1 \times - 8x - 1(- 1)\\&= - 4(- 8) \times x^{1 + 1} - 1(- 4) \times x - 1(- 8) \times x + 1\\&= 4x + 8x + 32x^{2} + 1\\&= (4 + 8) \times x + 32x^{2} + 1\\&= 32x^{2} + 12x + 1
\end{align*}
C'est un polynôme de degré 2 avec $a = 32$, $b = 12$ et $c = 1$.
\item
\begin{align*}
g(x) &= (7x + 8)^{2}\\&= (7x + 8)(7x + 8)\\&= 7x \times 7x + 7x \times 8 + 8 \times 7x + 8 \times 8\\&= 7 \times 7 \times x^{1 + 1} + 8 \times 7 \times x + 8 \times 7 \times x + 64\\&= 56x + 56x + 49x^{2} + 64\\&= (56 + 56) \times x + 49x^{2} + 64\\&= 49x^{2} + 112x + 64
\end{align*}
C'est un polynôme de degré 2 avec $a = 49$, $b = 112$ et $c = 64$.
\item
\begin{align*}
h(x) &= 3 + x(7x + 7)\\&= 3 + x \times 7x + x \times 7\\&= 7x^{2} + 7x + 3
\end{align*}
C'est un polynôme de degré 2 avec $a = 7$, $b = 7$ et $c = 3$.
\item
\begin{align*}
i(x) &= - 9x^{2} + x(- 3x + 10)\\&= - 9x^{2} + x \times - 3x + x \times 10\\&= - 9x^{2} - 3x^{2} + 10x\\&= - 9x^{2} - 3x^{2} + 10x\\&= (- 9 - 3) \times x^{2} + 10x\\&= - 12x^{2} + 10x
\end{align*}
C'est un polynôme de degré 2 avec $a = - 12$, $b = 10$ et $c = 0$.
\item
\begin{align*}
j(x) &= - 5(x + 7)(x + 5)\\&= (- 5x - 5 \times 7)(x + 5)\\&= (- 5x - 35)(x + 5)\\&= - 5x \times x - 5x \times 5 - 35x - 35 \times 5\\&= 5(- 5) \times x - 175 - 5x^{2} - 35x\\&= - 25x - 175 - 5x^{2} - 35x\\&= - 5x^{2} - 25x - 35x - 175\\&= - 5x^{2} + (- 25 - 35) \times x - 175\\&= - 5x^{2} - 60x - 175
\end{align*}
C'est un polynôme de degré 2 avec $a = - 5$, $b = - 60$ et $c = - 175$.
\item
\begin{align*}
k(x) &= - 4(x - 6)(x + 2)\\&= (- 4x - 4(- 6))(x + 2)\\&= (- 4x + 24)(x + 2)\\&= - 4x \times x - 4x \times 2 + 24x + 24 \times 2\\&= 2(- 4) \times x + 48 - 4x^{2} + 24x\\&= - 8x + 48 - 4x^{2} + 24x\\&= - 4x^{2} - 8x + 24x + 48\\&= - 4x^{2} + (- 8 + 24) \times x + 48\\&= - 4x^{2} + 16x + 48
\end{align*}
C'est un polynôme de degré 2 avec $a = - 4$, $b = 16$ et $c = 48$.
\end{enumerate}
\end{solution}
\begin{exercise}[subtitle={Étude de fonctions}]
Soit $f(x) = - 3x^{2} - 33x - 30$ une fonction définie sur $\R$.
\begin{enumerate}
\item Calculer les valeurs suivantes
\[
f(1) \qquad f(-2)
\]
\item Dériver la fonction $f$
\item Étudier le signe de $f'$ puis en déduire les variations de $f$.
\item Est-ce que $f$ admet un maximum? un minimum? Calculer sa valeur.
\end{enumerate}
\end{exercise}
\begin{solution}
\begin{enumerate}
\item On remplace $x$ par les valeurs demandées
\[
f(1) = - 3 \times 1^{2} - 33 \times 1 - 30=- 3 \times 1 - 33 - 30=- 3 - 63=- 66
\]
\[
f(-1) = - 3 \times - 1^{2} - 33(- 1) - 30=- 3 \times 1 + 33 - 30=- 3 + 3=0
\]
\item Dérivation
\[
f'(x) = - 6x - 33
\]
\item Pas de solutions automatiques.
\item Pas de solutions automatiques.
\end{enumerate}
\end{solution}
\begin{exercise}[subtitle={Enclos}]
Dans son garage, Jean a trouvé 35m de grillage. \\
Il décide de l'utiliser pour faire un enclos rectangulaire. Afin d'obtenir un enclos encore plus grand, il veut utiliser le mur du jardin qui formera un côté. Le grillage formera les 3 autres.
\begin{center}
\includegraphics[scale=0.8]{./fig/enclos}
\end{center}
Comment placer les poteaux pour avoir un enclos le plus grand possible ?
\textit{Cet exercice est un exercice de recherche. Vous êtes invité à utiliser les outils que vous voulez. Une part importante de la note sera dédiée à la rédaction et aux explications de ce que vous faites. Vous êtes encouragé à faire des schémas.}
\textit{Si vous utilisez le tableur, vous devrez le mentionner et m'envoyer le tableur à l'adresse \url{benjamin.bertrand@ac-lyon.fr}}
\end{exercise}
\begin{solution}
Formule de l'aire (en notant $x$ la longueur du côté de l'enclos à côté du mur)
\[
A(x) = x(35 - 2x) = - 2x^{2} + 35x
\]
On va donc étudier les variations de la fonction $A(x) = - 2x^{2} + 35x$
\begin{itemize}
\item Fonction dérivée : $A'(x) = - 4x + 35$
\item On résout l'inéquation $A'(x) \geq 0$ pour déterminer quand la fonction $A'$ est positive.
\begin{align*}
A(x) & \geq 0 \\
- 4x + 35 & \geq 0 \\
- 4x + 35 + - 35 &\geq 0 + - 35 \\
- 4x &\geq - 35 \\
\frac{- 4x}{- 4} &\leq \frac{- 35}{- 4} \\
x &\leq \dfrac{35}{4} \\
\end{align*}
Donc $A(x)$ est positif quand $x$ est plus \textbf{petit} que $\dfrac{35}{4}$
\item
\begin{center}
\begin{tikzpicture}
\tkzTabInit[lgt=3,espcl=4]{$x$/1,Signe de $f'(x)$/2, Variations de $f(x)$/2}{, $\dfrac{35}{4}$ ,}%
\tkzTabLine{, +, z, -, }
\tkzTabVar{-/ ,+/$f(\dfrac{35}{4}) = \dfrac{2450}{16}$ , -/}%
\end{tikzpicture}
\end{center}
\end{itemize}
Pour avoir le plus grand enclos, il faut placer les poteaux à 6m du mur et on aura alors un enclos de $72m^2$.
\end{solution}
\end{document}
%%% Local Variables:
%%% mode: latex
%%% TeX-master: "master"
%%% End:

Binary file not shown.

View File

@ -0,0 +1,158 @@
\documentclass[a4paper,12pt]{article}
\usepackage{myXsim}
\usepackage{pgfplots}
\usetikzlibrary{decorations.markings}
\pgfplotsset{compat=1.18}
\title{ DM1 \hfill HAMMOUDI Lyna}
\tribe{1ST}
\date{A rendre pour le lundi 27 mars 2023}
\duree{}
\xsimsetup{
solution/print = false
}
\pagestyle{empty}
\begin{document}
\maketitle
Le barème est donné à titre indicatif, il pourra être modifié.
\begin{exercise}[subtitle={Polynôme de degré 2}]
Développer les expressions suivantes pour vérifier que ce sont des polynômes de degré 2. Vous préciserez les valeurs de $a$, $b$ et $c$.
\begin{multicols}{2}
\begin{enumerate}
\item $f(x) = (- 7x + 10)(- 8x + 10)$
\item $g(x) = (6x - 2)^{2}$
\item $h(x) = 2 + x(- 5x + 4)$
\item $i(x) = - 9x^{2} + x(3x - 2)$
\item $j(x) = 6(x + 9)(x + 10)$
\item $k(x) = - 8(x - 10)(x - 4)$
\end{enumerate}
\end{multicols}
\end{exercise}
\begin{solution}
\begin{enumerate}
\item
\begin{align*}
f(x) &= (- 7x + 10)(- 8x + 10)\\&= - 7x \times - 8x - 7x \times 10 + 10 \times - 8x + 10 \times 10\\&= - 7(- 8) \times x^{1 + 1} + 10(- 7) \times x + 10(- 8) \times x + 100\\&= - 70x - 80x + 56x^{2} + 100\\&= (- 70 - 80) \times x + 56x^{2} + 100\\&= 56x^{2} - 150x + 100
\end{align*}
C'est un polynôme de degré 2 avec $a = 56$, $b = - 150$ et $c = 100$.
\item
\begin{align*}
g(x) &= (6x - 2)^{2}\\&= (6x - 2)(6x - 2)\\&= 6x \times 6x + 6x(- 2) - 2 \times 6x - 2(- 2)\\&= 6 \times 6 \times x^{1 + 1} - 2 \times 6 \times x - 2 \times 6 \times x + 4\\&= - 12x - 12x + 36x^{2} + 4\\&= (- 12 - 12) \times x + 36x^{2} + 4\\&= 36x^{2} - 24x + 4
\end{align*}
C'est un polynôme de degré 2 avec $a = 36$, $b = - 24$ et $c = 4$.
\item
\begin{align*}
h(x) &= 2 + x(- 5x + 4)\\&= 2 + x \times - 5x + x \times 4\\&= - 5x^{2} + 4x + 2
\end{align*}
C'est un polynôme de degré 2 avec $a = - 5$, $b = 4$ et $c = 2$.
\item
\begin{align*}
i(x) &= - 9x^{2} + x(3x - 2)\\&= - 9x^{2} + x \times 3x + x(- 2)\\&= - 9x^{2} + 3x^{2} - 2x\\&= - 9x^{2} + 3x^{2} - 2x\\&= (- 9 + 3) \times x^{2} - 2x\\&= - 6x^{2} - 2x
\end{align*}
C'est un polynôme de degré 2 avec $a = - 6$, $b = - 2$ et $c = 0$.
\item
\begin{align*}
j(x) &= 6(x + 9)(x + 10)\\&= (6x + 6 \times 9)(x + 10)\\&= (6x + 54)(x + 10)\\&= 6x \times x + 6x \times 10 + 54x + 54 \times 10\\&= 10 \times 6 \times x + 540 + 6x^{2} + 54x\\&= 60x + 540 + 6x^{2} + 54x\\&= 6x^{2} + 60x + 54x + 540\\&= 6x^{2} + (60 + 54) \times x + 540\\&= 6x^{2} + 114x + 540
\end{align*}
C'est un polynôme de degré 2 avec $a = 6$, $b = 114$ et $c = 540$.
\item
\begin{align*}
k(x) &= - 8(x - 10)(x - 4)\\&= (- 8x - 8(- 10))(x - 4)\\&= (- 8x + 80)(x - 4)\\&= - 8x \times x - 8x(- 4) + 80x + 80(- 4)\\&= - 4(- 8) \times x - 320 - 8x^{2} + 80x\\&= 32x - 320 - 8x^{2} + 80x\\&= - 8x^{2} + 32x + 80x - 320\\&= - 8x^{2} + (32 + 80) \times x - 320\\&= - 8x^{2} + 112x - 320
\end{align*}
C'est un polynôme de degré 2 avec $a = - 8$, $b = 112$ et $c = - 320$.
\end{enumerate}
\end{solution}
\begin{exercise}[subtitle={Étude de fonctions}]
Soit $f(x) = - x^{2} - 2x + 48$ une fonction définie sur $\R$.
\begin{enumerate}
\item Calculer les valeurs suivantes
\[
f(1) \qquad f(-2)
\]
\item Dériver la fonction $f$
\item Étudier le signe de $f'$ puis en déduire les variations de $f$.
\item Est-ce que $f$ admet un maximum? un minimum? Calculer sa valeur.
\end{enumerate}
\end{exercise}
\begin{solution}
\begin{enumerate}
\item On remplace $x$ par les valeurs demandées
\[
f(1) = - 1^{2} - 2 \times 1 + 48=- 1 - 2 + 48=- 1 + 46=45
\]
\[
f(-1) = - - 1^{2} - 2(- 1) + 48=- 1 + 2 + 48=- 1 + 50=49
\]
\item Dérivation
\[
f'(x) = - 2x - 2
\]
\item Pas de solutions automatiques.
\item Pas de solutions automatiques.
\end{enumerate}
\end{solution}
\begin{exercise}[subtitle={Enclos}]
Dans son garage, Jean a trouvé 37m de grillage. \\
Il décide de l'utiliser pour faire un enclos rectangulaire. Afin d'obtenir un enclos encore plus grand, il veut utiliser le mur du jardin qui formera un côté. Le grillage formera les 3 autres.
\begin{center}
\includegraphics[scale=0.8]{./fig/enclos}
\end{center}
Comment placer les poteaux pour avoir un enclos le plus grand possible ?
\textit{Cet exercice est un exercice de recherche. Vous êtes invité à utiliser les outils que vous voulez. Une part importante de la note sera dédiée à la rédaction et aux explications de ce que vous faites. Vous êtes encouragé à faire des schémas.}
\textit{Si vous utilisez le tableur, vous devrez le mentionner et m'envoyer le tableur à l'adresse \url{benjamin.bertrand@ac-lyon.fr}}
\end{exercise}
\begin{solution}
Formule de l'aire (en notant $x$ la longueur du côté de l'enclos à côté du mur)
\[
A(x) = x(37 - 2x) = - 2x^{2} + 37x
\]
On va donc étudier les variations de la fonction $A(x) = - 2x^{2} + 37x$
\begin{itemize}
\item Fonction dérivée : $A'(x) = - 4x + 37$
\item On résout l'inéquation $A'(x) \geq 0$ pour déterminer quand la fonction $A'$ est positive.
\begin{align*}
A(x) & \geq 0 \\
- 4x + 37 & \geq 0 \\
- 4x + 37 + - 37 &\geq 0 + - 37 \\
- 4x &\geq - 37 \\
\frac{- 4x}{- 4} &\leq \frac{- 37}{- 4} \\
x &\leq \dfrac{37}{4} \\
\end{align*}
Donc $A(x)$ est positif quand $x$ est plus \textbf{petit} que $\dfrac{37}{4}$
\item
\begin{center}
\begin{tikzpicture}
\tkzTabInit[lgt=3,espcl=4]{$x$/1,Signe de $f'(x)$/2, Variations de $f(x)$/2}{, $\dfrac{37}{4}$ ,}%
\tkzTabLine{, +, z, -, }
\tkzTabVar{-/ ,+/$f(\dfrac{37}{4}) = \dfrac{2738}{16}$ , -/}%
\end{tikzpicture}
\end{center}
\end{itemize}
Pour avoir le plus grand enclos, il faut placer les poteaux à 6m du mur et on aura alors un enclos de $72m^2$.
\end{solution}
\end{document}
%%% Local Variables:
%%% mode: latex
%%% TeX-master: "master"
%%% End:

Binary file not shown.

View File

@ -0,0 +1,158 @@
\documentclass[a4paper,12pt]{article}
\usepackage{myXsim}
\usepackage{pgfplots}
\usetikzlibrary{decorations.markings}
\pgfplotsset{compat=1.18}
\title{ DM1 \hfill KITOUNI Zakaria}
\tribe{1ST}
\date{A rendre pour le lundi 27 mars 2023}
\duree{}
\xsimsetup{
solution/print = false
}
\pagestyle{empty}
\begin{document}
\maketitle
Le barème est donné à titre indicatif, il pourra être modifié.
\begin{exercise}[subtitle={Polynôme de degré 2}]
Développer les expressions suivantes pour vérifier que ce sont des polynômes de degré 2. Vous préciserez les valeurs de $a$, $b$ et $c$.
\begin{multicols}{2}
\begin{enumerate}
\item $f(x) = (- 7x - 6)(- 10x - 6)$
\item $g(x) = (4x + 2)^{2}$
\item $h(x) = 8 + x(- 9x + 7)$
\item $i(x) = 9x^{2} + x(9x - 5)$
\item $j(x) = - 1(x - 6)(x - 7)$
\item $k(x) = - 7(x - 1)(x + 3)$
\end{enumerate}
\end{multicols}
\end{exercise}
\begin{solution}
\begin{enumerate}
\item
\begin{align*}
f(x) &= (- 7x - 6)(- 10x - 6)\\&= - 7x \times - 10x - 7x(- 6) - 6 \times - 10x - 6(- 6)\\&= - 7(- 10) \times x^{1 + 1} - 6(- 7) \times x - 6(- 10) \times x + 36\\&= 42x + 60x + 70x^{2} + 36\\&= (42 + 60) \times x + 70x^{2} + 36\\&= 70x^{2} + 102x + 36
\end{align*}
C'est un polynôme de degré 2 avec $a = 70$, $b = 102$ et $c = 36$.
\item
\begin{align*}
g(x) &= (4x + 2)^{2}\\&= (4x + 2)(4x + 2)\\&= 4x \times 4x + 4x \times 2 + 2 \times 4x + 2 \times 2\\&= 4 \times 4 \times x^{1 + 1} + 2 \times 4 \times x + 2 \times 4 \times x + 4\\&= 8x + 8x + 16x^{2} + 4\\&= (8 + 8) \times x + 16x^{2} + 4\\&= 16x^{2} + 16x + 4
\end{align*}
C'est un polynôme de degré 2 avec $a = 16$, $b = 16$ et $c = 4$.
\item
\begin{align*}
h(x) &= 8 + x(- 9x + 7)\\&= 8 + x \times - 9x + x \times 7\\&= - 9x^{2} + 7x + 8
\end{align*}
C'est un polynôme de degré 2 avec $a = - 9$, $b = 7$ et $c = 8$.
\item
\begin{align*}
i(x) &= 9x^{2} + x(9x - 5)\\&= 9x^{2} + x \times 9x + x(- 5)\\&= 9x^{2} + 9x^{2} - 5x\\&= 9x^{2} + 9x^{2} - 5x\\&= (9 + 9) \times x^{2} - 5x\\&= 18x^{2} - 5x
\end{align*}
C'est un polynôme de degré 2 avec $a = 18$, $b = - 5$ et $c = 0$.
\item
\begin{align*}
j(x) &= - 1(x - 6)(x - 7)\\&= (- 1x - 1(- 6))(x - 7)\\&= (- x + 6)(x - 7)\\&= (- x) \times x + (- x)(- 7) + 6x + 6(- 7)\\&= - 7(- 1) \times x - 42 - x^{2} + 6x\\&= 7x - 42 - x^{2} + 6x\\&= - x^{2} + 7x + 6x - 42\\&= - x^{2} + (7 + 6) \times x - 42\\&= - x^{2} + 13x - 42
\end{align*}
C'est un polynôme de degré 2 avec $a = - 1$, $b = 13$ et $c = - 42$.
\item
\begin{align*}
k(x) &= - 7(x - 1)(x + 3)\\&= (- 7x - 7(- 1))(x + 3)\\&= (- 7x + 7)(x + 3)\\&= - 7x \times x - 7x \times 3 + 7x + 7 \times 3\\&= 3(- 7) \times x + 21 - 7x^{2} + 7x\\&= - 21x + 21 - 7x^{2} + 7x\\&= - 7x^{2} - 21x + 7x + 21\\&= - 7x^{2} + (- 21 + 7) \times x + 21\\&= - 7x^{2} - 14x + 21
\end{align*}
C'est un polynôme de degré 2 avec $a = - 7$, $b = - 14$ et $c = 21$.
\end{enumerate}
\end{solution}
\begin{exercise}[subtitle={Étude de fonctions}]
Soit $f(x) = 3x^{2} - 3x - 126$ une fonction définie sur $\R$.
\begin{enumerate}
\item Calculer les valeurs suivantes
\[
f(1) \qquad f(-2)
\]
\item Dériver la fonction $f$
\item Étudier le signe de $f'$ puis en déduire les variations de $f$.
\item Est-ce que $f$ admet un maximum? un minimum? Calculer sa valeur.
\end{enumerate}
\end{exercise}
\begin{solution}
\begin{enumerate}
\item On remplace $x$ par les valeurs demandées
\[
f(1) = 3 \times 1^{2} - 3 \times 1 - 126=3 \times 1 - 3 - 126=3 - 129=- 126
\]
\[
f(-1) = 3 \times - 1^{2} - 3(- 1) - 126=3 \times 1 + 3 - 126=3 - 123=- 120
\]
\item Dérivation
\[
f'(x) = 6x - 3
\]
\item Pas de solutions automatiques.
\item Pas de solutions automatiques.
\end{enumerate}
\end{solution}
\begin{exercise}[subtitle={Enclos}]
Dans son garage, Jean a trouvé 29m de grillage. \\
Il décide de l'utiliser pour faire un enclos rectangulaire. Afin d'obtenir un enclos encore plus grand, il veut utiliser le mur du jardin qui formera un côté. Le grillage formera les 3 autres.
\begin{center}
\includegraphics[scale=0.8]{./fig/enclos}
\end{center}
Comment placer les poteaux pour avoir un enclos le plus grand possible ?
\textit{Cet exercice est un exercice de recherche. Vous êtes invité à utiliser les outils que vous voulez. Une part importante de la note sera dédiée à la rédaction et aux explications de ce que vous faites. Vous êtes encouragé à faire des schémas.}
\textit{Si vous utilisez le tableur, vous devrez le mentionner et m'envoyer le tableur à l'adresse \url{benjamin.bertrand@ac-lyon.fr}}
\end{exercise}
\begin{solution}
Formule de l'aire (en notant $x$ la longueur du côté de l'enclos à côté du mur)
\[
A(x) = x(29 - 2x) = - 2x^{2} + 29x
\]
On va donc étudier les variations de la fonction $A(x) = - 2x^{2} + 29x$
\begin{itemize}
\item Fonction dérivée : $A'(x) = - 4x + 29$
\item On résout l'inéquation $A'(x) \geq 0$ pour déterminer quand la fonction $A'$ est positive.
\begin{align*}
A(x) & \geq 0 \\
- 4x + 29 & \geq 0 \\
- 4x + 29 + - 29 &\geq 0 + - 29 \\
- 4x &\geq - 29 \\
\frac{- 4x}{- 4} &\leq \frac{- 29}{- 4} \\
x &\leq \dfrac{29}{4} \\
\end{align*}
Donc $A(x)$ est positif quand $x$ est plus \textbf{petit} que $\dfrac{29}{4}$
\item
\begin{center}
\begin{tikzpicture}
\tkzTabInit[lgt=3,espcl=4]{$x$/1,Signe de $f'(x)$/2, Variations de $f(x)$/2}{, $\dfrac{29}{4}$ ,}%
\tkzTabLine{, +, z, -, }
\tkzTabVar{-/ ,+/$f(\dfrac{29}{4}) = \dfrac{1682}{16}$ , -/}%
\end{tikzpicture}
\end{center}
\end{itemize}
Pour avoir le plus grand enclos, il faut placer les poteaux à 6m du mur et on aura alors un enclos de $72m^2$.
\end{solution}
\end{document}
%%% Local Variables:
%%% mode: latex
%%% TeX-master: "master"
%%% End:

Binary file not shown.

View File

@ -0,0 +1,158 @@
\documentclass[a4paper,12pt]{article}
\usepackage{myXsim}
\usepackage{pgfplots}
\usetikzlibrary{decorations.markings}
\pgfplotsset{compat=1.18}
\title{ DM1 \hfill LAFAVERGES Joana}
\tribe{1ST}
\date{A rendre pour le lundi 27 mars 2023}
\duree{}
\xsimsetup{
solution/print = false
}
\pagestyle{empty}
\begin{document}
\maketitle
Le barème est donné à titre indicatif, il pourra être modifié.
\begin{exercise}[subtitle={Polynôme de degré 2}]
Développer les expressions suivantes pour vérifier que ce sont des polynômes de degré 2. Vous préciserez les valeurs de $a$, $b$ et $c$.
\begin{multicols}{2}
\begin{enumerate}
\item $f(x) = (10x + 7)(4x + 7)$
\item $g(x) = (- 5x - 3)^{2}$
\item $h(x) = - 4 + x(- 2x + 2)$
\item $i(x) = 3x^{2} + x(- 4x - 9)$
\item $j(x) = - 5(x + 3)(x - 10)$
\item $k(x) = 8(x - 2)(x - 6)$
\end{enumerate}
\end{multicols}
\end{exercise}
\begin{solution}
\begin{enumerate}
\item
\begin{align*}
f(x) &= (10x + 7)(4x + 7)\\&= 10x \times 4x + 10x \times 7 + 7 \times 4x + 7 \times 7\\&= 10 \times 4 \times x^{1 + 1} + 7 \times 10 \times x + 7 \times 4 \times x + 49\\&= 70x + 28x + 40x^{2} + 49\\&= (70 + 28) \times x + 40x^{2} + 49\\&= 40x^{2} + 98x + 49
\end{align*}
C'est un polynôme de degré 2 avec $a = 40$, $b = 98$ et $c = 49$.
\item
\begin{align*}
g(x) &= (- 5x - 3)^{2}\\&= (- 5x - 3)(- 5x - 3)\\&= - 5x \times - 5x - 5x(- 3) - 3 \times - 5x - 3(- 3)\\&= - 5(- 5) \times x^{1 + 1} - 3(- 5) \times x - 3(- 5) \times x + 9\\&= 15x + 15x + 25x^{2} + 9\\&= (15 + 15) \times x + 25x^{2} + 9\\&= 25x^{2} + 30x + 9
\end{align*}
C'est un polynôme de degré 2 avec $a = 25$, $b = 30$ et $c = 9$.
\item
\begin{align*}
h(x) &= - 4 + x(- 2x + 2)\\&= - 4 + x \times - 2x + x \times 2\\&= - 2x^{2} + 2x - 4
\end{align*}
C'est un polynôme de degré 2 avec $a = - 2$, $b = 2$ et $c = - 4$.
\item
\begin{align*}
i(x) &= 3x^{2} + x(- 4x - 9)\\&= 3x^{2} + x \times - 4x + x(- 9)\\&= 3x^{2} - 4x^{2} - 9x\\&= 3x^{2} - 4x^{2} - 9x\\&= (3 - 4) \times x^{2} - 9x\\&= - x^{2} - 9x
\end{align*}
C'est un polynôme de degré 2 avec $a = - 1$, $b = - 9$ et $c = 0$.
\item
\begin{align*}
j(x) &= - 5(x + 3)(x - 10)\\&= (- 5x - 5 \times 3)(x - 10)\\&= (- 5x - 15)(x - 10)\\&= - 5x \times x - 5x(- 10) - 15x - 15(- 10)\\&= - 10(- 5) \times x + 150 - 5x^{2} - 15x\\&= 50x + 150 - 5x^{2} - 15x\\&= - 5x^{2} + 50x - 15x + 150\\&= - 5x^{2} + (50 - 15) \times x + 150\\&= - 5x^{2} + 35x + 150
\end{align*}
C'est un polynôme de degré 2 avec $a = - 5$, $b = 35$ et $c = 150$.
\item
\begin{align*}
k(x) &= 8(x - 2)(x - 6)\\&= (8x + 8(- 2))(x - 6)\\&= (8x - 16)(x - 6)\\&= 8x \times x + 8x(- 6) - 16x - 16(- 6)\\&= - 6 \times 8 \times x + 96 + 8x^{2} - 16x\\&= - 48x + 96 + 8x^{2} - 16x\\&= 8x^{2} - 48x - 16x + 96\\&= 8x^{2} + (- 48 - 16) \times x + 96\\&= 8x^{2} - 64x + 96
\end{align*}
C'est un polynôme de degré 2 avec $a = 8$, $b = - 64$ et $c = 96$.
\end{enumerate}
\end{solution}
\begin{exercise}[subtitle={Étude de fonctions}]
Soit $f(x) = 6x^{2} - 36x + 48$ une fonction définie sur $\R$.
\begin{enumerate}
\item Calculer les valeurs suivantes
\[
f(1) \qquad f(-2)
\]
\item Dériver la fonction $f$
\item Étudier le signe de $f'$ puis en déduire les variations de $f$.
\item Est-ce que $f$ admet un maximum? un minimum? Calculer sa valeur.
\end{enumerate}
\end{exercise}
\begin{solution}
\begin{enumerate}
\item On remplace $x$ par les valeurs demandées
\[
f(1) = 6 \times 1^{2} - 36 \times 1 + 48=6 \times 1 - 36 + 48=6 + 12=18
\]
\[
f(-1) = 6 \times - 1^{2} - 36(- 1) + 48=6 \times 1 + 36 + 48=6 + 84=90
\]
\item Dérivation
\[
f'(x) = 12x - 36
\]
\item Pas de solutions automatiques.
\item Pas de solutions automatiques.
\end{enumerate}
\end{solution}
\begin{exercise}[subtitle={Enclos}]
Dans son garage, Jean a trouvé 27m de grillage. \\
Il décide de l'utiliser pour faire un enclos rectangulaire. Afin d'obtenir un enclos encore plus grand, il veut utiliser le mur du jardin qui formera un côté. Le grillage formera les 3 autres.
\begin{center}
\includegraphics[scale=0.8]{./fig/enclos}
\end{center}
Comment placer les poteaux pour avoir un enclos le plus grand possible ?
\textit{Cet exercice est un exercice de recherche. Vous êtes invité à utiliser les outils que vous voulez. Une part importante de la note sera dédiée à la rédaction et aux explications de ce que vous faites. Vous êtes encouragé à faire des schémas.}
\textit{Si vous utilisez le tableur, vous devrez le mentionner et m'envoyer le tableur à l'adresse \url{benjamin.bertrand@ac-lyon.fr}}
\end{exercise}
\begin{solution}
Formule de l'aire (en notant $x$ la longueur du côté de l'enclos à côté du mur)
\[
A(x) = x(27 - 2x) = - 2x^{2} + 27x
\]
On va donc étudier les variations de la fonction $A(x) = - 2x^{2} + 27x$
\begin{itemize}
\item Fonction dérivée : $A'(x) = - 4x + 27$
\item On résout l'inéquation $A'(x) \geq 0$ pour déterminer quand la fonction $A'$ est positive.
\begin{align*}
A(x) & \geq 0 \\
- 4x + 27 & \geq 0 \\
- 4x + 27 + - 27 &\geq 0 + - 27 \\
- 4x &\geq - 27 \\
\frac{- 4x}{- 4} &\leq \frac{- 27}{- 4} \\
x &\leq \dfrac{27}{4} \\
\end{align*}
Donc $A(x)$ est positif quand $x$ est plus \textbf{petit} que $\dfrac{27}{4}$
\item
\begin{center}
\begin{tikzpicture}
\tkzTabInit[lgt=3,espcl=4]{$x$/1,Signe de $f'(x)$/2, Variations de $f(x)$/2}{, $\dfrac{27}{4}$ ,}%
\tkzTabLine{, +, z, -, }
\tkzTabVar{-/ ,+/$f(\dfrac{27}{4}) = \dfrac{1458}{16}$ , -/}%
\end{tikzpicture}
\end{center}
\end{itemize}
Pour avoir le plus grand enclos, il faut placer les poteaux à 6m du mur et on aura alors un enclos de $72m^2$.
\end{solution}
\end{document}
%%% Local Variables:
%%% mode: latex
%%% TeX-master: "master"
%%% End:

Binary file not shown.

View File

@ -0,0 +1,158 @@
\documentclass[a4paper,12pt]{article}
\usepackage{myXsim}
\usepackage{pgfplots}
\usetikzlibrary{decorations.markings}
\pgfplotsset{compat=1.18}
\title{ DM1 \hfill NAKIR SAILAH Mohamed}
\tribe{1ST}
\date{A rendre pour le lundi 27 mars 2023}
\duree{}
\xsimsetup{
solution/print = false
}
\pagestyle{empty}
\begin{document}
\maketitle
Le barème est donné à titre indicatif, il pourra être modifié.
\begin{exercise}[subtitle={Polynôme de degré 2}]
Développer les expressions suivantes pour vérifier que ce sont des polynômes de degré 2. Vous préciserez les valeurs de $a$, $b$ et $c$.
\begin{multicols}{2}
\begin{enumerate}
\item $f(x) = (- 9x + 2)(- 6x + 2)$
\item $g(x) = (8x + 2)^{2}$
\item $h(x) = 2 + x(- 3x + 5)$
\item $i(x) = 8x^{2} + x(5x + 8)$
\item $j(x) = 10(x + 7)(x - 10)$
\item $k(x) = - 4(x - 7)(x + 5)$
\end{enumerate}
\end{multicols}
\end{exercise}
\begin{solution}
\begin{enumerate}
\item
\begin{align*}
f(x) &= (- 9x + 2)(- 6x + 2)\\&= - 9x \times - 6x - 9x \times 2 + 2 \times - 6x + 2 \times 2\\&= - 9(- 6) \times x^{1 + 1} + 2(- 9) \times x + 2(- 6) \times x + 4\\&= - 18x - 12x + 54x^{2} + 4\\&= (- 18 - 12) \times x + 54x^{2} + 4\\&= 54x^{2} - 30x + 4
\end{align*}
C'est un polynôme de degré 2 avec $a = 54$, $b = - 30$ et $c = 4$.
\item
\begin{align*}
g(x) &= (8x + 2)^{2}\\&= (8x + 2)(8x + 2)\\&= 8x \times 8x + 8x \times 2 + 2 \times 8x + 2 \times 2\\&= 8 \times 8 \times x^{1 + 1} + 2 \times 8 \times x + 2 \times 8 \times x + 4\\&= 16x + 16x + 64x^{2} + 4\\&= (16 + 16) \times x + 64x^{2} + 4\\&= 64x^{2} + 32x + 4
\end{align*}
C'est un polynôme de degré 2 avec $a = 64$, $b = 32$ et $c = 4$.
\item
\begin{align*}
h(x) &= 2 + x(- 3x + 5)\\&= 2 + x \times - 3x + x \times 5\\&= - 3x^{2} + 5x + 2
\end{align*}
C'est un polynôme de degré 2 avec $a = - 3$, $b = 5$ et $c = 2$.
\item
\begin{align*}
i(x) &= 8x^{2} + x(5x + 8)\\&= 8x^{2} + x \times 5x + x \times 8\\&= 8x^{2} + 5x^{2} + 8x\\&= 8x^{2} + 5x^{2} + 8x\\&= (8 + 5) \times x^{2} + 8x\\&= 13x^{2} + 8x
\end{align*}
C'est un polynôme de degré 2 avec $a = 13$, $b = 8$ et $c = 0$.
\item
\begin{align*}
j(x) &= 10(x + 7)(x - 10)\\&= (10x + 10 \times 7)(x - 10)\\&= (10x + 70)(x - 10)\\&= 10x \times x + 10x(- 10) + 70x + 70(- 10)\\&= - 10 \times 10 \times x - 700 + 10x^{2} + 70x\\&= - 100x - 700 + 10x^{2} + 70x\\&= 10x^{2} - 100x + 70x - 700\\&= 10x^{2} + (- 100 + 70) \times x - 700\\&= 10x^{2} - 30x - 700
\end{align*}
C'est un polynôme de degré 2 avec $a = 10$, $b = - 30$ et $c = - 700$.
\item
\begin{align*}
k(x) &= - 4(x - 7)(x + 5)\\&= (- 4x - 4(- 7))(x + 5)\\&= (- 4x + 28)(x + 5)\\&= - 4x \times x - 4x \times 5 + 28x + 28 \times 5\\&= 5(- 4) \times x + 140 - 4x^{2} + 28x\\&= - 20x + 140 - 4x^{2} + 28x\\&= - 4x^{2} - 20x + 28x + 140\\&= - 4x^{2} + (- 20 + 28) \times x + 140\\&= - 4x^{2} + 8x + 140
\end{align*}
C'est un polynôme de degré 2 avec $a = - 4$, $b = 8$ et $c = 140$.
\end{enumerate}
\end{solution}
\begin{exercise}[subtitle={Étude de fonctions}]
Soit $f(x) = 2x^{2} - 28x + 80$ une fonction définie sur $\R$.
\begin{enumerate}
\item Calculer les valeurs suivantes
\[
f(1) \qquad f(-2)
\]
\item Dériver la fonction $f$
\item Étudier le signe de $f'$ puis en déduire les variations de $f$.
\item Est-ce que $f$ admet un maximum? un minimum? Calculer sa valeur.
\end{enumerate}
\end{exercise}
\begin{solution}
\begin{enumerate}
\item On remplace $x$ par les valeurs demandées
\[
f(1) = 2 \times 1^{2} - 28 \times 1 + 80=2 \times 1 - 28 + 80=2 + 52=54
\]
\[
f(-1) = 2 \times - 1^{2} - 28(- 1) + 80=2 \times 1 + 28 + 80=2 + 108=110
\]
\item Dérivation
\[
f'(x) = 4x - 28
\]
\item Pas de solutions automatiques.
\item Pas de solutions automatiques.
\end{enumerate}
\end{solution}
\begin{exercise}[subtitle={Enclos}]
Dans son garage, Jean a trouvé 29m de grillage. \\
Il décide de l'utiliser pour faire un enclos rectangulaire. Afin d'obtenir un enclos encore plus grand, il veut utiliser le mur du jardin qui formera un côté. Le grillage formera les 3 autres.
\begin{center}
\includegraphics[scale=0.8]{./fig/enclos}
\end{center}
Comment placer les poteaux pour avoir un enclos le plus grand possible ?
\textit{Cet exercice est un exercice de recherche. Vous êtes invité à utiliser les outils que vous voulez. Une part importante de la note sera dédiée à la rédaction et aux explications de ce que vous faites. Vous êtes encouragé à faire des schémas.}
\textit{Si vous utilisez le tableur, vous devrez le mentionner et m'envoyer le tableur à l'adresse \url{benjamin.bertrand@ac-lyon.fr}}
\end{exercise}
\begin{solution}
Formule de l'aire (en notant $x$ la longueur du côté de l'enclos à côté du mur)
\[
A(x) = x(29 - 2x) = - 2x^{2} + 29x
\]
On va donc étudier les variations de la fonction $A(x) = - 2x^{2} + 29x$
\begin{itemize}
\item Fonction dérivée : $A'(x) = - 4x + 29$
\item On résout l'inéquation $A'(x) \geq 0$ pour déterminer quand la fonction $A'$ est positive.
\begin{align*}
A(x) & \geq 0 \\
- 4x + 29 & \geq 0 \\
- 4x + 29 + - 29 &\geq 0 + - 29 \\
- 4x &\geq - 29 \\
\frac{- 4x}{- 4} &\leq \frac{- 29}{- 4} \\
x &\leq \dfrac{29}{4} \\
\end{align*}
Donc $A(x)$ est positif quand $x$ est plus \textbf{petit} que $\dfrac{29}{4}$
\item
\begin{center}
\begin{tikzpicture}
\tkzTabInit[lgt=3,espcl=4]{$x$/1,Signe de $f'(x)$/2, Variations de $f(x)$/2}{, $\dfrac{29}{4}$ ,}%
\tkzTabLine{, +, z, -, }
\tkzTabVar{-/ ,+/$f(\dfrac{29}{4}) = \dfrac{1682}{16}$ , -/}%
\end{tikzpicture}
\end{center}
\end{itemize}
Pour avoir le plus grand enclos, il faut placer les poteaux à 6m du mur et on aura alors un enclos de $72m^2$.
\end{solution}
\end{document}
%%% Local Variables:
%%% mode: latex
%%% TeX-master: "master"
%%% End:

Binary file not shown.

View File

@ -0,0 +1,158 @@
\documentclass[a4paper,12pt]{article}
\usepackage{myXsim}
\usepackage{pgfplots}
\usetikzlibrary{decorations.markings}
\pgfplotsset{compat=1.18}
\title{ DM1 \hfill NEIVA Diego}
\tribe{1ST}
\date{A rendre pour le lundi 27 mars 2023}
\duree{}
\xsimsetup{
solution/print = false
}
\pagestyle{empty}
\begin{document}
\maketitle
Le barème est donné à titre indicatif, il pourra être modifié.
\begin{exercise}[subtitle={Polynôme de degré 2}]
Développer les expressions suivantes pour vérifier que ce sont des polynômes de degré 2. Vous préciserez les valeurs de $a$, $b$ et $c$.
\begin{multicols}{2}
\begin{enumerate}
\item $f(x) = (10x + 6)(5x + 6)$
\item $g(x) = (- 3x - 8)^{2}$
\item $h(x) = 5 + x(6x + 10)$
\item $i(x) = 2x^{2} + x(3x + 3)$
\item $j(x) = 10(x - 2)(x + 8)$
\item $k(x) = - 8(x - 1)(x - 9)$
\end{enumerate}
\end{multicols}
\end{exercise}
\begin{solution}
\begin{enumerate}
\item
\begin{align*}
f(x) &= (10x + 6)(5x + 6)\\&= 10x \times 5x + 10x \times 6 + 6 \times 5x + 6 \times 6\\&= 10 \times 5 \times x^{1 + 1} + 6 \times 10 \times x + 6 \times 5 \times x + 36\\&= 60x + 30x + 50x^{2} + 36\\&= (60 + 30) \times x + 50x^{2} + 36\\&= 50x^{2} + 90x + 36
\end{align*}
C'est un polynôme de degré 2 avec $a = 50$, $b = 90$ et $c = 36$.
\item
\begin{align*}
g(x) &= (- 3x - 8)^{2}\\&= (- 3x - 8)(- 3x - 8)\\&= - 3x \times - 3x - 3x(- 8) - 8 \times - 3x - 8(- 8)\\&= - 3(- 3) \times x^{1 + 1} - 8(- 3) \times x - 8(- 3) \times x + 64\\&= 24x + 24x + 9x^{2} + 64\\&= (24 + 24) \times x + 9x^{2} + 64\\&= 9x^{2} + 48x + 64
\end{align*}
C'est un polynôme de degré 2 avec $a = 9$, $b = 48$ et $c = 64$.
\item
\begin{align*}
h(x) &= 5 + x(6x + 10)\\&= 5 + x \times 6x + x \times 10\\&= 6x^{2} + 10x + 5
\end{align*}
C'est un polynôme de degré 2 avec $a = 6$, $b = 10$ et $c = 5$.
\item
\begin{align*}
i(x) &= 2x^{2} + x(3x + 3)\\&= 2x^{2} + x \times 3x + x \times 3\\&= 2x^{2} + 3x^{2} + 3x\\&= 2x^{2} + 3x^{2} + 3x\\&= (2 + 3) \times x^{2} + 3x\\&= 5x^{2} + 3x
\end{align*}
C'est un polynôme de degré 2 avec $a = 5$, $b = 3$ et $c = 0$.
\item
\begin{align*}
j(x) &= 10(x - 2)(x + 8)\\&= (10x + 10(- 2))(x + 8)\\&= (10x - 20)(x + 8)\\&= 10x \times x + 10x \times 8 - 20x - 20 \times 8\\&= 8 \times 10 \times x - 160 + 10x^{2} - 20x\\&= 80x - 160 + 10x^{2} - 20x\\&= 10x^{2} + 80x - 20x - 160\\&= 10x^{2} + (80 - 20) \times x - 160\\&= 10x^{2} + 60x - 160
\end{align*}
C'est un polynôme de degré 2 avec $a = 10$, $b = 60$ et $c = - 160$.
\item
\begin{align*}
k(x) &= - 8(x - 1)(x - 9)\\&= (- 8x - 8(- 1))(x - 9)\\&= (- 8x + 8)(x - 9)\\&= - 8x \times x - 8x(- 9) + 8x + 8(- 9)\\&= - 9(- 8) \times x - 72 - 8x^{2} + 8x\\&= 72x - 72 - 8x^{2} + 8x\\&= - 8x^{2} + 72x + 8x - 72\\&= - 8x^{2} + (72 + 8) \times x - 72\\&= - 8x^{2} + 80x - 72
\end{align*}
C'est un polynôme de degré 2 avec $a = - 8$, $b = 80$ et $c = - 72$.
\end{enumerate}
\end{solution}
\begin{exercise}[subtitle={Étude de fonctions}]
Soit $f(x) = - 4x^{2} - 8x + 60$ une fonction définie sur $\R$.
\begin{enumerate}
\item Calculer les valeurs suivantes
\[
f(1) \qquad f(-2)
\]
\item Dériver la fonction $f$
\item Étudier le signe de $f'$ puis en déduire les variations de $f$.
\item Est-ce que $f$ admet un maximum? un minimum? Calculer sa valeur.
\end{enumerate}
\end{exercise}
\begin{solution}
\begin{enumerate}
\item On remplace $x$ par les valeurs demandées
\[
f(1) = - 4 \times 1^{2} - 8 \times 1 + 60=- 4 \times 1 - 8 + 60=- 4 + 52=48
\]
\[
f(-1) = - 4 \times - 1^{2} - 8(- 1) + 60=- 4 \times 1 + 8 + 60=- 4 + 68=64
\]
\item Dérivation
\[
f'(x) = - 8x - 8
\]
\item Pas de solutions automatiques.
\item Pas de solutions automatiques.
\end{enumerate}
\end{solution}
\begin{exercise}[subtitle={Enclos}]
Dans son garage, Jean a trouvé 35m de grillage. \\
Il décide de l'utiliser pour faire un enclos rectangulaire. Afin d'obtenir un enclos encore plus grand, il veut utiliser le mur du jardin qui formera un côté. Le grillage formera les 3 autres.
\begin{center}
\includegraphics[scale=0.8]{./fig/enclos}
\end{center}
Comment placer les poteaux pour avoir un enclos le plus grand possible ?
\textit{Cet exercice est un exercice de recherche. Vous êtes invité à utiliser les outils que vous voulez. Une part importante de la note sera dédiée à la rédaction et aux explications de ce que vous faites. Vous êtes encouragé à faire des schémas.}
\textit{Si vous utilisez le tableur, vous devrez le mentionner et m'envoyer le tableur à l'adresse \url{benjamin.bertrand@ac-lyon.fr}}
\end{exercise}
\begin{solution}
Formule de l'aire (en notant $x$ la longueur du côté de l'enclos à côté du mur)
\[
A(x) = x(35 - 2x) = - 2x^{2} + 35x
\]
On va donc étudier les variations de la fonction $A(x) = - 2x^{2} + 35x$
\begin{itemize}
\item Fonction dérivée : $A'(x) = - 4x + 35$
\item On résout l'inéquation $A'(x) \geq 0$ pour déterminer quand la fonction $A'$ est positive.
\begin{align*}
A(x) & \geq 0 \\
- 4x + 35 & \geq 0 \\
- 4x + 35 + - 35 &\geq 0 + - 35 \\
- 4x &\geq - 35 \\
\frac{- 4x}{- 4} &\leq \frac{- 35}{- 4} \\
x &\leq \dfrac{35}{4} \\
\end{align*}
Donc $A(x)$ est positif quand $x$ est plus \textbf{petit} que $\dfrac{35}{4}$
\item
\begin{center}
\begin{tikzpicture}
\tkzTabInit[lgt=3,espcl=4]{$x$/1,Signe de $f'(x)$/2, Variations de $f(x)$/2}{, $\dfrac{35}{4}$ ,}%
\tkzTabLine{, +, z, -, }
\tkzTabVar{-/ ,+/$f(\dfrac{35}{4}) = \dfrac{2450}{16}$ , -/}%
\end{tikzpicture}
\end{center}
\end{itemize}
Pour avoir le plus grand enclos, il faut placer les poteaux à 6m du mur et on aura alors un enclos de $72m^2$.
\end{solution}
\end{document}
%%% Local Variables:
%%% mode: latex
%%% TeX-master: "master"
%%% End:

Binary file not shown.

View File

@ -0,0 +1,158 @@
\documentclass[a4paper,12pt]{article}
\usepackage{myXsim}
\usepackage{pgfplots}
\usetikzlibrary{decorations.markings}
\pgfplotsset{compat=1.18}
\title{ DM1 \hfill SAADI Yazid}
\tribe{1ST}
\date{A rendre pour le lundi 27 mars 2023}
\duree{}
\xsimsetup{
solution/print = false
}
\pagestyle{empty}
\begin{document}
\maketitle
Le barème est donné à titre indicatif, il pourra être modifié.
\begin{exercise}[subtitle={Polynôme de degré 2}]
Développer les expressions suivantes pour vérifier que ce sont des polynômes de degré 2. Vous préciserez les valeurs de $a$, $b$ et $c$.
\begin{multicols}{2}
\begin{enumerate}
\item $f(x) = (- 8x - 2)(- 6x - 2)$
\item $g(x) = (7x + 8)^{2}$
\item $h(x) = - 7 + x(- 1x - 10)$
\item $i(x) = 2x^{2} + x(6x + 6)$
\item $j(x) = - 9(x - 9)(x + 10)$
\item $k(x) = 4(x - 9)(x - 8)$
\end{enumerate}
\end{multicols}
\end{exercise}
\begin{solution}
\begin{enumerate}
\item
\begin{align*}
f(x) &= (- 8x - 2)(- 6x - 2)\\&= - 8x \times - 6x - 8x(- 2) - 2 \times - 6x - 2(- 2)\\&= - 8(- 6) \times x^{1 + 1} - 2(- 8) \times x - 2(- 6) \times x + 4\\&= 16x + 12x + 48x^{2} + 4\\&= (16 + 12) \times x + 48x^{2} + 4\\&= 48x^{2} + 28x + 4
\end{align*}
C'est un polynôme de degré 2 avec $a = 48$, $b = 28$ et $c = 4$.
\item
\begin{align*}
g(x) &= (7x + 8)^{2}\\&= (7x + 8)(7x + 8)\\&= 7x \times 7x + 7x \times 8 + 8 \times 7x + 8 \times 8\\&= 7 \times 7 \times x^{1 + 1} + 8 \times 7 \times x + 8 \times 7 \times x + 64\\&= 56x + 56x + 49x^{2} + 64\\&= (56 + 56) \times x + 49x^{2} + 64\\&= 49x^{2} + 112x + 64
\end{align*}
C'est un polynôme de degré 2 avec $a = 49$, $b = 112$ et $c = 64$.
\item
\begin{align*}
h(x) &= - 7 + x(- 1x - 10)\\&= - 7 + x(- x) + x(- 10)\\&= - x^{2} - 10x - 7
\end{align*}
C'est un polynôme de degré 2 avec $a = - 1$, $b = - 10$ et $c = - 7$.
\item
\begin{align*}
i(x) &= 2x^{2} + x(6x + 6)\\&= 2x^{2} + x \times 6x + x \times 6\\&= 2x^{2} + 6x^{2} + 6x\\&= 2x^{2} + 6x^{2} + 6x\\&= (2 + 6) \times x^{2} + 6x\\&= 8x^{2} + 6x
\end{align*}
C'est un polynôme de degré 2 avec $a = 8$, $b = 6$ et $c = 0$.
\item
\begin{align*}
j(x) &= - 9(x - 9)(x + 10)\\&= (- 9x - 9(- 9))(x + 10)\\&= (- 9x + 81)(x + 10)\\&= - 9x \times x - 9x \times 10 + 81x + 81 \times 10\\&= 10(- 9) \times x + 810 - 9x^{2} + 81x\\&= - 90x + 810 - 9x^{2} + 81x\\&= - 9x^{2} - 90x + 81x + 810\\&= - 9x^{2} + (- 90 + 81) \times x + 810\\&= - 9x^{2} - 9x + 810
\end{align*}
C'est un polynôme de degré 2 avec $a = - 9$, $b = - 9$ et $c = 810$.
\item
\begin{align*}
k(x) &= 4(x - 9)(x - 8)\\&= (4x + 4(- 9))(x - 8)\\&= (4x - 36)(x - 8)\\&= 4x \times x + 4x(- 8) - 36x - 36(- 8)\\&= - 8 \times 4 \times x + 288 + 4x^{2} - 36x\\&= - 32x + 288 + 4x^{2} - 36x\\&= 4x^{2} - 32x - 36x + 288\\&= 4x^{2} + (- 32 - 36) \times x + 288\\&= 4x^{2} - 68x + 288
\end{align*}
C'est un polynôme de degré 2 avec $a = 4$, $b = - 68$ et $c = 288$.
\end{enumerate}
\end{solution}
\begin{exercise}[subtitle={Étude de fonctions}]
Soit $f(x) = 3x^{2} - 39x + 90$ une fonction définie sur $\R$.
\begin{enumerate}
\item Calculer les valeurs suivantes
\[
f(1) \qquad f(-2)
\]
\item Dériver la fonction $f$
\item Étudier le signe de $f'$ puis en déduire les variations de $f$.
\item Est-ce que $f$ admet un maximum? un minimum? Calculer sa valeur.
\end{enumerate}
\end{exercise}
\begin{solution}
\begin{enumerate}
\item On remplace $x$ par les valeurs demandées
\[
f(1) = 3 \times 1^{2} - 39 \times 1 + 90=3 \times 1 - 39 + 90=3 + 51=54
\]
\[
f(-1) = 3 \times - 1^{2} - 39(- 1) + 90=3 \times 1 + 39 + 90=3 + 129=132
\]
\item Dérivation
\[
f'(x) = 6x - 39
\]
\item Pas de solutions automatiques.
\item Pas de solutions automatiques.
\end{enumerate}
\end{solution}
\begin{exercise}[subtitle={Enclos}]
Dans son garage, Jean a trouvé 20m de grillage. \\
Il décide de l'utiliser pour faire un enclos rectangulaire. Afin d'obtenir un enclos encore plus grand, il veut utiliser le mur du jardin qui formera un côté. Le grillage formera les 3 autres.
\begin{center}
\includegraphics[scale=0.8]{./fig/enclos}
\end{center}
Comment placer les poteaux pour avoir un enclos le plus grand possible ?
\textit{Cet exercice est un exercice de recherche. Vous êtes invité à utiliser les outils que vous voulez. Une part importante de la note sera dédiée à la rédaction et aux explications de ce que vous faites. Vous êtes encouragé à faire des schémas.}
\textit{Si vous utilisez le tableur, vous devrez le mentionner et m'envoyer le tableur à l'adresse \url{benjamin.bertrand@ac-lyon.fr}}
\end{exercise}
\begin{solution}
Formule de l'aire (en notant $x$ la longueur du côté de l'enclos à côté du mur)
\[
A(x) = x(20 - 2x) = - 2x^{2} + 20x
\]
On va donc étudier les variations de la fonction $A(x) = - 2x^{2} + 20x$
\begin{itemize}
\item Fonction dérivée : $A'(x) = - 4x + 20$
\item On résout l'inéquation $A'(x) \geq 0$ pour déterminer quand la fonction $A'$ est positive.
\begin{align*}
A(x) & \geq 0 \\
- 4x + 20 & \geq 0 \\
- 4x + 20 + - 20 &\geq 0 + - 20 \\
- 4x &\geq - 20 \\
\frac{- 4x}{- 4} &\leq \frac{- 20}{- 4} \\
x &\leq 5 \\
\end{align*}
Donc $A(x)$ est positif quand $x$ est plus \textbf{petit} que $5$
\item
\begin{center}
\begin{tikzpicture}
\tkzTabInit[lgt=3,espcl=4]{$x$/1,Signe de $f'(x)$/2, Variations de $f(x)$/2}{, $5$ ,}%
\tkzTabLine{, +, z, -, }
\tkzTabVar{-/ ,+/$f(5) = 50$ , -/}%
\end{tikzpicture}
\end{center}
\end{itemize}
Pour avoir le plus grand enclos, il faut placer les poteaux à 6m du mur et on aura alors un enclos de $72m^2$.
\end{solution}
\end{document}
%%% Local Variables:
%%% mode: latex
%%% TeX-master: "master"
%%% End:

Binary file not shown.

View File

@ -0,0 +1,158 @@
\documentclass[a4paper,12pt]{article}
\usepackage{myXsim}
\usepackage{pgfplots}
\usetikzlibrary{decorations.markings}
\pgfplotsset{compat=1.18}
\title{ DM1 \hfill SCOPELLITI Martina}
\tribe{1ST}
\date{A rendre pour le lundi 27 mars 2023}
\duree{}
\xsimsetup{
solution/print = false
}
\pagestyle{empty}
\begin{document}
\maketitle
Le barème est donné à titre indicatif, il pourra être modifié.
\begin{exercise}[subtitle={Polynôme de degré 2}]
Développer les expressions suivantes pour vérifier que ce sont des polynômes de degré 2. Vous préciserez les valeurs de $a$, $b$ et $c$.
\begin{multicols}{2}
\begin{enumerate}
\item $f(x) = (- 1x - 10)(6x - 10)$
\item $g(x) = (- 8x - 7)^{2}$
\item $h(x) = - 2 + x(6x - 10)$
\item $i(x) = 5x^{2} + x(- 4x + 9)$
\item $j(x) = - 10(x + 7)(x + 3)$
\item $k(x) = 4(x - 10)(x - 3)$
\end{enumerate}
\end{multicols}
\end{exercise}
\begin{solution}
\begin{enumerate}
\item
\begin{align*}
f(x) &= (- 1x - 10)(6x - 10)\\&= (- x) \times 6x + (- x)(- 10) - 10 \times 6x - 10(- 10)\\&= - 1 \times 6 \times x^{1 + 1} - 10(- 1) \times x - 10 \times 6 \times x + 100\\&= 10x - 60x - 6x^{2} + 100\\&= (10 - 60) \times x - 6x^{2} + 100\\&= - 6x^{2} - 50x + 100
\end{align*}
C'est un polynôme de degré 2 avec $a = - 6$, $b = - 50$ et $c = 100$.
\item
\begin{align*}
g(x) &= (- 8x - 7)^{2}\\&= (- 8x - 7)(- 8x - 7)\\&= - 8x \times - 8x - 8x(- 7) - 7 \times - 8x - 7(- 7)\\&= - 8(- 8) \times x^{1 + 1} - 7(- 8) \times x - 7(- 8) \times x + 49\\&= 56x + 56x + 64x^{2} + 49\\&= (56 + 56) \times x + 64x^{2} + 49\\&= 64x^{2} + 112x + 49
\end{align*}
C'est un polynôme de degré 2 avec $a = 64$, $b = 112$ et $c = 49$.
\item
\begin{align*}
h(x) &= - 2 + x(6x - 10)\\&= - 2 + x \times 6x + x(- 10)\\&= 6x^{2} - 10x - 2
\end{align*}
C'est un polynôme de degré 2 avec $a = 6$, $b = - 10$ et $c = - 2$.
\item
\begin{align*}
i(x) &= 5x^{2} + x(- 4x + 9)\\&= 5x^{2} + x \times - 4x + x \times 9\\&= 5x^{2} - 4x^{2} + 9x\\&= 5x^{2} - 4x^{2} + 9x\\&= (5 - 4) \times x^{2} + 9x\\&= x^{2} + 9x
\end{align*}
C'est un polynôme de degré 2 avec $a = 1$, $b = 9$ et $c = 0$.
\item
\begin{align*}
j(x) &= - 10(x + 7)(x + 3)\\&= (- 10x - 10 \times 7)(x + 3)\\&= (- 10x - 70)(x + 3)\\&= - 10x \times x - 10x \times 3 - 70x - 70 \times 3\\&= 3(- 10) \times x - 210 - 10x^{2} - 70x\\&= - 30x - 210 - 10x^{2} - 70x\\&= - 10x^{2} - 30x - 70x - 210\\&= - 10x^{2} + (- 30 - 70) \times x - 210\\&= - 10x^{2} - 100x - 210
\end{align*}
C'est un polynôme de degré 2 avec $a = - 10$, $b = - 100$ et $c = - 210$.
\item
\begin{align*}
k(x) &= 4(x - 10)(x - 3)\\&= (4x + 4(- 10))(x - 3)\\&= (4x - 40)(x - 3)\\&= 4x \times x + 4x(- 3) - 40x - 40(- 3)\\&= - 3 \times 4 \times x + 120 + 4x^{2} - 40x\\&= - 12x + 120 + 4x^{2} - 40x\\&= 4x^{2} - 12x - 40x + 120\\&= 4x^{2} + (- 12 - 40) \times x + 120\\&= 4x^{2} - 52x + 120
\end{align*}
C'est un polynôme de degré 2 avec $a = 4$, $b = - 52$ et $c = 120$.
\end{enumerate}
\end{solution}
\begin{exercise}[subtitle={Étude de fonctions}]
Soit $f(x) = - 9x^{2} - 153x - 630$ une fonction définie sur $\R$.
\begin{enumerate}
\item Calculer les valeurs suivantes
\[
f(1) \qquad f(-2)
\]
\item Dériver la fonction $f$
\item Étudier le signe de $f'$ puis en déduire les variations de $f$.
\item Est-ce que $f$ admet un maximum? un minimum? Calculer sa valeur.
\end{enumerate}
\end{exercise}
\begin{solution}
\begin{enumerate}
\item On remplace $x$ par les valeurs demandées
\[
f(1) = - 9 \times 1^{2} - 153 \times 1 - 630=- 9 \times 1 - 153 - 630=- 9 - 783=- 792
\]
\[
f(-1) = - 9 \times - 1^{2} - 153(- 1) - 630=- 9 \times 1 + 153 - 630=- 9 - 477=- 486
\]
\item Dérivation
\[
f'(x) = - 18x - 153
\]
\item Pas de solutions automatiques.
\item Pas de solutions automatiques.
\end{enumerate}
\end{solution}
\begin{exercise}[subtitle={Enclos}]
Dans son garage, Jean a trouvé 28m de grillage. \\
Il décide de l'utiliser pour faire un enclos rectangulaire. Afin d'obtenir un enclos encore plus grand, il veut utiliser le mur du jardin qui formera un côté. Le grillage formera les 3 autres.
\begin{center}
\includegraphics[scale=0.8]{./fig/enclos}
\end{center}
Comment placer les poteaux pour avoir un enclos le plus grand possible ?
\textit{Cet exercice est un exercice de recherche. Vous êtes invité à utiliser les outils que vous voulez. Une part importante de la note sera dédiée à la rédaction et aux explications de ce que vous faites. Vous êtes encouragé à faire des schémas.}
\textit{Si vous utilisez le tableur, vous devrez le mentionner et m'envoyer le tableur à l'adresse \url{benjamin.bertrand@ac-lyon.fr}}
\end{exercise}
\begin{solution}
Formule de l'aire (en notant $x$ la longueur du côté de l'enclos à côté du mur)
\[
A(x) = x(28 - 2x) = - 2x^{2} + 28x
\]
On va donc étudier les variations de la fonction $A(x) = - 2x^{2} + 28x$
\begin{itemize}
\item Fonction dérivée : $A'(x) = - 4x + 28$
\item On résout l'inéquation $A'(x) \geq 0$ pour déterminer quand la fonction $A'$ est positive.
\begin{align*}
A(x) & \geq 0 \\
- 4x + 28 & \geq 0 \\
- 4x + 28 + - 28 &\geq 0 + - 28 \\
- 4x &\geq - 28 \\
\frac{- 4x}{- 4} &\leq \frac{- 28}{- 4} \\
x &\leq 7 \\
\end{align*}
Donc $A(x)$ est positif quand $x$ est plus \textbf{petit} que $7$
\item
\begin{center}
\begin{tikzpicture}
\tkzTabInit[lgt=3,espcl=4]{$x$/1,Signe de $f'(x)$/2, Variations de $f(x)$/2}{, $7$ ,}%
\tkzTabLine{, +, z, -, }
\tkzTabVar{-/ ,+/$f(7) = 98$ , -/}%
\end{tikzpicture}
\end{center}
\end{itemize}
Pour avoir le plus grand enclos, il faut placer les poteaux à 6m du mur et on aura alors un enclos de $72m^2$.
\end{solution}
\end{document}
%%% Local Variables:
%%% mode: latex
%%% TeX-master: "master"
%%% End:

Binary file not shown.

View File

@ -0,0 +1,158 @@
\documentclass[a4paper,12pt]{article}
\usepackage{myXsim}
\usepackage{pgfplots}
\usetikzlibrary{decorations.markings}
\pgfplotsset{compat=1.18}
\title{ DM1 \hfill SORIANO Johan}
\tribe{1ST}
\date{A rendre pour le lundi 27 mars 2023}
\duree{}
\xsimsetup{
solution/print = false
}
\pagestyle{empty}
\begin{document}
\maketitle
Le barème est donné à titre indicatif, il pourra être modifié.
\begin{exercise}[subtitle={Polynôme de degré 2}]
Développer les expressions suivantes pour vérifier que ce sont des polynômes de degré 2. Vous préciserez les valeurs de $a$, $b$ et $c$.
\begin{multicols}{2}
\begin{enumerate}
\item $f(x) = (5x - 2)(5x - 2)$
\item $g(x) = (- 4x - 4)^{2}$
\item $h(x) = - 9 + x(- 9x + 5)$
\item $i(x) = - 4x^{2} + x(5x - 4)$
\item $j(x) = 3(x - 10)(x - 6)$
\item $k(x) = 6(x + 7)(x - 9)$
\end{enumerate}
\end{multicols}
\end{exercise}
\begin{solution}
\begin{enumerate}
\item
\begin{align*}
f(x) &= (5x - 2)(5x - 2)\\&= 5x \times 5x + 5x(- 2) - 2 \times 5x - 2(- 2)\\&= 5 \times 5 \times x^{1 + 1} - 2 \times 5 \times x - 2 \times 5 \times x + 4\\&= - 10x - 10x + 25x^{2} + 4\\&= (- 10 - 10) \times x + 25x^{2} + 4\\&= 25x^{2} - 20x + 4
\end{align*}
C'est un polynôme de degré 2 avec $a = 25$, $b = - 20$ et $c = 4$.
\item
\begin{align*}
g(x) &= (- 4x - 4)^{2}\\&= (- 4x - 4)(- 4x - 4)\\&= - 4x \times - 4x - 4x(- 4) - 4 \times - 4x - 4(- 4)\\&= - 4(- 4) \times x^{1 + 1} - 4(- 4) \times x - 4(- 4) \times x + 16\\&= 16x + 16x + 16x^{2} + 16\\&= (16 + 16) \times x + 16x^{2} + 16\\&= 16x^{2} + 32x + 16
\end{align*}
C'est un polynôme de degré 2 avec $a = 16$, $b = 32$ et $c = 16$.
\item
\begin{align*}
h(x) &= - 9 + x(- 9x + 5)\\&= - 9 + x \times - 9x + x \times 5\\&= - 9x^{2} + 5x - 9
\end{align*}
C'est un polynôme de degré 2 avec $a = - 9$, $b = 5$ et $c = - 9$.
\item
\begin{align*}
i(x) &= - 4x^{2} + x(5x - 4)\\&= - 4x^{2} + x \times 5x + x(- 4)\\&= - 4x^{2} + 5x^{2} - 4x\\&= - 4x^{2} + 5x^{2} - 4x\\&= (- 4 + 5) \times x^{2} - 4x\\&= x^{2} - 4x
\end{align*}
C'est un polynôme de degré 2 avec $a = 1$, $b = - 4$ et $c = 0$.
\item
\begin{align*}
j(x) &= 3(x - 10)(x - 6)\\&= (3x + 3(- 10))(x - 6)\\&= (3x - 30)(x - 6)\\&= 3x \times x + 3x(- 6) - 30x - 30(- 6)\\&= - 6 \times 3 \times x + 180 + 3x^{2} - 30x\\&= - 18x + 180 + 3x^{2} - 30x\\&= 3x^{2} - 18x - 30x + 180\\&= 3x^{2} + (- 18 - 30) \times x + 180\\&= 3x^{2} - 48x + 180
\end{align*}
C'est un polynôme de degré 2 avec $a = 3$, $b = - 48$ et $c = 180$.
\item
\begin{align*}
k(x) &= 6(x + 7)(x - 9)\\&= (6x + 6 \times 7)(x - 9)\\&= (6x + 42)(x - 9)\\&= 6x \times x + 6x(- 9) + 42x + 42(- 9)\\&= - 9 \times 6 \times x - 378 + 6x^{2} + 42x\\&= - 54x - 378 + 6x^{2} + 42x\\&= 6x^{2} - 54x + 42x - 378\\&= 6x^{2} + (- 54 + 42) \times x - 378\\&= 6x^{2} - 12x - 378
\end{align*}
C'est un polynôme de degré 2 avec $a = 6$, $b = - 12$ et $c = - 378$.
\end{enumerate}
\end{solution}
\begin{exercise}[subtitle={Étude de fonctions}]
Soit $f(x) = - 7x^{2} + 21x + 28$ une fonction définie sur $\R$.
\begin{enumerate}
\item Calculer les valeurs suivantes
\[
f(1) \qquad f(-2)
\]
\item Dériver la fonction $f$
\item Étudier le signe de $f'$ puis en déduire les variations de $f$.
\item Est-ce que $f$ admet un maximum? un minimum? Calculer sa valeur.
\end{enumerate}
\end{exercise}
\begin{solution}
\begin{enumerate}
\item On remplace $x$ par les valeurs demandées
\[
f(1) = - 7 \times 1^{2} + 21 \times 1 + 28=- 7 \times 1 + 21 + 28=- 7 + 49=42
\]
\[
f(-1) = - 7 \times - 1^{2} + 21(- 1) + 28=- 7 \times 1 - 21 + 28=- 7 + 7=0
\]
\item Dérivation
\[
f'(x) = - 14x + 21
\]
\item Pas de solutions automatiques.
\item Pas de solutions automatiques.
\end{enumerate}
\end{solution}
\begin{exercise}[subtitle={Enclos}]
Dans son garage, Jean a trouvé 17m de grillage. \\
Il décide de l'utiliser pour faire un enclos rectangulaire. Afin d'obtenir un enclos encore plus grand, il veut utiliser le mur du jardin qui formera un côté. Le grillage formera les 3 autres.
\begin{center}
\includegraphics[scale=0.8]{./fig/enclos}
\end{center}
Comment placer les poteaux pour avoir un enclos le plus grand possible ?
\textit{Cet exercice est un exercice de recherche. Vous êtes invité à utiliser les outils que vous voulez. Une part importante de la note sera dédiée à la rédaction et aux explications de ce que vous faites. Vous êtes encouragé à faire des schémas.}
\textit{Si vous utilisez le tableur, vous devrez le mentionner et m'envoyer le tableur à l'adresse \url{benjamin.bertrand@ac-lyon.fr}}
\end{exercise}
\begin{solution}
Formule de l'aire (en notant $x$ la longueur du côté de l'enclos à côté du mur)
\[
A(x) = x(17 - 2x) = - 2x^{2} + 17x
\]
On va donc étudier les variations de la fonction $A(x) = - 2x^{2} + 17x$
\begin{itemize}
\item Fonction dérivée : $A'(x) = - 4x + 17$
\item On résout l'inéquation $A'(x) \geq 0$ pour déterminer quand la fonction $A'$ est positive.
\begin{align*}
A(x) & \geq 0 \\
- 4x + 17 & \geq 0 \\
- 4x + 17 + - 17 &\geq 0 + - 17 \\
- 4x &\geq - 17 \\
\frac{- 4x}{- 4} &\leq \frac{- 17}{- 4} \\
x &\leq \dfrac{17}{4} \\
\end{align*}
Donc $A(x)$ est positif quand $x$ est plus \textbf{petit} que $\dfrac{17}{4}$
\item
\begin{center}
\begin{tikzpicture}
\tkzTabInit[lgt=3,espcl=4]{$x$/1,Signe de $f'(x)$/2, Variations de $f(x)$/2}{, $\dfrac{17}{4}$ ,}%
\tkzTabLine{, +, z, -, }
\tkzTabVar{-/ ,+/$f(\dfrac{17}{4}) = \dfrac{578}{16}$ , -/}%
\end{tikzpicture}
\end{center}
\end{itemize}
Pour avoir le plus grand enclos, il faut placer les poteaux à 6m du mur et on aura alors un enclos de $72m^2$.
\end{solution}
\end{document}
%%% Local Variables:
%%% mode: latex
%%% TeX-master: "master"
%%% End:

Binary file not shown.

View File

@ -0,0 +1,158 @@
\documentclass[a4paper,12pt]{article}
\usepackage{myXsim}
\usepackage{pgfplots}
\usetikzlibrary{decorations.markings}
\pgfplotsset{compat=1.18}
\title{ DM1 \hfill STRUKELJ Charles}
\tribe{1ST}
\date{A rendre pour le lundi 27 mars 2023}
\duree{}
\xsimsetup{
solution/print = false
}
\pagestyle{empty}
\begin{document}
\maketitle
Le barème est donné à titre indicatif, il pourra être modifié.
\begin{exercise}[subtitle={Polynôme de degré 2}]
Développer les expressions suivantes pour vérifier que ce sont des polynômes de degré 2. Vous préciserez les valeurs de $a$, $b$ et $c$.
\begin{multicols}{2}
\begin{enumerate}
\item $f(x) = (8x + 9)(4x + 9)$
\item $g(x) = (3x - 7)^{2}$
\item $h(x) = 2 + x(5x - 7)$
\item $i(x) = - 2x^{2} + x(- 3x - 10)$
\item $j(x) = - 4(x - 1)(x - 5)$
\item $k(x) = - 6(x + 4)(x + 8)$
\end{enumerate}
\end{multicols}
\end{exercise}
\begin{solution}
\begin{enumerate}
\item
\begin{align*}
f(x) &= (8x + 9)(4x + 9)\\&= 8x \times 4x + 8x \times 9 + 9 \times 4x + 9 \times 9\\&= 8 \times 4 \times x^{1 + 1} + 9 \times 8 \times x + 9 \times 4 \times x + 81\\&= 72x + 36x + 32x^{2} + 81\\&= (72 + 36) \times x + 32x^{2} + 81\\&= 32x^{2} + 108x + 81
\end{align*}
C'est un polynôme de degré 2 avec $a = 32$, $b = 108$ et $c = 81$.
\item
\begin{align*}
g(x) &= (3x - 7)^{2}\\&= (3x - 7)(3x - 7)\\&= 3x \times 3x + 3x(- 7) - 7 \times 3x - 7(- 7)\\&= 3 \times 3 \times x^{1 + 1} - 7 \times 3 \times x - 7 \times 3 \times x + 49\\&= - 21x - 21x + 9x^{2} + 49\\&= (- 21 - 21) \times x + 9x^{2} + 49\\&= 9x^{2} - 42x + 49
\end{align*}
C'est un polynôme de degré 2 avec $a = 9$, $b = - 42$ et $c = 49$.
\item
\begin{align*}
h(x) &= 2 + x(5x - 7)\\&= 2 + x \times 5x + x(- 7)\\&= 5x^{2} - 7x + 2
\end{align*}
C'est un polynôme de degré 2 avec $a = 5$, $b = - 7$ et $c = 2$.
\item
\begin{align*}
i(x) &= - 2x^{2} + x(- 3x - 10)\\&= - 2x^{2} + x \times - 3x + x(- 10)\\&= - 2x^{2} - 3x^{2} - 10x\\&= - 2x^{2} - 3x^{2} - 10x\\&= (- 2 - 3) \times x^{2} - 10x\\&= - 5x^{2} - 10x
\end{align*}
C'est un polynôme de degré 2 avec $a = - 5$, $b = - 10$ et $c = 0$.
\item
\begin{align*}
j(x) &= - 4(x - 1)(x - 5)\\&= (- 4x - 4(- 1))(x - 5)\\&= (- 4x + 4)(x - 5)\\&= - 4x \times x - 4x(- 5) + 4x + 4(- 5)\\&= - 5(- 4) \times x - 20 - 4x^{2} + 4x\\&= 20x - 20 - 4x^{2} + 4x\\&= - 4x^{2} + 20x + 4x - 20\\&= - 4x^{2} + (20 + 4) \times x - 20\\&= - 4x^{2} + 24x - 20
\end{align*}
C'est un polynôme de degré 2 avec $a = - 4$, $b = 24$ et $c = - 20$.
\item
\begin{align*}
k(x) &= - 6(x + 4)(x + 8)\\&= (- 6x - 6 \times 4)(x + 8)\\&= (- 6x - 24)(x + 8)\\&= - 6x \times x - 6x \times 8 - 24x - 24 \times 8\\&= 8(- 6) \times x - 192 - 6x^{2} - 24x\\&= - 48x - 192 - 6x^{2} - 24x\\&= - 6x^{2} - 48x - 24x - 192\\&= - 6x^{2} + (- 48 - 24) \times x - 192\\&= - 6x^{2} - 72x - 192
\end{align*}
C'est un polynôme de degré 2 avec $a = - 6$, $b = - 72$ et $c = - 192$.
\end{enumerate}
\end{solution}
\begin{exercise}[subtitle={Étude de fonctions}]
Soit $f(x) = 9x^{2} - 18x - 720$ une fonction définie sur $\R$.
\begin{enumerate}
\item Calculer les valeurs suivantes
\[
f(1) \qquad f(-2)
\]
\item Dériver la fonction $f$
\item Étudier le signe de $f'$ puis en déduire les variations de $f$.
\item Est-ce que $f$ admet un maximum? un minimum? Calculer sa valeur.
\end{enumerate}
\end{exercise}
\begin{solution}
\begin{enumerate}
\item On remplace $x$ par les valeurs demandées
\[
f(1) = 9 \times 1^{2} - 18 \times 1 - 720=9 \times 1 - 18 - 720=9 - 738=- 729
\]
\[
f(-1) = 9 \times - 1^{2} - 18(- 1) - 720=9 \times 1 + 18 - 720=9 - 702=- 693
\]
\item Dérivation
\[
f'(x) = 18x - 18
\]
\item Pas de solutions automatiques.
\item Pas de solutions automatiques.
\end{enumerate}
\end{solution}
\begin{exercise}[subtitle={Enclos}]
Dans son garage, Jean a trouvé 33m de grillage. \\
Il décide de l'utiliser pour faire un enclos rectangulaire. Afin d'obtenir un enclos encore plus grand, il veut utiliser le mur du jardin qui formera un côté. Le grillage formera les 3 autres.
\begin{center}
\includegraphics[scale=0.8]{./fig/enclos}
\end{center}
Comment placer les poteaux pour avoir un enclos le plus grand possible ?
\textit{Cet exercice est un exercice de recherche. Vous êtes invité à utiliser les outils que vous voulez. Une part importante de la note sera dédiée à la rédaction et aux explications de ce que vous faites. Vous êtes encouragé à faire des schémas.}
\textit{Si vous utilisez le tableur, vous devrez le mentionner et m'envoyer le tableur à l'adresse \url{benjamin.bertrand@ac-lyon.fr}}
\end{exercise}
\begin{solution}
Formule de l'aire (en notant $x$ la longueur du côté de l'enclos à côté du mur)
\[
A(x) = x(33 - 2x) = - 2x^{2} + 33x
\]
On va donc étudier les variations de la fonction $A(x) = - 2x^{2} + 33x$
\begin{itemize}
\item Fonction dérivée : $A'(x) = - 4x + 33$
\item On résout l'inéquation $A'(x) \geq 0$ pour déterminer quand la fonction $A'$ est positive.
\begin{align*}
A(x) & \geq 0 \\
- 4x + 33 & \geq 0 \\
- 4x + 33 + - 33 &\geq 0 + - 33 \\
- 4x &\geq - 33 \\
\frac{- 4x}{- 4} &\leq \frac{- 33}{- 4} \\
x &\leq \dfrac{33}{4} \\
\end{align*}
Donc $A(x)$ est positif quand $x$ est plus \textbf{petit} que $\dfrac{33}{4}$
\item
\begin{center}
\begin{tikzpicture}
\tkzTabInit[lgt=3,espcl=4]{$x$/1,Signe de $f'(x)$/2, Variations de $f(x)$/2}{, $\dfrac{33}{4}$ ,}%
\tkzTabLine{, +, z, -, }
\tkzTabVar{-/ ,+/$f(\dfrac{33}{4}) = \dfrac{2178}{16}$ , -/}%
\end{tikzpicture}
\end{center}
\end{itemize}
Pour avoir le plus grand enclos, il faut placer les poteaux à 6m du mur et on aura alors un enclos de $72m^2$.
\end{solution}
\end{document}
%%% Local Variables:
%%% mode: latex
%%% TeX-master: "master"
%%% End:

Binary file not shown.

View File

@ -0,0 +1,158 @@
\documentclass[a4paper,12pt]{article}
\usepackage{myXsim}
\usepackage{pgfplots}
\usetikzlibrary{decorations.markings}
\pgfplotsset{compat=1.18}
\title{ DM1 \hfill THERET Olympe}
\tribe{1ST}
\date{A rendre pour le lundi 27 mars 2023}
\duree{}
\xsimsetup{
solution/print = false
}
\pagestyle{empty}
\begin{document}
\maketitle
Le barème est donné à titre indicatif, il pourra être modifié.
\begin{exercise}[subtitle={Polynôme de degré 2}]
Développer les expressions suivantes pour vérifier que ce sont des polynômes de degré 2. Vous préciserez les valeurs de $a$, $b$ et $c$.
\begin{multicols}{2}
\begin{enumerate}
\item $f(x) = (- 6x + 8)(7x + 8)$
\item $g(x) = (- 8x + 5)^{2}$
\item $h(x) = 4 + x(2x - 3)$
\item $i(x) = - 5x^{2} + x(8x + 5)$
\item $j(x) = - 7(x + 10)(x - 4)$
\item $k(x) = 3(x + 8)(x + 8)$
\end{enumerate}
\end{multicols}
\end{exercise}
\begin{solution}
\begin{enumerate}
\item
\begin{align*}
f(x) &= (- 6x + 8)(7x + 8)\\&= - 6x \times 7x - 6x \times 8 + 8 \times 7x + 8 \times 8\\&= - 6 \times 7 \times x^{1 + 1} + 8(- 6) \times x + 8 \times 7 \times x + 64\\&= - 48x + 56x - 42x^{2} + 64\\&= (- 48 + 56) \times x - 42x^{2} + 64\\&= - 42x^{2} + 8x + 64
\end{align*}
C'est un polynôme de degré 2 avec $a = - 42$, $b = 8$ et $c = 64$.
\item
\begin{align*}
g(x) &= (- 8x + 5)^{2}\\&= (- 8x + 5)(- 8x + 5)\\&= - 8x \times - 8x - 8x \times 5 + 5 \times - 8x + 5 \times 5\\&= - 8(- 8) \times x^{1 + 1} + 5(- 8) \times x + 5(- 8) \times x + 25\\&= - 40x - 40x + 64x^{2} + 25\\&= (- 40 - 40) \times x + 64x^{2} + 25\\&= 64x^{2} - 80x + 25
\end{align*}
C'est un polynôme de degré 2 avec $a = 64$, $b = - 80$ et $c = 25$.
\item
\begin{align*}
h(x) &= 4 + x(2x - 3)\\&= 4 + x \times 2x + x(- 3)\\&= 2x^{2} - 3x + 4
\end{align*}
C'est un polynôme de degré 2 avec $a = 2$, $b = - 3$ et $c = 4$.
\item
\begin{align*}
i(x) &= - 5x^{2} + x(8x + 5)\\&= - 5x^{2} + x \times 8x + x \times 5\\&= - 5x^{2} + 8x^{2} + 5x\\&= - 5x^{2} + 8x^{2} + 5x\\&= (- 5 + 8) \times x^{2} + 5x\\&= 3x^{2} + 5x
\end{align*}
C'est un polynôme de degré 2 avec $a = 3$, $b = 5$ et $c = 0$.
\item
\begin{align*}
j(x) &= - 7(x + 10)(x - 4)\\&= (- 7x - 7 \times 10)(x - 4)\\&= (- 7x - 70)(x - 4)\\&= - 7x \times x - 7x(- 4) - 70x - 70(- 4)\\&= - 4(- 7) \times x + 280 - 7x^{2} - 70x\\&= 28x + 280 - 7x^{2} - 70x\\&= - 7x^{2} + 28x - 70x + 280\\&= - 7x^{2} + (28 - 70) \times x + 280\\&= - 7x^{2} - 42x + 280
\end{align*}
C'est un polynôme de degré 2 avec $a = - 7$, $b = - 42$ et $c = 280$.
\item
\begin{align*}
k(x) &= 3(x + 8)(x + 8)\\&= (3x + 3 \times 8)(x + 8)\\&= (3x + 24)(x + 8)\\&= 3x \times x + 3x \times 8 + 24x + 24 \times 8\\&= 8 \times 3 \times x + 192 + 3x^{2} + 24x\\&= 24x + 192 + 3x^{2} + 24x\\&= 3x^{2} + 24x + 24x + 192\\&= 3x^{2} + (24 + 24) \times x + 192\\&= 3x^{2} + 48x + 192
\end{align*}
C'est un polynôme de degré 2 avec $a = 3$, $b = 48$ et $c = 192$.
\end{enumerate}
\end{solution}
\begin{exercise}[subtitle={Étude de fonctions}]
Soit $f(x) = 5x^{2} - 70x + 240$ une fonction définie sur $\R$.
\begin{enumerate}
\item Calculer les valeurs suivantes
\[
f(1) \qquad f(-2)
\]
\item Dériver la fonction $f$
\item Étudier le signe de $f'$ puis en déduire les variations de $f$.
\item Est-ce que $f$ admet un maximum? un minimum? Calculer sa valeur.
\end{enumerate}
\end{exercise}
\begin{solution}
\begin{enumerate}
\item On remplace $x$ par les valeurs demandées
\[
f(1) = 5 \times 1^{2} - 70 \times 1 + 240=5 \times 1 - 70 + 240=5 + 170=175
\]
\[
f(-1) = 5 \times - 1^{2} - 70(- 1) + 240=5 \times 1 + 70 + 240=5 + 310=315
\]
\item Dérivation
\[
f'(x) = 10x - 70
\]
\item Pas de solutions automatiques.
\item Pas de solutions automatiques.
\end{enumerate}
\end{solution}
\begin{exercise}[subtitle={Enclos}]
Dans son garage, Jean a trouvé 29m de grillage. \\
Il décide de l'utiliser pour faire un enclos rectangulaire. Afin d'obtenir un enclos encore plus grand, il veut utiliser le mur du jardin qui formera un côté. Le grillage formera les 3 autres.
\begin{center}
\includegraphics[scale=0.8]{./fig/enclos}
\end{center}
Comment placer les poteaux pour avoir un enclos le plus grand possible ?
\textit{Cet exercice est un exercice de recherche. Vous êtes invité à utiliser les outils que vous voulez. Une part importante de la note sera dédiée à la rédaction et aux explications de ce que vous faites. Vous êtes encouragé à faire des schémas.}
\textit{Si vous utilisez le tableur, vous devrez le mentionner et m'envoyer le tableur à l'adresse \url{benjamin.bertrand@ac-lyon.fr}}
\end{exercise}
\begin{solution}
Formule de l'aire (en notant $x$ la longueur du côté de l'enclos à côté du mur)
\[
A(x) = x(29 - 2x) = - 2x^{2} + 29x
\]
On va donc étudier les variations de la fonction $A(x) = - 2x^{2} + 29x$
\begin{itemize}
\item Fonction dérivée : $A'(x) = - 4x + 29$
\item On résout l'inéquation $A'(x) \geq 0$ pour déterminer quand la fonction $A'$ est positive.
\begin{align*}
A(x) & \geq 0 \\
- 4x + 29 & \geq 0 \\
- 4x + 29 + - 29 &\geq 0 + - 29 \\
- 4x &\geq - 29 \\
\frac{- 4x}{- 4} &\leq \frac{- 29}{- 4} \\
x &\leq \dfrac{29}{4} \\
\end{align*}
Donc $A(x)$ est positif quand $x$ est plus \textbf{petit} que $\dfrac{29}{4}$
\item
\begin{center}
\begin{tikzpicture}
\tkzTabInit[lgt=3,espcl=4]{$x$/1,Signe de $f'(x)$/2, Variations de $f(x)$/2}{, $\dfrac{29}{4}$ ,}%
\tkzTabLine{, +, z, -, }
\tkzTabVar{-/ ,+/$f(\dfrac{29}{4}) = \dfrac{1682}{16}$ , -/}%
\end{tikzpicture}
\end{center}
\end{itemize}
Pour avoir le plus grand enclos, il faut placer les poteaux à 6m du mur et on aura alors un enclos de $72m^2$.
\end{solution}
\end{document}
%%% Local Variables:
%%% mode: latex
%%% TeX-master: "master"
%%% End:

Binary file not shown.

View File

@ -0,0 +1,158 @@
\documentclass[a4paper,12pt]{article}
\usepackage{myXsim}
\usepackage{pgfplots}
\usetikzlibrary{decorations.markings}
\pgfplotsset{compat=1.18}
\title{ DM1 \hfill }
\tribe{1ST}
\date{A rendre pour le lundi 27 mars 2023}
\duree{}
\xsimsetup{
solution/print = false
}
\pagestyle{empty}
\begin{document}
\maketitle
Le barème est donné à titre indicatif, il pourra être modifié.
\begin{exercise}[subtitle={Polynôme de degré 2}]
Développer les expressions suivantes pour vérifier que ce sont des polynômes de degré 2. Vous préciserez les valeurs de $a$, $b$ et $c$.
\begin{multicols}{2}
\begin{enumerate}
\item $f(x) = (3x - 6)(- 7x - 6)$
\item $g(x) = (- 3x + 10)^{2}$
\item $h(x) = - 10 + x(- 7x - 4)$
\item $i(x) = - 1x^{2} + x(- 8x - 4)$
\item $j(x) = 6(x - 3)(x + 8)$
\item $k(x) = 5(x + 6)(x - 10)$
\end{enumerate}
\end{multicols}
\end{exercise}
\begin{solution}
\begin{enumerate}
\item
\begin{align*}
f(x) &= (3x - 6)(- 7x - 6)\\&= 3x \times - 7x + 3x(- 6) - 6 \times - 7x - 6(- 6)\\&= 3(- 7) \times x^{1 + 1} - 6 \times 3 \times x - 6(- 7) \times x + 36\\&= - 18x + 42x - 21x^{2} + 36\\&= (- 18 + 42) \times x - 21x^{2} + 36\\&= - 21x^{2} + 24x + 36
\end{align*}
C'est un polynôme de degré 2 avec $a = - 21$, $b = 24$ et $c = 36$.
\item
\begin{align*}
g(x) &= (- 3x + 10)^{2}\\&= (- 3x + 10)(- 3x + 10)\\&= - 3x \times - 3x - 3x \times 10 + 10 \times - 3x + 10 \times 10\\&= - 3(- 3) \times x^{1 + 1} + 10(- 3) \times x + 10(- 3) \times x + 100\\&= - 30x - 30x + 9x^{2} + 100\\&= (- 30 - 30) \times x + 9x^{2} + 100\\&= 9x^{2} - 60x + 100
\end{align*}
C'est un polynôme de degré 2 avec $a = 9$, $b = - 60$ et $c = 100$.
\item
\begin{align*}
h(x) &= - 10 + x(- 7x - 4)\\&= - 10 + x \times - 7x + x(- 4)\\&= - 7x^{2} - 4x - 10
\end{align*}
C'est un polynôme de degré 2 avec $a = - 7$, $b = - 4$ et $c = - 10$.
\item
\begin{align*}
i(x) &= - 1x^{2} + x(- 8x - 4)\\&= - x^{2} + x \times - 8x + x(- 4)\\&= - x^{2} - 8x^{2} - 4x\\&= - x^{2} - 8x^{2} - 4x\\&= (- 1 - 8) \times x^{2} - 4x\\&= - 9x^{2} - 4x
\end{align*}
C'est un polynôme de degré 2 avec $a = - 9$, $b = - 4$ et $c = 0$.
\item
\begin{align*}
j(x) &= 6(x - 3)(x + 8)\\&= (6x + 6(- 3))(x + 8)\\&= (6x - 18)(x + 8)\\&= 6x \times x + 6x \times 8 - 18x - 18 \times 8\\&= 8 \times 6 \times x - 144 + 6x^{2} - 18x\\&= 48x - 144 + 6x^{2} - 18x\\&= 6x^{2} + 48x - 18x - 144\\&= 6x^{2} + (48 - 18) \times x - 144\\&= 6x^{2} + 30x - 144
\end{align*}
C'est un polynôme de degré 2 avec $a = 6$, $b = 30$ et $c = - 144$.
\item
\begin{align*}
k(x) &= 5(x + 6)(x - 10)\\&= (5x + 5 \times 6)(x - 10)\\&= (5x + 30)(x - 10)\\&= 5x \times x + 5x(- 10) + 30x + 30(- 10)\\&= - 10 \times 5 \times x - 300 + 5x^{2} + 30x\\&= - 50x - 300 + 5x^{2} + 30x\\&= 5x^{2} - 50x + 30x - 300\\&= 5x^{2} + (- 50 + 30) \times x - 300\\&= 5x^{2} - 20x - 300
\end{align*}
C'est un polynôme de degré 2 avec $a = 5$, $b = - 20$ et $c = - 300$.
\end{enumerate}
\end{solution}
\begin{exercise}[subtitle={Étude de fonctions}]
Soit $f(x) = 6x^{2} - 48x - 120$ une fonction définie sur $\R$.
\begin{enumerate}
\item Calculer les valeurs suivantes
\[
f(1) \qquad f(-2)
\]
\item Dériver la fonction $f$
\item Étudier le signe de $f'$ puis en déduire les variations de $f$.
\item Est-ce que $f$ admet un maximum? un minimum? Calculer sa valeur.
\end{enumerate}
\end{exercise}
\begin{solution}
\begin{enumerate}
\item On remplace $x$ par les valeurs demandées
\[
f(1) = 6 \times 1^{2} - 48 \times 1 - 120=6 \times 1 - 48 - 120=6 - 168=- 162
\]
\[
f(-1) = 6 \times - 1^{2} - 48(- 1) - 120=6 \times 1 + 48 - 120=6 - 72=- 66
\]
\item Dérivation
\[
f'(x) = 12x - 48
\]
\item Pas de solutions automatiques.
\item Pas de solutions automatiques.
\end{enumerate}
\end{solution}
\begin{exercise}[subtitle={Enclos}]
Dans son garage, Jean a trouvé 24m de grillage. \\
Il décide de l'utiliser pour faire un enclos rectangulaire. Afin d'obtenir un enclos encore plus grand, il veut utiliser le mur du jardin qui formera un côté. Le grillage formera les 3 autres.
\begin{center}
\includegraphics[scale=0.8]{./fig/enclos}
\end{center}
Comment placer les poteaux pour avoir un enclos le plus grand possible ?
\textit{Cet exercice est un exercice de recherche. Vous êtes invité à utiliser les outils que vous voulez. Une part importante de la note sera dédiée à la rédaction et aux explications de ce que vous faites. Vous êtes encouragé à faire des schémas.}
\textit{Si vous utilisez le tableur, vous devrez le mentionner et m'envoyer le tableur à l'adresse \url{benjamin.bertrand@ac-lyon.fr}}
\end{exercise}
\begin{solution}
Formule de l'aire (en notant $x$ la longueur du côté de l'enclos à côté du mur)
\[
A(x) = x(24 - 2x) = - 2x^{2} + 24x
\]
On va donc étudier les variations de la fonction $A(x) = - 2x^{2} + 24x$
\begin{itemize}
\item Fonction dérivée : $A'(x) = - 4x + 24$
\item On résout l'inéquation $A'(x) \geq 0$ pour déterminer quand la fonction $A'$ est positive.
\begin{align*}
A(x) & \geq 0 \\
- 4x + 24 & \geq 0 \\
- 4x + 24 + - 24 &\geq 0 + - 24 \\
- 4x &\geq - 24 \\
\frac{- 4x}{- 4} &\leq \frac{- 24}{- 4} \\
x &\leq 6 \\
\end{align*}
Donc $A(x)$ est positif quand $x$ est plus \textbf{petit} que $6$
\item
\begin{center}
\begin{tikzpicture}
\tkzTabInit[lgt=3,espcl=4]{$x$/1,Signe de $f'(x)$/2, Variations de $f(x)$/2}{, $6$ ,}%
\tkzTabLine{, +, z, -, }
\tkzTabVar{-/ ,+/$f(6) = 72$ , -/}%
\end{tikzpicture}
\end{center}
\end{itemize}
Pour avoir le plus grand enclos, il faut placer les poteaux à 6m du mur et on aura alors un enclos de $72m^2$.
\end{solution}
\end{document}
%%% Local Variables:
%%% mode: latex
%%% TeX-master: "master"
%%% End:

Binary file not shown.

View File

@ -0,0 +1,158 @@
\documentclass[a4paper,12pt]{article}
\usepackage{myXsim}
\usepackage{pgfplots}
\usetikzlibrary{decorations.markings}
\pgfplotsset{compat=1.18}
\title{ DM1 \hfill TODESCHINI Alissa}
\tribe{1ST}
\date{A rendre pour le lundi 27 mars 2023}
\duree{}
\xsimsetup{
solution/print = false
}
\pagestyle{empty}
\begin{document}
\maketitle
Le barème est donné à titre indicatif, il pourra être modifié.
\begin{exercise}[subtitle={Polynôme de degré 2}]
Développer les expressions suivantes pour vérifier que ce sont des polynômes de degré 2. Vous préciserez les valeurs de $a$, $b$ et $c$.
\begin{multicols}{2}
\begin{enumerate}
\item $f(x) = (- 8x - 2)(- 7x - 2)$
\item $g(x) = (6x + 5)^{2}$
\item $h(x) = - 4 + x(6x - 1)$
\item $i(x) = - 7x^{2} + x(- 6x - 3)$
\item $j(x) = - 9(x - 4)(x - 4)$
\item $k(x) = 8(x - 2)(x - 4)$
\end{enumerate}
\end{multicols}
\end{exercise}
\begin{solution}
\begin{enumerate}
\item
\begin{align*}
f(x) &= (- 8x - 2)(- 7x - 2)\\&= - 8x \times - 7x - 8x(- 2) - 2 \times - 7x - 2(- 2)\\&= - 8(- 7) \times x^{1 + 1} - 2(- 8) \times x - 2(- 7) \times x + 4\\&= 16x + 14x + 56x^{2} + 4\\&= (16 + 14) \times x + 56x^{2} + 4\\&= 56x^{2} + 30x + 4
\end{align*}
C'est un polynôme de degré 2 avec $a = 56$, $b = 30$ et $c = 4$.
\item
\begin{align*}
g(x) &= (6x + 5)^{2}\\&= (6x + 5)(6x + 5)\\&= 6x \times 6x + 6x \times 5 + 5 \times 6x + 5 \times 5\\&= 6 \times 6 \times x^{1 + 1} + 5 \times 6 \times x + 5 \times 6 \times x + 25\\&= 30x + 30x + 36x^{2} + 25\\&= (30 + 30) \times x + 36x^{2} + 25\\&= 36x^{2} + 60x + 25
\end{align*}
C'est un polynôme de degré 2 avec $a = 36$, $b = 60$ et $c = 25$.
\item
\begin{align*}
h(x) &= - 4 + x(6x - 1)\\&= - 4 + x \times 6x + x(- 1)\\&= 6x^{2} - x - 4
\end{align*}
C'est un polynôme de degré 2 avec $a = 6$, $b = - 1$ et $c = - 4$.
\item
\begin{align*}
i(x) &= - 7x^{2} + x(- 6x - 3)\\&= - 7x^{2} + x \times - 6x + x(- 3)\\&= - 7x^{2} - 6x^{2} - 3x\\&= - 7x^{2} - 6x^{2} - 3x\\&= (- 7 - 6) \times x^{2} - 3x\\&= - 13x^{2} - 3x
\end{align*}
C'est un polynôme de degré 2 avec $a = - 13$, $b = - 3$ et $c = 0$.
\item
\begin{align*}
j(x) &= - 9(x - 4)(x - 4)\\&= (- 9x - 9(- 4))(x - 4)\\&= (- 9x + 36)(x - 4)\\&= - 9x \times x - 9x(- 4) + 36x + 36(- 4)\\&= - 4(- 9) \times x - 144 - 9x^{2} + 36x\\&= 36x - 144 - 9x^{2} + 36x\\&= - 9x^{2} + 36x + 36x - 144\\&= - 9x^{2} + (36 + 36) \times x - 144\\&= - 9x^{2} + 72x - 144
\end{align*}
C'est un polynôme de degré 2 avec $a = - 9$, $b = 72$ et $c = - 144$.
\item
\begin{align*}
k(x) &= 8(x - 2)(x - 4)\\&= (8x + 8(- 2))(x - 4)\\&= (8x - 16)(x - 4)\\&= 8x \times x + 8x(- 4) - 16x - 16(- 4)\\&= - 4 \times 8 \times x + 64 + 8x^{2} - 16x\\&= - 32x + 64 + 8x^{2} - 16x\\&= 8x^{2} - 32x - 16x + 64\\&= 8x^{2} + (- 32 - 16) \times x + 64\\&= 8x^{2} - 48x + 64
\end{align*}
C'est un polynôme de degré 2 avec $a = 8$, $b = - 48$ et $c = 64$.
\end{enumerate}
\end{solution}
\begin{exercise}[subtitle={Étude de fonctions}]
Soit $f(x) = 2x^{2} - 162$ une fonction définie sur $\R$.
\begin{enumerate}
\item Calculer les valeurs suivantes
\[
f(1) \qquad f(-2)
\]
\item Dériver la fonction $f$
\item Étudier le signe de $f'$ puis en déduire les variations de $f$.
\item Est-ce que $f$ admet un maximum? un minimum? Calculer sa valeur.
\end{enumerate}
\end{exercise}
\begin{solution}
\begin{enumerate}
\item On remplace $x$ par les valeurs demandées
\[
f(1) = 2 \times 1^{2} - 162=2 \times 1 - 162=2 - 162=- 160
\]
\[
f(-1) = 2 \times - 1^{2} - 162=2 \times 1 - 162=2 - 162=- 160
\]
\item Dérivation
\[
f'(x) = 4x
\]
\item Pas de solutions automatiques.
\item Pas de solutions automatiques.
\end{enumerate}
\end{solution}
\begin{exercise}[subtitle={Enclos}]
Dans son garage, Jean a trouvé 37m de grillage. \\
Il décide de l'utiliser pour faire un enclos rectangulaire. Afin d'obtenir un enclos encore plus grand, il veut utiliser le mur du jardin qui formera un côté. Le grillage formera les 3 autres.
\begin{center}
\includegraphics[scale=0.8]{./fig/enclos}
\end{center}
Comment placer les poteaux pour avoir un enclos le plus grand possible ?
\textit{Cet exercice est un exercice de recherche. Vous êtes invité à utiliser les outils que vous voulez. Une part importante de la note sera dédiée à la rédaction et aux explications de ce que vous faites. Vous êtes encouragé à faire des schémas.}
\textit{Si vous utilisez le tableur, vous devrez le mentionner et m'envoyer le tableur à l'adresse \url{benjamin.bertrand@ac-lyon.fr}}
\end{exercise}
\begin{solution}
Formule de l'aire (en notant $x$ la longueur du côté de l'enclos à côté du mur)
\[
A(x) = x(37 - 2x) = - 2x^{2} + 37x
\]
On va donc étudier les variations de la fonction $A(x) = - 2x^{2} + 37x$
\begin{itemize}
\item Fonction dérivée : $A'(x) = - 4x + 37$
\item On résout l'inéquation $A'(x) \geq 0$ pour déterminer quand la fonction $A'$ est positive.
\begin{align*}
A(x) & \geq 0 \\
- 4x + 37 & \geq 0 \\
- 4x + 37 + - 37 &\geq 0 + - 37 \\
- 4x &\geq - 37 \\
\frac{- 4x}{- 4} &\leq \frac{- 37}{- 4} \\
x &\leq \dfrac{37}{4} \\
\end{align*}
Donc $A(x)$ est positif quand $x$ est plus \textbf{petit} que $\dfrac{37}{4}$
\item
\begin{center}
\begin{tikzpicture}
\tkzTabInit[lgt=3,espcl=4]{$x$/1,Signe de $f'(x)$/2, Variations de $f(x)$/2}{, $\dfrac{37}{4}$ ,}%
\tkzTabLine{, +, z, -, }
\tkzTabVar{-/ ,+/$f(\dfrac{37}{4}) = \dfrac{2738}{16}$ , -/}%
\end{tikzpicture}
\end{center}
\end{itemize}
Pour avoir le plus grand enclos, il faut placer les poteaux à 6m du mur et on aura alors un enclos de $72m^2$.
\end{solution}
\end{document}
%%% Local Variables:
%%% mode: latex
%%% TeX-master: "master"
%%% End:

Binary file not shown.

View File

@ -0,0 +1,158 @@
\documentclass[a4paper,12pt]{article}
\usepackage{myXsim}
\usepackage{pgfplots}
\usetikzlibrary{decorations.markings}
\pgfplotsset{compat=1.18}
\title{ DM1 \hfill VAN ES Tristan}
\tribe{1ST}
\date{A rendre pour le lundi 27 mars 2023}
\duree{}
\xsimsetup{
solution/print = false
}
\pagestyle{empty}
\begin{document}
\maketitle
Le barème est donné à titre indicatif, il pourra être modifié.
\begin{exercise}[subtitle={Polynôme de degré 2}]
Développer les expressions suivantes pour vérifier que ce sont des polynômes de degré 2. Vous préciserez les valeurs de $a$, $b$ et $c$.
\begin{multicols}{2}
\begin{enumerate}
\item $f(x) = (- 9x + 3)(- 9x + 3)$
\item $g(x) = (- 3x + 6)^{2}$
\item $h(x) = 10 + x(6x - 7)$
\item $i(x) = - 1x^{2} + x(- 10x - 4)$
\item $j(x) = 4(x - 3)(x + 4)$
\item $k(x) = 4(x + 9)(x + 3)$
\end{enumerate}
\end{multicols}
\end{exercise}
\begin{solution}
\begin{enumerate}
\item
\begin{align*}
f(x) &= (- 9x + 3)(- 9x + 3)\\&= - 9x \times - 9x - 9x \times 3 + 3 \times - 9x + 3 \times 3\\&= - 9(- 9) \times x^{1 + 1} + 3(- 9) \times x + 3(- 9) \times x + 9\\&= - 27x - 27x + 81x^{2} + 9\\&= (- 27 - 27) \times x + 81x^{2} + 9\\&= 81x^{2} - 54x + 9
\end{align*}
C'est un polynôme de degré 2 avec $a = 81$, $b = - 54$ et $c = 9$.
\item
\begin{align*}
g(x) &= (- 3x + 6)^{2}\\&= (- 3x + 6)(- 3x + 6)\\&= - 3x \times - 3x - 3x \times 6 + 6 \times - 3x + 6 \times 6\\&= - 3(- 3) \times x^{1 + 1} + 6(- 3) \times x + 6(- 3) \times x + 36\\&= - 18x - 18x + 9x^{2} + 36\\&= (- 18 - 18) \times x + 9x^{2} + 36\\&= 9x^{2} - 36x + 36
\end{align*}
C'est un polynôme de degré 2 avec $a = 9$, $b = - 36$ et $c = 36$.
\item
\begin{align*}
h(x) &= 10 + x(6x - 7)\\&= 10 + x \times 6x + x(- 7)\\&= 6x^{2} - 7x + 10
\end{align*}
C'est un polynôme de degré 2 avec $a = 6$, $b = - 7$ et $c = 10$.
\item
\begin{align*}
i(x) &= - 1x^{2} + x(- 10x - 4)\\&= - x^{2} + x \times - 10x + x(- 4)\\&= - x^{2} - 10x^{2} - 4x\\&= - x^{2} - 10x^{2} - 4x\\&= (- 1 - 10) \times x^{2} - 4x\\&= - 11x^{2} - 4x
\end{align*}
C'est un polynôme de degré 2 avec $a = - 11$, $b = - 4$ et $c = 0$.
\item
\begin{align*}
j(x) &= 4(x - 3)(x + 4)\\&= (4x + 4(- 3))(x + 4)\\&= (4x - 12)(x + 4)\\&= 4x \times x + 4x \times 4 - 12x - 12 \times 4\\&= 4 \times 4 \times x - 48 + 4x^{2} - 12x\\&= 16x - 48 + 4x^{2} - 12x\\&= 4x^{2} + 16x - 12x - 48\\&= 4x^{2} + (16 - 12) \times x - 48\\&= 4x^{2} + 4x - 48
\end{align*}
C'est un polynôme de degré 2 avec $a = 4$, $b = 4$ et $c = - 48$.
\item
\begin{align*}
k(x) &= 4(x + 9)(x + 3)\\&= (4x + 4 \times 9)(x + 3)\\&= (4x + 36)(x + 3)\\&= 4x \times x + 4x \times 3 + 36x + 36 \times 3\\&= 3 \times 4 \times x + 108 + 4x^{2} + 36x\\&= 12x + 108 + 4x^{2} + 36x\\&= 4x^{2} + 12x + 36x + 108\\&= 4x^{2} + (12 + 36) \times x + 108\\&= 4x^{2} + 48x + 108
\end{align*}
C'est un polynôme de degré 2 avec $a = 4$, $b = 48$ et $c = 108$.
\end{enumerate}
\end{solution}
\begin{exercise}[subtitle={Étude de fonctions}]
Soit $f(x) = 3x^{2} + 12x - 180$ une fonction définie sur $\R$.
\begin{enumerate}
\item Calculer les valeurs suivantes
\[
f(1) \qquad f(-2)
\]
\item Dériver la fonction $f$
\item Étudier le signe de $f'$ puis en déduire les variations de $f$.
\item Est-ce que $f$ admet un maximum? un minimum? Calculer sa valeur.
\end{enumerate}
\end{exercise}
\begin{solution}
\begin{enumerate}
\item On remplace $x$ par les valeurs demandées
\[
f(1) = 3 \times 1^{2} + 12 \times 1 - 180=3 \times 1 + 12 - 180=3 - 168=- 165
\]
\[
f(-1) = 3 \times - 1^{2} + 12(- 1) - 180=3 \times 1 - 12 - 180=3 - 192=- 189
\]
\item Dérivation
\[
f'(x) = 6x + 12
\]
\item Pas de solutions automatiques.
\item Pas de solutions automatiques.
\end{enumerate}
\end{solution}
\begin{exercise}[subtitle={Enclos}]
Dans son garage, Jean a trouvé 15m de grillage. \\
Il décide de l'utiliser pour faire un enclos rectangulaire. Afin d'obtenir un enclos encore plus grand, il veut utiliser le mur du jardin qui formera un côté. Le grillage formera les 3 autres.
\begin{center}
\includegraphics[scale=0.8]{./fig/enclos}
\end{center}
Comment placer les poteaux pour avoir un enclos le plus grand possible ?
\textit{Cet exercice est un exercice de recherche. Vous êtes invité à utiliser les outils que vous voulez. Une part importante de la note sera dédiée à la rédaction et aux explications de ce que vous faites. Vous êtes encouragé à faire des schémas.}
\textit{Si vous utilisez le tableur, vous devrez le mentionner et m'envoyer le tableur à l'adresse \url{benjamin.bertrand@ac-lyon.fr}}
\end{exercise}
\begin{solution}
Formule de l'aire (en notant $x$ la longueur du côté de l'enclos à côté du mur)
\[
A(x) = x(15 - 2x) = - 2x^{2} + 15x
\]
On va donc étudier les variations de la fonction $A(x) = - 2x^{2} + 15x$
\begin{itemize}
\item Fonction dérivée : $A'(x) = - 4x + 15$
\item On résout l'inéquation $A'(x) \geq 0$ pour déterminer quand la fonction $A'$ est positive.
\begin{align*}
A(x) & \geq 0 \\
- 4x + 15 & \geq 0 \\
- 4x + 15 + - 15 &\geq 0 + - 15 \\
- 4x &\geq - 15 \\
\frac{- 4x}{- 4} &\leq \frac{- 15}{- 4} \\
x &\leq \dfrac{15}{4} \\
\end{align*}
Donc $A(x)$ est positif quand $x$ est plus \textbf{petit} que $\dfrac{15}{4}$
\item
\begin{center}
\begin{tikzpicture}
\tkzTabInit[lgt=3,espcl=4]{$x$/1,Signe de $f'(x)$/2, Variations de $f(x)$/2}{, $\dfrac{15}{4}$ ,}%
\tkzTabLine{, +, z, -, }
\tkzTabVar{-/ ,+/$f(\dfrac{15}{4}) = \dfrac{450}{16}$ , -/}%
\end{tikzpicture}
\end{center}
\end{itemize}
Pour avoir le plus grand enclos, il faut placer les poteaux à 6m du mur et on aura alors un enclos de $72m^2$.
\end{solution}
\end{document}
%%% Local Variables:
%%% mode: latex
%%% TeX-master: "master"
%%% End:

Binary file not shown.

View File

@ -0,0 +1,158 @@
\documentclass[a4paper,12pt]{article}
\usepackage{myXsim}
\usepackage{pgfplots}
\usetikzlibrary{decorations.markings}
\pgfplotsset{compat=1.18}
\title{ DM1 \hfill YANIK Azra}
\tribe{1ST}
\date{A rendre pour le lundi 27 mars 2023}
\duree{}
\xsimsetup{
solution/print = false
}
\pagestyle{empty}
\begin{document}
\maketitle
Le barème est donné à titre indicatif, il pourra être modifié.
\begin{exercise}[subtitle={Polynôme de degré 2}]
Développer les expressions suivantes pour vérifier que ce sont des polynômes de degré 2. Vous préciserez les valeurs de $a$, $b$ et $c$.
\begin{multicols}{2}
\begin{enumerate}
\item $f(x) = (- 9x + 7)(- 5x + 7)$
\item $g(x) = (6x - 5)^{2}$
\item $h(x) = - 8 + x(- 3x - 7)$
\item $i(x) = - 3x^{2} + x(- 3x + 3)$
\item $j(x) = 4(x - 3)(x - 1)$
\item $k(x) = - 6(x + 8)(x + 4)$
\end{enumerate}
\end{multicols}
\end{exercise}
\begin{solution}
\begin{enumerate}
\item
\begin{align*}
f(x) &= (- 9x + 7)(- 5x + 7)\\&= - 9x \times - 5x - 9x \times 7 + 7 \times - 5x + 7 \times 7\\&= - 9(- 5) \times x^{1 + 1} + 7(- 9) \times x + 7(- 5) \times x + 49\\&= - 63x - 35x + 45x^{2} + 49\\&= (- 63 - 35) \times x + 45x^{2} + 49\\&= 45x^{2} - 98x + 49
\end{align*}
C'est un polynôme de degré 2 avec $a = 45$, $b = - 98$ et $c = 49$.
\item
\begin{align*}
g(x) &= (6x - 5)^{2}\\&= (6x - 5)(6x - 5)\\&= 6x \times 6x + 6x(- 5) - 5 \times 6x - 5(- 5)\\&= 6 \times 6 \times x^{1 + 1} - 5 \times 6 \times x - 5 \times 6 \times x + 25\\&= - 30x - 30x + 36x^{2} + 25\\&= (- 30 - 30) \times x + 36x^{2} + 25\\&= 36x^{2} - 60x + 25
\end{align*}
C'est un polynôme de degré 2 avec $a = 36$, $b = - 60$ et $c = 25$.
\item
\begin{align*}
h(x) &= - 8 + x(- 3x - 7)\\&= - 8 + x \times - 3x + x(- 7)\\&= - 3x^{2} - 7x - 8
\end{align*}
C'est un polynôme de degré 2 avec $a = - 3$, $b = - 7$ et $c = - 8$.
\item
\begin{align*}
i(x) &= - 3x^{2} + x(- 3x + 3)\\&= - 3x^{2} + x \times - 3x + x \times 3\\&= - 3x^{2} - 3x^{2} + 3x\\&= - 3x^{2} - 3x^{2} + 3x\\&= (- 3 - 3) \times x^{2} + 3x\\&= - 6x^{2} + 3x
\end{align*}
C'est un polynôme de degré 2 avec $a = - 6$, $b = 3$ et $c = 0$.
\item
\begin{align*}
j(x) &= 4(x - 3)(x - 1)\\&= (4x + 4(- 3))(x - 1)\\&= (4x - 12)(x - 1)\\&= 4x \times x + 4x(- 1) - 12x - 12(- 1)\\&= - 1 \times 4 \times x + 12 + 4x^{2} - 12x\\&= - 4x + 12 + 4x^{2} - 12x\\&= 4x^{2} - 4x - 12x + 12\\&= 4x^{2} + (- 4 - 12) \times x + 12\\&= 4x^{2} - 16x + 12
\end{align*}
C'est un polynôme de degré 2 avec $a = 4$, $b = - 16$ et $c = 12$.
\item
\begin{align*}
k(x) &= - 6(x + 8)(x + 4)\\&= (- 6x - 6 \times 8)(x + 4)\\&= (- 6x - 48)(x + 4)\\&= - 6x \times x - 6x \times 4 - 48x - 48 \times 4\\&= 4(- 6) \times x - 192 - 6x^{2} - 48x\\&= - 24x - 192 - 6x^{2} - 48x\\&= - 6x^{2} - 24x - 48x - 192\\&= - 6x^{2} + (- 24 - 48) \times x - 192\\&= - 6x^{2} - 72x - 192
\end{align*}
C'est un polynôme de degré 2 avec $a = - 6$, $b = - 72$ et $c = - 192$.
\end{enumerate}
\end{solution}
\begin{exercise}[subtitle={Étude de fonctions}]
Soit $f(x) = - 2x^{2} - 32x - 120$ une fonction définie sur $\R$.
\begin{enumerate}
\item Calculer les valeurs suivantes
\[
f(1) \qquad f(-2)
\]
\item Dériver la fonction $f$
\item Étudier le signe de $f'$ puis en déduire les variations de $f$.
\item Est-ce que $f$ admet un maximum? un minimum? Calculer sa valeur.
\end{enumerate}
\end{exercise}
\begin{solution}
\begin{enumerate}
\item On remplace $x$ par les valeurs demandées
\[
f(1) = - 2 \times 1^{2} - 32 \times 1 - 120=- 2 \times 1 - 32 - 120=- 2 - 152=- 154
\]
\[
f(-1) = - 2 \times - 1^{2} - 32(- 1) - 120=- 2 \times 1 + 32 - 120=- 2 - 88=- 90
\]
\item Dérivation
\[
f'(x) = - 4x - 32
\]
\item Pas de solutions automatiques.
\item Pas de solutions automatiques.
\end{enumerate}
\end{solution}
\begin{exercise}[subtitle={Enclos}]
Dans son garage, Jean a trouvé 30m de grillage. \\
Il décide de l'utiliser pour faire un enclos rectangulaire. Afin d'obtenir un enclos encore plus grand, il veut utiliser le mur du jardin qui formera un côté. Le grillage formera les 3 autres.
\begin{center}
\includegraphics[scale=0.8]{./fig/enclos}
\end{center}
Comment placer les poteaux pour avoir un enclos le plus grand possible ?
\textit{Cet exercice est un exercice de recherche. Vous êtes invité à utiliser les outils que vous voulez. Une part importante de la note sera dédiée à la rédaction et aux explications de ce que vous faites. Vous êtes encouragé à faire des schémas.}
\textit{Si vous utilisez le tableur, vous devrez le mentionner et m'envoyer le tableur à l'adresse \url{benjamin.bertrand@ac-lyon.fr}}
\end{exercise}
\begin{solution}
Formule de l'aire (en notant $x$ la longueur du côté de l'enclos à côté du mur)
\[
A(x) = x(30 - 2x) = - 2x^{2} + 30x
\]
On va donc étudier les variations de la fonction $A(x) = - 2x^{2} + 30x$
\begin{itemize}
\item Fonction dérivée : $A'(x) = - 4x + 30$
\item On résout l'inéquation $A'(x) \geq 0$ pour déterminer quand la fonction $A'$ est positive.
\begin{align*}
A(x) & \geq 0 \\
- 4x + 30 & \geq 0 \\
- 4x + 30 + - 30 &\geq 0 + - 30 \\
- 4x &\geq - 30 \\
\frac{- 4x}{- 4} &\leq \frac{- 30}{- 4} \\
x &\leq \dfrac{15}{2} \\
\end{align*}
Donc $A(x)$ est positif quand $x$ est plus \textbf{petit} que $\dfrac{15}{2}$
\item
\begin{center}
\begin{tikzpicture}
\tkzTabInit[lgt=3,espcl=4]{$x$/1,Signe de $f'(x)$/2, Variations de $f(x)$/2}{, $\dfrac{15}{2}$ ,}%
\tkzTabLine{, +, z, -, }
\tkzTabVar{-/ ,+/$f(\dfrac{15}{2}) = \dfrac{450}{4}$ , -/}%
\end{tikzpicture}
\end{center}
\end{itemize}
Pour avoir le plus grand enclos, il faut placer les poteaux à 6m du mur et on aura alors un enclos de $72m^2$.
\end{solution}
\end{document}
%%% Local Variables:
%%% mode: latex
%%% TeX-master: "master"
%%% End:

Binary file not shown.

View File

@ -0,0 +1,158 @@
\documentclass[a4paper,12pt]{article}
\usepackage{myXsim}
\usepackage{pgfplots}
\usetikzlibrary{decorations.markings}
\pgfplotsset{compat=1.18}
\title{ DM1 \hfill ZINBI Myriem}
\tribe{1ST}
\date{A rendre pour le lundi 27 mars 2023}
\duree{}
\xsimsetup{
solution/print = false
}
\pagestyle{empty}
\begin{document}
\maketitle
Le barème est donné à titre indicatif, il pourra être modifié.
\begin{exercise}[subtitle={Polynôme de degré 2}]
Développer les expressions suivantes pour vérifier que ce sont des polynômes de degré 2. Vous préciserez les valeurs de $a$, $b$ et $c$.
\begin{multicols}{2}
\begin{enumerate}
\item $f(x) = (- 1x + 9)(- 7x + 9)$
\item $g(x) = (2x + 2)^{2}$
\item $h(x) = 7 + x(- 10x - 3)$
\item $i(x) = - 1x^{2} + x(10x - 9)$
\item $j(x) = - 4(x + 10)(x + 4)$
\item $k(x) = - 6(x - 7)(x - 10)$
\end{enumerate}
\end{multicols}
\end{exercise}
\begin{solution}
\begin{enumerate}
\item
\begin{align*}
f(x) &= (- 1x + 9)(- 7x + 9)\\&= (- x) \times - 7x + (- x) \times 9 + 9 \times - 7x + 9 \times 9\\&= - 1(- 7) \times x^{1 + 1} + 9(- 1) \times x + 9(- 7) \times x + 81\\&= - 9x - 63x + 7x^{2} + 81\\&= (- 9 - 63) \times x + 7x^{2} + 81\\&= 7x^{2} - 72x + 81
\end{align*}
C'est un polynôme de degré 2 avec $a = 7$, $b = - 72$ et $c = 81$.
\item
\begin{align*}
g(x) &= (2x + 2)^{2}\\&= (2x + 2)(2x + 2)\\&= 2x \times 2x + 2x \times 2 + 2 \times 2x + 2 \times 2\\&= 2 \times 2 \times x^{1 + 1} + 2 \times 2 \times x + 2 \times 2 \times x + 4\\&= 4x + 4x + 4x^{2} + 4\\&= (4 + 4) \times x + 4x^{2} + 4\\&= 4x^{2} + 8x + 4
\end{align*}
C'est un polynôme de degré 2 avec $a = 4$, $b = 8$ et $c = 4$.
\item
\begin{align*}
h(x) &= 7 + x(- 10x - 3)\\&= 7 + x \times - 10x + x(- 3)\\&= - 10x^{2} - 3x + 7
\end{align*}
C'est un polynôme de degré 2 avec $a = - 10$, $b = - 3$ et $c = 7$.
\item
\begin{align*}
i(x) &= - 1x^{2} + x(10x - 9)\\&= - x^{2} + x \times 10x + x(- 9)\\&= - x^{2} + 10x^{2} - 9x\\&= - x^{2} + 10x^{2} - 9x\\&= (- 1 + 10) \times x^{2} - 9x\\&= 9x^{2} - 9x
\end{align*}
C'est un polynôme de degré 2 avec $a = 9$, $b = - 9$ et $c = 0$.
\item
\begin{align*}
j(x) &= - 4(x + 10)(x + 4)\\&= (- 4x - 4 \times 10)(x + 4)\\&= (- 4x - 40)(x + 4)\\&= - 4x \times x - 4x \times 4 - 40x - 40 \times 4\\&= 4(- 4) \times x - 160 - 4x^{2} - 40x\\&= - 16x - 160 - 4x^{2} - 40x\\&= - 4x^{2} - 16x - 40x - 160\\&= - 4x^{2} + (- 16 - 40) \times x - 160\\&= - 4x^{2} - 56x - 160
\end{align*}
C'est un polynôme de degré 2 avec $a = - 4$, $b = - 56$ et $c = - 160$.
\item
\begin{align*}
k(x) &= - 6(x - 7)(x - 10)\\&= (- 6x - 6(- 7))(x - 10)\\&= (- 6x + 42)(x - 10)\\&= - 6x \times x - 6x(- 10) + 42x + 42(- 10)\\&= - 10(- 6) \times x - 420 - 6x^{2} + 42x\\&= 60x - 420 - 6x^{2} + 42x\\&= - 6x^{2} + 60x + 42x - 420\\&= - 6x^{2} + (60 + 42) \times x - 420\\&= - 6x^{2} + 102x - 420
\end{align*}
C'est un polynôme de degré 2 avec $a = - 6$, $b = 102$ et $c = - 420$.
\end{enumerate}
\end{solution}
\begin{exercise}[subtitle={Étude de fonctions}]
Soit $f(x) = 3x^{2} - 147$ une fonction définie sur $\R$.
\begin{enumerate}
\item Calculer les valeurs suivantes
\[
f(1) \qquad f(-2)
\]
\item Dériver la fonction $f$
\item Étudier le signe de $f'$ puis en déduire les variations de $f$.
\item Est-ce que $f$ admet un maximum? un minimum? Calculer sa valeur.
\end{enumerate}
\end{exercise}
\begin{solution}
\begin{enumerate}
\item On remplace $x$ par les valeurs demandées
\[
f(1) = 3 \times 1^{2} - 147=3 \times 1 - 147=3 - 147=- 144
\]
\[
f(-1) = 3 \times - 1^{2} - 147=3 \times 1 - 147=3 - 147=- 144
\]
\item Dérivation
\[
f'(x) = 6x
\]
\item Pas de solutions automatiques.
\item Pas de solutions automatiques.
\end{enumerate}
\end{solution}
\begin{exercise}[subtitle={Enclos}]
Dans son garage, Jean a trouvé 35m de grillage. \\
Il décide de l'utiliser pour faire un enclos rectangulaire. Afin d'obtenir un enclos encore plus grand, il veut utiliser le mur du jardin qui formera un côté. Le grillage formera les 3 autres.
\begin{center}
\includegraphics[scale=0.8]{./fig/enclos}
\end{center}
Comment placer les poteaux pour avoir un enclos le plus grand possible ?
\textit{Cet exercice est un exercice de recherche. Vous êtes invité à utiliser les outils que vous voulez. Une part importante de la note sera dédiée à la rédaction et aux explications de ce que vous faites. Vous êtes encouragé à faire des schémas.}
\textit{Si vous utilisez le tableur, vous devrez le mentionner et m'envoyer le tableur à l'adresse \url{benjamin.bertrand@ac-lyon.fr}}
\end{exercise}
\begin{solution}
Formule de l'aire (en notant $x$ la longueur du côté de l'enclos à côté du mur)
\[
A(x) = x(35 - 2x) = - 2x^{2} + 35x
\]
On va donc étudier les variations de la fonction $A(x) = - 2x^{2} + 35x$
\begin{itemize}
\item Fonction dérivée : $A'(x) = - 4x + 35$
\item On résout l'inéquation $A'(x) \geq 0$ pour déterminer quand la fonction $A'$ est positive.
\begin{align*}
A(x) & \geq 0 \\
- 4x + 35 & \geq 0 \\
- 4x + 35 + - 35 &\geq 0 + - 35 \\
- 4x &\geq - 35 \\
\frac{- 4x}{- 4} &\leq \frac{- 35}{- 4} \\
x &\leq \dfrac{35}{4} \\
\end{align*}
Donc $A(x)$ est positif quand $x$ est plus \textbf{petit} que $\dfrac{35}{4}$
\item
\begin{center}
\begin{tikzpicture}
\tkzTabInit[lgt=3,espcl=4]{$x$/1,Signe de $f'(x)$/2, Variations de $f(x)$/2}{, $\dfrac{35}{4}$ ,}%
\tkzTabLine{, +, z, -, }
\tkzTabVar{-/ ,+/$f(\dfrac{35}{4}) = \dfrac{2450}{16}$ , -/}%
\end{tikzpicture}
\end{center}
\end{itemize}
Pour avoir le plus grand enclos, il faut placer les poteaux à 6m du mur et on aura alors un enclos de $72m^2$.
\end{solution}
\end{document}
%%% Local Variables:
%%% mode: latex
%%% TeX-master: "master"
%%% End:

View File

@ -0,0 +1,19 @@
# bopytex_config.py
from mapytex.calculus.random import expression as random_expression
from mapytex.calculus.random import list as random_list
from mapytex.calculus.API.tokens.polynomial import Polynomial
from mapytex import render
import random
random.seed(0) # Controlling the seed allows to make subject reproductible
render.set_render("tex")
direct_access = {
"Polynomial": Polynomial,
"random_expression": random_expression,
"random_list": random_list,
"random": random,
"min": min,
"max": max,
}

Binary file not shown.

View File

@ -0,0 +1,158 @@
\documentclass[a4paper,12pt]{article}
\usepackage{myXsim}
\usepackage{pgfplots}
\usetikzlibrary{decorations.markings}
\pgfplotsset{compat=1.18}
\title{ DM1 \hfill ANEX DIT CHENAUD Lou}
\tribe{1ST}
\date{A rendre pour le lundi 27 mars 2023}
\duree{}
\xsimsetup{
solution/print = true
}
\pagestyle{empty}
\begin{document}
\maketitle
Le barème est donné à titre indicatif, il pourra être modifié.
\begin{exercise}[subtitle={Polynôme de degré 2}]
Développer les expressions suivantes pour vérifier que ce sont des polynômes de degré 2. Vous préciserez les valeurs de $a$, $b$ et $c$.
\begin{multicols}{2}
\begin{enumerate}
\item $f(x) = (- 6x + 3)(- 7x + 3)$
\item $g(x) = (10x - 3)^{2}$
\item $h(x) = - 4 + x(- 7x - 10)$
\item $i(x) = - 4x^{2} + x(- 8x - 1)$
\item $j(x) = - 3(x + 6)(x + 8)$
\item $k(x) = 6(x + 5)(x - 10)$
\end{enumerate}
\end{multicols}
\end{exercise}
\begin{solution}
\begin{enumerate}
\item
\begin{align*}
f(x) &= (- 6x + 3)(- 7x + 3)\\&= - 6x \times - 7x - 6x \times 3 + 3 \times - 7x + 3 \times 3\\&= - 6(- 7) \times x^{1 + 1} + 3(- 6) \times x + 3(- 7) \times x + 9\\&= - 18x - 21x + 42x^{2} + 9\\&= (- 18 - 21) \times x + 42x^{2} + 9\\&= 42x^{2} - 39x + 9
\end{align*}
C'est un polynôme de degré 2 avec $a = 42$, $b = - 39$ et $c = 9$.
\item
\begin{align*}
g(x) &= (10x - 3)^{2}\\&= (10x - 3)(10x - 3)\\&= 10x \times 10x + 10x(- 3) - 3 \times 10x - 3(- 3)\\&= 10 \times 10 \times x^{1 + 1} - 3 \times 10 \times x - 3 \times 10 \times x + 9\\&= - 30x - 30x + 100x^{2} + 9\\&= (- 30 - 30) \times x + 100x^{2} + 9\\&= 100x^{2} - 60x + 9
\end{align*}
C'est un polynôme de degré 2 avec $a = 100$, $b = - 60$ et $c = 9$.
\item
\begin{align*}
h(x) &= - 4 + x(- 7x - 10)\\&= - 4 + x \times - 7x + x(- 10)\\&= - 7x^{2} - 10x - 4
\end{align*}
C'est un polynôme de degré 2 avec $a = - 7$, $b = - 10$ et $c = - 4$.
\item
\begin{align*}
i(x) &= - 4x^{2} + x(- 8x - 1)\\&= - 4x^{2} + x \times - 8x + x(- 1)\\&= - 4x^{2} - 8x^{2} - x\\&= - 4x^{2} - 8x^{2} - x\\&= (- 4 - 8) \times x^{2} - x\\&= - 12x^{2} - x
\end{align*}
C'est un polynôme de degré 2 avec $a = - 12$, $b = - 1$ et $c = 0$.
\item
\begin{align*}
j(x) &= - 3(x + 6)(x + 8)\\&= (- 3x - 3 \times 6)(x + 8)\\&= (- 3x - 18)(x + 8)\\&= - 3x \times x - 3x \times 8 - 18x - 18 \times 8\\&= 8(- 3) \times x - 144 - 3x^{2} - 18x\\&= - 24x - 144 - 3x^{2} - 18x\\&= - 3x^{2} - 24x - 18x - 144\\&= - 3x^{2} + (- 24 - 18) \times x - 144\\&= - 3x^{2} - 42x - 144
\end{align*}
C'est un polynôme de degré 2 avec $a = - 3$, $b = - 42$ et $c = - 144$.
\item
\begin{align*}
k(x) &= 6(x + 5)(x - 10)\\&= (6x + 6 \times 5)(x - 10)\\&= (6x + 30)(x - 10)\\&= 6x \times x + 6x(- 10) + 30x + 30(- 10)\\&= - 10 \times 6 \times x - 300 + 6x^{2} + 30x\\&= - 60x - 300 + 6x^{2} + 30x\\&= 6x^{2} - 60x + 30x - 300\\&= 6x^{2} + (- 60 + 30) \times x - 300\\&= 6x^{2} - 30x - 300
\end{align*}
C'est un polynôme de degré 2 avec $a = 6$, $b = - 30$ et $c = - 300$.
\end{enumerate}
\end{solution}
\begin{exercise}[subtitle={Étude de fonctions}]
Soit $f(x) = - 2x^{2} + 32x - 120$ une fonction définie sur $\R$.
\begin{enumerate}
\item Calculer les valeurs suivantes
\[
f(1) \qquad f(-2)
\]
\item Dériver la fonction $f$
\item Étudier le signe de $f'$ puis en déduire les variations de $f$.
\item Est-ce que $f$ admet un maximum? un minimum? Calculer sa valeur.
\end{enumerate}
\end{exercise}
\begin{solution}
\begin{enumerate}
\item On remplace $x$ par les valeurs demandées
\[
f(1) = - 2 \times 1^{2} + 32 \times 1 - 120=- 2 \times 1 + 32 - 120=- 2 - 88=- 90
\]
\[
f(-1) = - 2 \times - 1^{2} + 32(- 1) - 120=- 2 \times 1 - 32 - 120=- 2 - 152=- 154
\]
\item Dérivation
\[
f'(x) = - 4x + 32
\]
\item Pas de solutions automatiques.
\item Pas de solutions automatiques.
\end{enumerate}
\end{solution}
\begin{exercise}[subtitle={Enclos}]
Dans son garage, Jean a trouvé 24m de grillage. \\
Il décide de l'utiliser pour faire un enclos rectangulaire. Afin d'obtenir un enclos encore plus grand, il veut utiliser le mur du jardin qui formera un côté. Le grillage formera les 3 autres.
\begin{center}
\includegraphics[scale=0.8]{./fig/enclos}
\end{center}
Comment placer les poteaux pour avoir un enclos le plus grand possible ?
\textit{Cet exercice est un exercice de recherche. Vous êtes invité à utiliser les outils que vous voulez. Une part importante de la note sera dédiée à la rédaction et aux explications de ce que vous faites. Vous êtes encouragé à faire des schémas.}
\textit{Si vous utilisez le tableur, vous devrez le mentionner et m'envoyer le tableur à l'adresse \url{benjamin.bertrand@ac-lyon.fr}}
\end{exercise}
\begin{solution}
Formule de l'aire (en notant $x$ la longueur du côté de l'enclos à côté du mur)
\[
A(x) = x(24 - 2x) = - 2x^{2} + 24x
\]
On va donc étudier les variations de la fonction $A(x) = - 2x^{2} + 24x$
\begin{itemize}
\item Fonction dérivée : $A'(x) = - 4x + 24$
\item On résout l'inéquation $A'(x) \geq 0$ pour déterminer quand la fonction $A'$ est positive.
\begin{align*}
A(x) & \geq 0 \\
- 4x + 24 & \geq 0 \\
- 4x + 24 + - 24 &\geq 0 + - 24 \\
- 4x &\geq - 24 \\
\frac{- 4x}{- 4} &\leq \frac{- 24}{- 4} \\
x &\leq 6 \\
\end{align*}
Donc $A(x)$ est positif quand $x$ est plus \textbf{petit} que $6$
\item
\begin{center}
\begin{tikzpicture}
\tkzTabInit[lgt=3,espcl=4]{$x$/1,Signe de $f'(x)$/2, Variations de $f(x)$/2}{, $6$ ,}%
\tkzTabLine{, +, z, -, }
\tkzTabVar{-/ ,+/$f(6) = 72$ , -/}%
\end{tikzpicture}
\end{center}
\end{itemize}
Pour avoir le plus grand enclos, il faut placer les poteaux à 6m du mur et on aura alors un enclos de $72m^2$.
\end{solution}
\end{document}
%%% Local Variables:
%%% mode: latex
%%% TeX-master: "master"
%%% End:

Binary file not shown.

View File

@ -0,0 +1,158 @@
\documentclass[a4paper,12pt]{article}
\usepackage{myXsim}
\usepackage{pgfplots}
\usetikzlibrary{decorations.markings}
\pgfplotsset{compat=1.18}
\title{ DM1 \hfill BADEL Melinda}
\tribe{1ST}
\date{A rendre pour le lundi 27 mars 2023}
\duree{}
\xsimsetup{
solution/print = true
}
\pagestyle{empty}
\begin{document}
\maketitle
Le barème est donné à titre indicatif, il pourra être modifié.
\begin{exercise}[subtitle={Polynôme de degré 2}]
Développer les expressions suivantes pour vérifier que ce sont des polynômes de degré 2. Vous préciserez les valeurs de $a$, $b$ et $c$.
\begin{multicols}{2}
\begin{enumerate}
\item $f(x) = (4x - 9)(5x - 9)$
\item $g(x) = (- 4x - 2)^{2}$
\item $h(x) = 5 + x(- 8x - 6)$
\item $i(x) = - 9x^{2} + x(8x - 2)$
\item $j(x) = - 5(x - 3)(x - 4)$
\item $k(x) = 3(x - 2)(x + 10)$
\end{enumerate}
\end{multicols}
\end{exercise}
\begin{solution}
\begin{enumerate}
\item
\begin{align*}
f(x) &= (4x - 9)(5x - 9)\\&= 4x \times 5x + 4x(- 9) - 9 \times 5x - 9(- 9)\\&= 4 \times 5 \times x^{1 + 1} - 9 \times 4 \times x - 9 \times 5 \times x + 81\\&= - 36x - 45x + 20x^{2} + 81\\&= (- 36 - 45) \times x + 20x^{2} + 81\\&= 20x^{2} - 81x + 81
\end{align*}
C'est un polynôme de degré 2 avec $a = 20$, $b = - 81$ et $c = 81$.
\item
\begin{align*}
g(x) &= (- 4x - 2)^{2}\\&= (- 4x - 2)(- 4x - 2)\\&= - 4x \times - 4x - 4x(- 2) - 2 \times - 4x - 2(- 2)\\&= - 4(- 4) \times x^{1 + 1} - 2(- 4) \times x - 2(- 4) \times x + 4\\&= 8x + 8x + 16x^{2} + 4\\&= (8 + 8) \times x + 16x^{2} + 4\\&= 16x^{2} + 16x + 4
\end{align*}
C'est un polynôme de degré 2 avec $a = 16$, $b = 16$ et $c = 4$.
\item
\begin{align*}
h(x) &= 5 + x(- 8x - 6)\\&= 5 + x \times - 8x + x(- 6)\\&= - 8x^{2} - 6x + 5
\end{align*}
C'est un polynôme de degré 2 avec $a = - 8$, $b = - 6$ et $c = 5$.
\item
\begin{align*}
i(x) &= - 9x^{2} + x(8x - 2)\\&= - 9x^{2} + x \times 8x + x(- 2)\\&= - 9x^{2} + 8x^{2} - 2x\\&= - 9x^{2} + 8x^{2} - 2x\\&= (- 9 + 8) \times x^{2} - 2x\\&= - x^{2} - 2x
\end{align*}
C'est un polynôme de degré 2 avec $a = - 1$, $b = - 2$ et $c = 0$.
\item
\begin{align*}
j(x) &= - 5(x - 3)(x - 4)\\&= (- 5x - 5(- 3))(x - 4)\\&= (- 5x + 15)(x - 4)\\&= - 5x \times x - 5x(- 4) + 15x + 15(- 4)\\&= - 4(- 5) \times x - 60 - 5x^{2} + 15x\\&= 20x - 60 - 5x^{2} + 15x\\&= - 5x^{2} + 20x + 15x - 60\\&= - 5x^{2} + (20 + 15) \times x - 60\\&= - 5x^{2} + 35x - 60
\end{align*}
C'est un polynôme de degré 2 avec $a = - 5$, $b = 35$ et $c = - 60$.
\item
\begin{align*}
k(x) &= 3(x - 2)(x + 10)\\&= (3x + 3(- 2))(x + 10)\\&= (3x - 6)(x + 10)\\&= 3x \times x + 3x \times 10 - 6x - 6 \times 10\\&= 10 \times 3 \times x - 60 + 3x^{2} - 6x\\&= 30x - 60 + 3x^{2} - 6x\\&= 3x^{2} + 30x - 6x - 60\\&= 3x^{2} + (30 - 6) \times x - 60\\&= 3x^{2} + 24x - 60
\end{align*}
C'est un polynôme de degré 2 avec $a = 3$, $b = 24$ et $c = - 60$.
\end{enumerate}
\end{solution}
\begin{exercise}[subtitle={Étude de fonctions}]
Soit $f(x) = 4x^{2} + 20x - 56$ une fonction définie sur $\R$.
\begin{enumerate}
\item Calculer les valeurs suivantes
\[
f(1) \qquad f(-2)
\]
\item Dériver la fonction $f$
\item Étudier le signe de $f'$ puis en déduire les variations de $f$.
\item Est-ce que $f$ admet un maximum? un minimum? Calculer sa valeur.
\end{enumerate}
\end{exercise}
\begin{solution}
\begin{enumerate}
\item On remplace $x$ par les valeurs demandées
\[
f(1) = 4 \times 1^{2} + 20 \times 1 - 56=4 \times 1 + 20 - 56=4 - 36=- 32
\]
\[
f(-1) = 4 \times - 1^{2} + 20(- 1) - 56=4 \times 1 - 20 - 56=4 - 76=- 72
\]
\item Dérivation
\[
f'(x) = 8x + 20
\]
\item Pas de solutions automatiques.
\item Pas de solutions automatiques.
\end{enumerate}
\end{solution}
\begin{exercise}[subtitle={Enclos}]
Dans son garage, Jean a trouvé 37m de grillage. \\
Il décide de l'utiliser pour faire un enclos rectangulaire. Afin d'obtenir un enclos encore plus grand, il veut utiliser le mur du jardin qui formera un côté. Le grillage formera les 3 autres.
\begin{center}
\includegraphics[scale=0.8]{./fig/enclos}
\end{center}
Comment placer les poteaux pour avoir un enclos le plus grand possible ?
\textit{Cet exercice est un exercice de recherche. Vous êtes invité à utiliser les outils que vous voulez. Une part importante de la note sera dédiée à la rédaction et aux explications de ce que vous faites. Vous êtes encouragé à faire des schémas.}
\textit{Si vous utilisez le tableur, vous devrez le mentionner et m'envoyer le tableur à l'adresse \url{benjamin.bertrand@ac-lyon.fr}}
\end{exercise}
\begin{solution}
Formule de l'aire (en notant $x$ la longueur du côté de l'enclos à côté du mur)
\[
A(x) = x(37 - 2x) = - 2x^{2} + 37x
\]
On va donc étudier les variations de la fonction $A(x) = - 2x^{2} + 37x$
\begin{itemize}
\item Fonction dérivée : $A'(x) = - 4x + 37$
\item On résout l'inéquation $A'(x) \geq 0$ pour déterminer quand la fonction $A'$ est positive.
\begin{align*}
A(x) & \geq 0 \\
- 4x + 37 & \geq 0 \\
- 4x + 37 + - 37 &\geq 0 + - 37 \\
- 4x &\geq - 37 \\
\frac{- 4x}{- 4} &\leq \frac{- 37}{- 4} \\
x &\leq \dfrac{37}{4} \\
\end{align*}
Donc $A(x)$ est positif quand $x$ est plus \textbf{petit} que $\dfrac{37}{4}$
\item
\begin{center}
\begin{tikzpicture}
\tkzTabInit[lgt=3,espcl=4]{$x$/1,Signe de $f'(x)$/2, Variations de $f(x)$/2}{, $\dfrac{37}{4}$ ,}%
\tkzTabLine{, +, z, -, }
\tkzTabVar{-/ ,+/$f(\dfrac{37}{4}) = \dfrac{2738}{16}$ , -/}%
\end{tikzpicture}
\end{center}
\end{itemize}
Pour avoir le plus grand enclos, il faut placer les poteaux à 6m du mur et on aura alors un enclos de $72m^2$.
\end{solution}
\end{document}
%%% Local Variables:
%%% mode: latex
%%% TeX-master: "master"
%%% End:

Binary file not shown.

View File

@ -0,0 +1,158 @@
\documentclass[a4paper,12pt]{article}
\usepackage{myXsim}
\usepackage{pgfplots}
\usetikzlibrary{decorations.markings}
\pgfplotsset{compat=1.18}
\title{ DM1 \hfill BALTA Zina}
\tribe{1ST}
\date{A rendre pour le lundi 27 mars 2023}
\duree{}
\xsimsetup{
solution/print = true
}
\pagestyle{empty}
\begin{document}
\maketitle
Le barème est donné à titre indicatif, il pourra être modifié.
\begin{exercise}[subtitle={Polynôme de degré 2}]
Développer les expressions suivantes pour vérifier que ce sont des polynômes de degré 2. Vous préciserez les valeurs de $a$, $b$ et $c$.
\begin{multicols}{2}
\begin{enumerate}
\item $f(x) = (- 4x - 9)(2x - 9)$
\item $g(x) = (9x - 9)^{2}$
\item $h(x) = 4 + x(8x - 1)$
\item $i(x) = 9x^{2} + x(9x + 6)$
\item $j(x) = - 5(x + 4)(x - 2)$
\item $k(x) = - 9(x - 3)(x + 4)$
\end{enumerate}
\end{multicols}
\end{exercise}
\begin{solution}
\begin{enumerate}
\item
\begin{align*}
f(x) &= (- 4x - 9)(2x - 9)\\&= - 4x \times 2x - 4x(- 9) - 9 \times 2x - 9(- 9)\\&= - 4 \times 2 \times x^{1 + 1} - 9(- 4) \times x - 9 \times 2 \times x + 81\\&= 36x - 18x - 8x^{2} + 81\\&= (36 - 18) \times x - 8x^{2} + 81\\&= - 8x^{2} + 18x + 81
\end{align*}
C'est un polynôme de degré 2 avec $a = - 8$, $b = 18$ et $c = 81$.
\item
\begin{align*}
g(x) &= (9x - 9)^{2}\\&= (9x - 9)(9x - 9)\\&= 9x \times 9x + 9x(- 9) - 9 \times 9x - 9(- 9)\\&= 9 \times 9 \times x^{1 + 1} - 9 \times 9 \times x - 9 \times 9 \times x + 81\\&= - 81x - 81x + 81x^{2} + 81\\&= (- 81 - 81) \times x + 81x^{2} + 81\\&= 81x^{2} - 162x + 81
\end{align*}
C'est un polynôme de degré 2 avec $a = 81$, $b = - 162$ et $c = 81$.
\item
\begin{align*}
h(x) &= 4 + x(8x - 1)\\&= 4 + x \times 8x + x(- 1)\\&= 8x^{2} - x + 4
\end{align*}
C'est un polynôme de degré 2 avec $a = 8$, $b = - 1$ et $c = 4$.
\item
\begin{align*}
i(x) &= 9x^{2} + x(9x + 6)\\&= 9x^{2} + x \times 9x + x \times 6\\&= 9x^{2} + 9x^{2} + 6x\\&= 9x^{2} + 9x^{2} + 6x\\&= (9 + 9) \times x^{2} + 6x\\&= 18x^{2} + 6x
\end{align*}
C'est un polynôme de degré 2 avec $a = 18$, $b = 6$ et $c = 0$.
\item
\begin{align*}
j(x) &= - 5(x + 4)(x - 2)\\&= (- 5x - 5 \times 4)(x - 2)\\&= (- 5x - 20)(x - 2)\\&= - 5x \times x - 5x(- 2) - 20x - 20(- 2)\\&= - 2(- 5) \times x + 40 - 5x^{2} - 20x\\&= 10x + 40 - 5x^{2} - 20x\\&= - 5x^{2} + 10x - 20x + 40\\&= - 5x^{2} + (10 - 20) \times x + 40\\&= - 5x^{2} - 10x + 40
\end{align*}
C'est un polynôme de degré 2 avec $a = - 5$, $b = - 10$ et $c = 40$.
\item
\begin{align*}
k(x) &= - 9(x - 3)(x + 4)\\&= (- 9x - 9(- 3))(x + 4)\\&= (- 9x + 27)(x + 4)\\&= - 9x \times x - 9x \times 4 + 27x + 27 \times 4\\&= 4(- 9) \times x + 108 - 9x^{2} + 27x\\&= - 36x + 108 - 9x^{2} + 27x\\&= - 9x^{2} - 36x + 27x + 108\\&= - 9x^{2} + (- 36 + 27) \times x + 108\\&= - 9x^{2} - 9x + 108
\end{align*}
C'est un polynôme de degré 2 avec $a = - 9$, $b = - 9$ et $c = 108$.
\end{enumerate}
\end{solution}
\begin{exercise}[subtitle={Étude de fonctions}]
Soit $f(x) = 5x^{2} + 70x + 200$ une fonction définie sur $\R$.
\begin{enumerate}
\item Calculer les valeurs suivantes
\[
f(1) \qquad f(-2)
\]
\item Dériver la fonction $f$
\item Étudier le signe de $f'$ puis en déduire les variations de $f$.
\item Est-ce que $f$ admet un maximum? un minimum? Calculer sa valeur.
\end{enumerate}
\end{exercise}
\begin{solution}
\begin{enumerate}
\item On remplace $x$ par les valeurs demandées
\[
f(1) = 5 \times 1^{2} + 70 \times 1 + 200=5 \times 1 + 70 + 200=5 + 270=275
\]
\[
f(-1) = 5 \times - 1^{2} + 70(- 1) + 200=5 \times 1 - 70 + 200=5 + 130=135
\]
\item Dérivation
\[
f'(x) = 10x + 70
\]
\item Pas de solutions automatiques.
\item Pas de solutions automatiques.
\end{enumerate}
\end{solution}
\begin{exercise}[subtitle={Enclos}]
Dans son garage, Jean a trouvé 32m de grillage. \\
Il décide de l'utiliser pour faire un enclos rectangulaire. Afin d'obtenir un enclos encore plus grand, il veut utiliser le mur du jardin qui formera un côté. Le grillage formera les 3 autres.
\begin{center}
\includegraphics[scale=0.8]{./fig/enclos}
\end{center}
Comment placer les poteaux pour avoir un enclos le plus grand possible ?
\textit{Cet exercice est un exercice de recherche. Vous êtes invité à utiliser les outils que vous voulez. Une part importante de la note sera dédiée à la rédaction et aux explications de ce que vous faites. Vous êtes encouragé à faire des schémas.}
\textit{Si vous utilisez le tableur, vous devrez le mentionner et m'envoyer le tableur à l'adresse \url{benjamin.bertrand@ac-lyon.fr}}
\end{exercise}
\begin{solution}
Formule de l'aire (en notant $x$ la longueur du côté de l'enclos à côté du mur)
\[
A(x) = x(32 - 2x) = - 2x^{2} + 32x
\]
On va donc étudier les variations de la fonction $A(x) = - 2x^{2} + 32x$
\begin{itemize}
\item Fonction dérivée : $A'(x) = - 4x + 32$
\item On résout l'inéquation $A'(x) \geq 0$ pour déterminer quand la fonction $A'$ est positive.
\begin{align*}
A(x) & \geq 0 \\
- 4x + 32 & \geq 0 \\
- 4x + 32 + - 32 &\geq 0 + - 32 \\
- 4x &\geq - 32 \\
\frac{- 4x}{- 4} &\leq \frac{- 32}{- 4} \\
x &\leq 8 \\
\end{align*}
Donc $A(x)$ est positif quand $x$ est plus \textbf{petit} que $8$
\item
\begin{center}
\begin{tikzpicture}
\tkzTabInit[lgt=3,espcl=4]{$x$/1,Signe de $f'(x)$/2, Variations de $f(x)$/2}{, $8$ ,}%
\tkzTabLine{, +, z, -, }
\tkzTabVar{-/ ,+/$f(8) = 128$ , -/}%
\end{tikzpicture}
\end{center}
\end{itemize}
Pour avoir le plus grand enclos, il faut placer les poteaux à 6m du mur et on aura alors un enclos de $72m^2$.
\end{solution}
\end{document}
%%% Local Variables:
%%% mode: latex
%%% TeX-master: "master"
%%% End:

Binary file not shown.

View File

@ -0,0 +1,158 @@
\documentclass[a4paper,12pt]{article}
\usepackage{myXsim}
\usepackage{pgfplots}
\usetikzlibrary{decorations.markings}
\pgfplotsset{compat=1.18}
\title{ DM1 \hfill BARDOUSSE Yanis}
\tribe{1ST}
\date{A rendre pour le lundi 27 mars 2023}
\duree{}
\xsimsetup{
solution/print = true
}
\pagestyle{empty}
\begin{document}
\maketitle
Le barème est donné à titre indicatif, il pourra être modifié.
\begin{exercise}[subtitle={Polynôme de degré 2}]
Développer les expressions suivantes pour vérifier que ce sont des polynômes de degré 2. Vous préciserez les valeurs de $a$, $b$ et $c$.
\begin{multicols}{2}
\begin{enumerate}
\item $f(x) = (5x - 2)(- 10x - 2)$
\item $g(x) = (- 9x - 2)^{2}$
\item $h(x) = 8 + x(- 2x - 1)$
\item $i(x) = - 10x^{2} + x(- 5x + 4)$
\item $j(x) = - 9(x - 1)(x - 7)$
\item $k(x) = - 10(x + 7)(x + 9)$
\end{enumerate}
\end{multicols}
\end{exercise}
\begin{solution}
\begin{enumerate}
\item
\begin{align*}
f(x) &= (5x - 2)(- 10x - 2)\\&= 5x \times - 10x + 5x(- 2) - 2 \times - 10x - 2(- 2)\\&= 5(- 10) \times x^{1 + 1} - 2 \times 5 \times x - 2(- 10) \times x + 4\\&= - 10x + 20x - 50x^{2} + 4\\&= (- 10 + 20) \times x - 50x^{2} + 4\\&= - 50x^{2} + 10x + 4
\end{align*}
C'est un polynôme de degré 2 avec $a = - 50$, $b = 10$ et $c = 4$.
\item
\begin{align*}
g(x) &= (- 9x - 2)^{2}\\&= (- 9x - 2)(- 9x - 2)\\&= - 9x \times - 9x - 9x(- 2) - 2 \times - 9x - 2(- 2)\\&= - 9(- 9) \times x^{1 + 1} - 2(- 9) \times x - 2(- 9) \times x + 4\\&= 18x + 18x + 81x^{2} + 4\\&= (18 + 18) \times x + 81x^{2} + 4\\&= 81x^{2} + 36x + 4
\end{align*}
C'est un polynôme de degré 2 avec $a = 81$, $b = 36$ et $c = 4$.
\item
\begin{align*}
h(x) &= 8 + x(- 2x - 1)\\&= 8 + x \times - 2x + x(- 1)\\&= - 2x^{2} - x + 8
\end{align*}
C'est un polynôme de degré 2 avec $a = - 2$, $b = - 1$ et $c = 8$.
\item
\begin{align*}
i(x) &= - 10x^{2} + x(- 5x + 4)\\&= - 10x^{2} + x \times - 5x + x \times 4\\&= - 10x^{2} - 5x^{2} + 4x\\&= - 10x^{2} - 5x^{2} + 4x\\&= (- 10 - 5) \times x^{2} + 4x\\&= - 15x^{2} + 4x
\end{align*}
C'est un polynôme de degré 2 avec $a = - 15$, $b = 4$ et $c = 0$.
\item
\begin{align*}
j(x) &= - 9(x - 1)(x - 7)\\&= (- 9x - 9(- 1))(x - 7)\\&= (- 9x + 9)(x - 7)\\&= - 9x \times x - 9x(- 7) + 9x + 9(- 7)\\&= - 7(- 9) \times x - 63 - 9x^{2} + 9x\\&= 63x - 63 - 9x^{2} + 9x\\&= - 9x^{2} + 63x + 9x - 63\\&= - 9x^{2} + (63 + 9) \times x - 63\\&= - 9x^{2} + 72x - 63
\end{align*}
C'est un polynôme de degré 2 avec $a = - 9$, $b = 72$ et $c = - 63$.
\item
\begin{align*}
k(x) &= - 10(x + 7)(x + 9)\\&= (- 10x - 10 \times 7)(x + 9)\\&= (- 10x - 70)(x + 9)\\&= - 10x \times x - 10x \times 9 - 70x - 70 \times 9\\&= 9(- 10) \times x - 630 - 10x^{2} - 70x\\&= - 90x - 630 - 10x^{2} - 70x\\&= - 10x^{2} - 90x - 70x - 630\\&= - 10x^{2} + (- 90 - 70) \times x - 630\\&= - 10x^{2} - 160x - 630
\end{align*}
C'est un polynôme de degré 2 avec $a = - 10$, $b = - 160$ et $c = - 630$.
\end{enumerate}
\end{solution}
\begin{exercise}[subtitle={Étude de fonctions}]
Soit $f(x) = 7x^{2} - 77x + 168$ une fonction définie sur $\R$.
\begin{enumerate}
\item Calculer les valeurs suivantes
\[
f(1) \qquad f(-2)
\]
\item Dériver la fonction $f$
\item Étudier le signe de $f'$ puis en déduire les variations de $f$.
\item Est-ce que $f$ admet un maximum? un minimum? Calculer sa valeur.
\end{enumerate}
\end{exercise}
\begin{solution}
\begin{enumerate}
\item On remplace $x$ par les valeurs demandées
\[
f(1) = 7 \times 1^{2} - 77 \times 1 + 168=7 \times 1 - 77 + 168=7 + 91=98
\]
\[
f(-1) = 7 \times - 1^{2} - 77(- 1) + 168=7 \times 1 + 77 + 168=7 + 245=252
\]
\item Dérivation
\[
f'(x) = 14x - 77
\]
\item Pas de solutions automatiques.
\item Pas de solutions automatiques.
\end{enumerate}
\end{solution}
\begin{exercise}[subtitle={Enclos}]
Dans son garage, Jean a trouvé 34m de grillage. \\
Il décide de l'utiliser pour faire un enclos rectangulaire. Afin d'obtenir un enclos encore plus grand, il veut utiliser le mur du jardin qui formera un côté. Le grillage formera les 3 autres.
\begin{center}
\includegraphics[scale=0.8]{./fig/enclos}
\end{center}
Comment placer les poteaux pour avoir un enclos le plus grand possible ?
\textit{Cet exercice est un exercice de recherche. Vous êtes invité à utiliser les outils que vous voulez. Une part importante de la note sera dédiée à la rédaction et aux explications de ce que vous faites. Vous êtes encouragé à faire des schémas.}
\textit{Si vous utilisez le tableur, vous devrez le mentionner et m'envoyer le tableur à l'adresse \url{benjamin.bertrand@ac-lyon.fr}}
\end{exercise}
\begin{solution}
Formule de l'aire (en notant $x$ la longueur du côté de l'enclos à côté du mur)
\[
A(x) = x(34 - 2x) = - 2x^{2} + 34x
\]
On va donc étudier les variations de la fonction $A(x) = - 2x^{2} + 34x$
\begin{itemize}
\item Fonction dérivée : $A'(x) = - 4x + 34$
\item On résout l'inéquation $A'(x) \geq 0$ pour déterminer quand la fonction $A'$ est positive.
\begin{align*}
A(x) & \geq 0 \\
- 4x + 34 & \geq 0 \\
- 4x + 34 + - 34 &\geq 0 + - 34 \\
- 4x &\geq - 34 \\
\frac{- 4x}{- 4} &\leq \frac{- 34}{- 4} \\
x &\leq \dfrac{17}{2} \\
\end{align*}
Donc $A(x)$ est positif quand $x$ est plus \textbf{petit} que $\dfrac{17}{2}$
\item
\begin{center}
\begin{tikzpicture}
\tkzTabInit[lgt=3,espcl=4]{$x$/1,Signe de $f'(x)$/2, Variations de $f(x)$/2}{, $\dfrac{17}{2}$ ,}%
\tkzTabLine{, +, z, -, }
\tkzTabVar{-/ ,+/$f(\dfrac{17}{2}) = \dfrac{578}{4}$ , -/}%
\end{tikzpicture}
\end{center}
\end{itemize}
Pour avoir le plus grand enclos, il faut placer les poteaux à 6m du mur et on aura alors un enclos de $72m^2$.
\end{solution}
\end{document}
%%% Local Variables:
%%% mode: latex
%%% TeX-master: "master"
%%% End:

Binary file not shown.

View File

@ -0,0 +1,158 @@
\documentclass[a4paper,12pt]{article}
\usepackage{myXsim}
\usepackage{pgfplots}
\usetikzlibrary{decorations.markings}
\pgfplotsset{compat=1.18}
\title{ DM1 \hfill BLONDIN Damien}
\tribe{1ST}
\date{A rendre pour le lundi 27 mars 2023}
\duree{}
\xsimsetup{
solution/print = true
}
\pagestyle{empty}
\begin{document}
\maketitle
Le barème est donné à titre indicatif, il pourra être modifié.
\begin{exercise}[subtitle={Polynôme de degré 2}]
Développer les expressions suivantes pour vérifier que ce sont des polynômes de degré 2. Vous préciserez les valeurs de $a$, $b$ et $c$.
\begin{multicols}{2}
\begin{enumerate}
\item $f(x) = (- 4x + 4)(10x + 4)$
\item $g(x) = (4x + 6)^{2}$
\item $h(x) = 6 + x(3x + 4)$
\item $i(x) = - 2x^{2} + x(- 7x - 3)$
\item $j(x) = - 3(x + 9)(x + 10)$
\item $k(x) = - 1(x - 6)(x + 8)$
\end{enumerate}
\end{multicols}
\end{exercise}
\begin{solution}
\begin{enumerate}
\item
\begin{align*}
f(x) &= (- 4x + 4)(10x + 4)\\&= - 4x \times 10x - 4x \times 4 + 4 \times 10x + 4 \times 4\\&= - 4 \times 10 \times x^{1 + 1} + 4(- 4) \times x + 4 \times 10 \times x + 16\\&= - 16x + 40x - 40x^{2} + 16\\&= (- 16 + 40) \times x - 40x^{2} + 16\\&= - 40x^{2} + 24x + 16
\end{align*}
C'est un polynôme de degré 2 avec $a = - 40$, $b = 24$ et $c = 16$.
\item
\begin{align*}
g(x) &= (4x + 6)^{2}\\&= (4x + 6)(4x + 6)\\&= 4x \times 4x + 4x \times 6 + 6 \times 4x + 6 \times 6\\&= 4 \times 4 \times x^{1 + 1} + 6 \times 4 \times x + 6 \times 4 \times x + 36\\&= 24x + 24x + 16x^{2} + 36\\&= (24 + 24) \times x + 16x^{2} + 36\\&= 16x^{2} + 48x + 36
\end{align*}
C'est un polynôme de degré 2 avec $a = 16$, $b = 48$ et $c = 36$.
\item
\begin{align*}
h(x) &= 6 + x(3x + 4)\\&= 6 + x \times 3x + x \times 4\\&= 3x^{2} + 4x + 6
\end{align*}
C'est un polynôme de degré 2 avec $a = 3$, $b = 4$ et $c = 6$.
\item
\begin{align*}
i(x) &= - 2x^{2} + x(- 7x - 3)\\&= - 2x^{2} + x \times - 7x + x(- 3)\\&= - 2x^{2} - 7x^{2} - 3x\\&= - 2x^{2} - 7x^{2} - 3x\\&= (- 2 - 7) \times x^{2} - 3x\\&= - 9x^{2} - 3x
\end{align*}
C'est un polynôme de degré 2 avec $a = - 9$, $b = - 3$ et $c = 0$.
\item
\begin{align*}
j(x) &= - 3(x + 9)(x + 10)\\&= (- 3x - 3 \times 9)(x + 10)\\&= (- 3x - 27)(x + 10)\\&= - 3x \times x - 3x \times 10 - 27x - 27 \times 10\\&= 10(- 3) \times x - 270 - 3x^{2} - 27x\\&= - 30x - 270 - 3x^{2} - 27x\\&= - 3x^{2} - 30x - 27x - 270\\&= - 3x^{2} + (- 30 - 27) \times x - 270\\&= - 3x^{2} - 57x - 270
\end{align*}
C'est un polynôme de degré 2 avec $a = - 3$, $b = - 57$ et $c = - 270$.
\item
\begin{align*}
k(x) &= - 1(x - 6)(x + 8)\\&= (- 1x - 1(- 6))(x + 8)\\&= (- x + 6)(x + 8)\\&= (- x) \times x + (- x) \times 8 + 6x + 6 \times 8\\&= 8(- 1) \times x + 48 - x^{2} + 6x\\&= - 8x + 48 - x^{2} + 6x\\&= - x^{2} - 8x + 6x + 48\\&= - x^{2} + (- 8 + 6) \times x + 48\\&= - x^{2} - 2x + 48
\end{align*}
C'est un polynôme de degré 2 avec $a = - 1$, $b = - 2$ et $c = 48$.
\end{enumerate}
\end{solution}
\begin{exercise}[subtitle={Étude de fonctions}]
Soit $f(x) = 10x^{2} + 60x + 50$ une fonction définie sur $\R$.
\begin{enumerate}
\item Calculer les valeurs suivantes
\[
f(1) \qquad f(-2)
\]
\item Dériver la fonction $f$
\item Étudier le signe de $f'$ puis en déduire les variations de $f$.
\item Est-ce que $f$ admet un maximum? un minimum? Calculer sa valeur.
\end{enumerate}
\end{exercise}
\begin{solution}
\begin{enumerate}
\item On remplace $x$ par les valeurs demandées
\[
f(1) = 10 \times 1^{2} + 60 \times 1 + 50=10 \times 1 + 60 + 50=10 + 110=120
\]
\[
f(-1) = 10 \times - 1^{2} + 60(- 1) + 50=10 \times 1 - 60 + 50=10 - 10=0
\]
\item Dérivation
\[
f'(x) = 20x + 60
\]
\item Pas de solutions automatiques.
\item Pas de solutions automatiques.
\end{enumerate}
\end{solution}
\begin{exercise}[subtitle={Enclos}]
Dans son garage, Jean a trouvé 30m de grillage. \\
Il décide de l'utiliser pour faire un enclos rectangulaire. Afin d'obtenir un enclos encore plus grand, il veut utiliser le mur du jardin qui formera un côté. Le grillage formera les 3 autres.
\begin{center}
\includegraphics[scale=0.8]{./fig/enclos}
\end{center}
Comment placer les poteaux pour avoir un enclos le plus grand possible ?
\textit{Cet exercice est un exercice de recherche. Vous êtes invité à utiliser les outils que vous voulez. Une part importante de la note sera dédiée à la rédaction et aux explications de ce que vous faites. Vous êtes encouragé à faire des schémas.}
\textit{Si vous utilisez le tableur, vous devrez le mentionner et m'envoyer le tableur à l'adresse \url{benjamin.bertrand@ac-lyon.fr}}
\end{exercise}
\begin{solution}
Formule de l'aire (en notant $x$ la longueur du côté de l'enclos à côté du mur)
\[
A(x) = x(30 - 2x) = - 2x^{2} + 30x
\]
On va donc étudier les variations de la fonction $A(x) = - 2x^{2} + 30x$
\begin{itemize}
\item Fonction dérivée : $A'(x) = - 4x + 30$
\item On résout l'inéquation $A'(x) \geq 0$ pour déterminer quand la fonction $A'$ est positive.
\begin{align*}
A(x) & \geq 0 \\
- 4x + 30 & \geq 0 \\
- 4x + 30 + - 30 &\geq 0 + - 30 \\
- 4x &\geq - 30 \\
\frac{- 4x}{- 4} &\leq \frac{- 30}{- 4} \\
x &\leq \dfrac{15}{2} \\
\end{align*}
Donc $A(x)$ est positif quand $x$ est plus \textbf{petit} que $\dfrac{15}{2}$
\item
\begin{center}
\begin{tikzpicture}
\tkzTabInit[lgt=3,espcl=4]{$x$/1,Signe de $f'(x)$/2, Variations de $f(x)$/2}{, $\dfrac{15}{2}$ ,}%
\tkzTabLine{, +, z, -, }
\tkzTabVar{-/ ,+/$f(\dfrac{15}{2}) = \dfrac{450}{4}$ , -/}%
\end{tikzpicture}
\end{center}
\end{itemize}
Pour avoir le plus grand enclos, il faut placer les poteaux à 6m du mur et on aura alors un enclos de $72m^2$.
\end{solution}
\end{document}
%%% Local Variables:
%%% mode: latex
%%% TeX-master: "master"
%%% End:

Binary file not shown.

View File

@ -0,0 +1,158 @@
\documentclass[a4paper,12pt]{article}
\usepackage{myXsim}
\usepackage{pgfplots}
\usetikzlibrary{decorations.markings}
\pgfplotsset{compat=1.18}
\title{ DM1 \hfill BOUAFIA Lina}
\tribe{1ST}
\date{A rendre pour le lundi 27 mars 2023}
\duree{}
\xsimsetup{
solution/print = true
}
\pagestyle{empty}
\begin{document}
\maketitle
Le barème est donné à titre indicatif, il pourra être modifié.
\begin{exercise}[subtitle={Polynôme de degré 2}]
Développer les expressions suivantes pour vérifier que ce sont des polynômes de degré 2. Vous préciserez les valeurs de $a$, $b$ et $c$.
\begin{multicols}{2}
\begin{enumerate}
\item $f(x) = (4x - 9)(- 8x - 9)$
\item $g(x) = (5x - 4)^{2}$
\item $h(x) = - 1 + x(- 9x + 8)$
\item $i(x) = 9x^{2} + x(2x + 10)$
\item $j(x) = 9(x - 8)(x + 5)$
\item $k(x) = - 9(x - 6)(x + 7)$
\end{enumerate}
\end{multicols}
\end{exercise}
\begin{solution}
\begin{enumerate}
\item
\begin{align*}
f(x) &= (4x - 9)(- 8x - 9)\\&= 4x \times - 8x + 4x(- 9) - 9 \times - 8x - 9(- 9)\\&= 4(- 8) \times x^{1 + 1} - 9 \times 4 \times x - 9(- 8) \times x + 81\\&= - 36x + 72x - 32x^{2} + 81\\&= (- 36 + 72) \times x - 32x^{2} + 81\\&= - 32x^{2} + 36x + 81
\end{align*}
C'est un polynôme de degré 2 avec $a = - 32$, $b = 36$ et $c = 81$.
\item
\begin{align*}
g(x) &= (5x - 4)^{2}\\&= (5x - 4)(5x - 4)\\&= 5x \times 5x + 5x(- 4) - 4 \times 5x - 4(- 4)\\&= 5 \times 5 \times x^{1 + 1} - 4 \times 5 \times x - 4 \times 5 \times x + 16\\&= - 20x - 20x + 25x^{2} + 16\\&= (- 20 - 20) \times x + 25x^{2} + 16\\&= 25x^{2} - 40x + 16
\end{align*}
C'est un polynôme de degré 2 avec $a = 25$, $b = - 40$ et $c = 16$.
\item
\begin{align*}
h(x) &= - 1 + x(- 9x + 8)\\&= - 1 + x \times - 9x + x \times 8\\&= - 9x^{2} + 8x - 1
\end{align*}
C'est un polynôme de degré 2 avec $a = - 9$, $b = 8$ et $c = - 1$.
\item
\begin{align*}
i(x) &= 9x^{2} + x(2x + 10)\\&= 9x^{2} + x \times 2x + x \times 10\\&= 9x^{2} + 2x^{2} + 10x\\&= 9x^{2} + 2x^{2} + 10x\\&= (9 + 2) \times x^{2} + 10x\\&= 11x^{2} + 10x
\end{align*}
C'est un polynôme de degré 2 avec $a = 11$, $b = 10$ et $c = 0$.
\item
\begin{align*}
j(x) &= 9(x - 8)(x + 5)\\&= (9x + 9(- 8))(x + 5)\\&= (9x - 72)(x + 5)\\&= 9x \times x + 9x \times 5 - 72x - 72 \times 5\\&= 5 \times 9 \times x - 360 + 9x^{2} - 72x\\&= 45x - 360 + 9x^{2} - 72x\\&= 9x^{2} + 45x - 72x - 360\\&= 9x^{2} + (45 - 72) \times x - 360\\&= 9x^{2} - 27x - 360
\end{align*}
C'est un polynôme de degré 2 avec $a = 9$, $b = - 27$ et $c = - 360$.
\item
\begin{align*}
k(x) &= - 9(x - 6)(x + 7)\\&= (- 9x - 9(- 6))(x + 7)\\&= (- 9x + 54)(x + 7)\\&= - 9x \times x - 9x \times 7 + 54x + 54 \times 7\\&= 7(- 9) \times x + 378 - 9x^{2} + 54x\\&= - 63x + 378 - 9x^{2} + 54x\\&= - 9x^{2} - 63x + 54x + 378\\&= - 9x^{2} + (- 63 + 54) \times x + 378\\&= - 9x^{2} - 9x + 378
\end{align*}
C'est un polynôme de degré 2 avec $a = - 9$, $b = - 9$ et $c = 378$.
\end{enumerate}
\end{solution}
\begin{exercise}[subtitle={Étude de fonctions}]
Soit $f(x) = 5x^{2} - 45x + 90$ une fonction définie sur $\R$.
\begin{enumerate}
\item Calculer les valeurs suivantes
\[
f(1) \qquad f(-2)
\]
\item Dériver la fonction $f$
\item Étudier le signe de $f'$ puis en déduire les variations de $f$.
\item Est-ce que $f$ admet un maximum? un minimum? Calculer sa valeur.
\end{enumerate}
\end{exercise}
\begin{solution}
\begin{enumerate}
\item On remplace $x$ par les valeurs demandées
\[
f(1) = 5 \times 1^{2} - 45 \times 1 + 90=5 \times 1 - 45 + 90=5 + 45=50
\]
\[
f(-1) = 5 \times - 1^{2} - 45(- 1) + 90=5 \times 1 + 45 + 90=5 + 135=140
\]
\item Dérivation
\[
f'(x) = 10x - 45
\]
\item Pas de solutions automatiques.
\item Pas de solutions automatiques.
\end{enumerate}
\end{solution}
\begin{exercise}[subtitle={Enclos}]
Dans son garage, Jean a trouvé 37m de grillage. \\
Il décide de l'utiliser pour faire un enclos rectangulaire. Afin d'obtenir un enclos encore plus grand, il veut utiliser le mur du jardin qui formera un côté. Le grillage formera les 3 autres.
\begin{center}
\includegraphics[scale=0.8]{./fig/enclos}
\end{center}
Comment placer les poteaux pour avoir un enclos le plus grand possible ?
\textit{Cet exercice est un exercice de recherche. Vous êtes invité à utiliser les outils que vous voulez. Une part importante de la note sera dédiée à la rédaction et aux explications de ce que vous faites. Vous êtes encouragé à faire des schémas.}
\textit{Si vous utilisez le tableur, vous devrez le mentionner et m'envoyer le tableur à l'adresse \url{benjamin.bertrand@ac-lyon.fr}}
\end{exercise}
\begin{solution}
Formule de l'aire (en notant $x$ la longueur du côté de l'enclos à côté du mur)
\[
A(x) = x(37 - 2x) = - 2x^{2} + 37x
\]
On va donc étudier les variations de la fonction $A(x) = - 2x^{2} + 37x$
\begin{itemize}
\item Fonction dérivée : $A'(x) = - 4x + 37$
\item On résout l'inéquation $A'(x) \geq 0$ pour déterminer quand la fonction $A'$ est positive.
\begin{align*}
A(x) & \geq 0 \\
- 4x + 37 & \geq 0 \\
- 4x + 37 + - 37 &\geq 0 + - 37 \\
- 4x &\geq - 37 \\
\frac{- 4x}{- 4} &\leq \frac{- 37}{- 4} \\
x &\leq \dfrac{37}{4} \\
\end{align*}
Donc $A(x)$ est positif quand $x$ est plus \textbf{petit} que $\dfrac{37}{4}$
\item
\begin{center}
\begin{tikzpicture}
\tkzTabInit[lgt=3,espcl=4]{$x$/1,Signe de $f'(x)$/2, Variations de $f(x)$/2}{, $\dfrac{37}{4}$ ,}%
\tkzTabLine{, +, z, -, }
\tkzTabVar{-/ ,+/$f(\dfrac{37}{4}) = \dfrac{2738}{16}$ , -/}%
\end{tikzpicture}
\end{center}
\end{itemize}
Pour avoir le plus grand enclos, il faut placer les poteaux à 6m du mur et on aura alors un enclos de $72m^2$.
\end{solution}
\end{document}
%%% Local Variables:
%%% mode: latex
%%% TeX-master: "master"
%%% End:

Binary file not shown.

View File

@ -0,0 +1,158 @@
\documentclass[a4paper,12pt]{article}
\usepackage{myXsim}
\usepackage{pgfplots}
\usetikzlibrary{decorations.markings}
\pgfplotsset{compat=1.18}
\title{ DM1 \hfill BOUQUARD James}
\tribe{1ST}
\date{A rendre pour le lundi 27 mars 2023}
\duree{}
\xsimsetup{
solution/print = true
}
\pagestyle{empty}
\begin{document}
\maketitle
Le barème est donné à titre indicatif, il pourra être modifié.
\begin{exercise}[subtitle={Polynôme de degré 2}]
Développer les expressions suivantes pour vérifier que ce sont des polynômes de degré 2. Vous préciserez les valeurs de $a$, $b$ et $c$.
\begin{multicols}{2}
\begin{enumerate}
\item $f(x) = (3x - 1)(9x - 1)$
\item $g(x) = (- 2x + 4)^{2}$
\item $h(x) = 2 + x(5x + 8)$
\item $i(x) = - 1x^{2} + x(5x - 7)$
\item $j(x) = - 8(x + 5)(x + 4)$
\item $k(x) = 2(x + 8)(x + 3)$
\end{enumerate}
\end{multicols}
\end{exercise}
\begin{solution}
\begin{enumerate}
\item
\begin{align*}
f(x) &= (3x - 1)(9x - 1)\\&= 3x \times 9x + 3x(- 1) - 1 \times 9x - 1(- 1)\\&= 3 \times 9 \times x^{1 + 1} - 1 \times 3 \times x - 1 \times 9 \times x + 1\\&= - 3x - 9x + 27x^{2} + 1\\&= (- 3 - 9) \times x + 27x^{2} + 1\\&= 27x^{2} - 12x + 1
\end{align*}
C'est un polynôme de degré 2 avec $a = 27$, $b = - 12$ et $c = 1$.
\item
\begin{align*}
g(x) &= (- 2x + 4)^{2}\\&= (- 2x + 4)(- 2x + 4)\\&= - 2x \times - 2x - 2x \times 4 + 4 \times - 2x + 4 \times 4\\&= - 2(- 2) \times x^{1 + 1} + 4(- 2) \times x + 4(- 2) \times x + 16\\&= - 8x - 8x + 4x^{2} + 16\\&= (- 8 - 8) \times x + 4x^{2} + 16\\&= 4x^{2} - 16x + 16
\end{align*}
C'est un polynôme de degré 2 avec $a = 4$, $b = - 16$ et $c = 16$.
\item
\begin{align*}
h(x) &= 2 + x(5x + 8)\\&= 2 + x \times 5x + x \times 8\\&= 5x^{2} + 8x + 2
\end{align*}
C'est un polynôme de degré 2 avec $a = 5$, $b = 8$ et $c = 2$.
\item
\begin{align*}
i(x) &= - 1x^{2} + x(5x - 7)\\&= - x^{2} + x \times 5x + x(- 7)\\&= - x^{2} + 5x^{2} - 7x\\&= - x^{2} + 5x^{2} - 7x\\&= (- 1 + 5) \times x^{2} - 7x\\&= 4x^{2} - 7x
\end{align*}
C'est un polynôme de degré 2 avec $a = 4$, $b = - 7$ et $c = 0$.
\item
\begin{align*}
j(x) &= - 8(x + 5)(x + 4)\\&= (- 8x - 8 \times 5)(x + 4)\\&= (- 8x - 40)(x + 4)\\&= - 8x \times x - 8x \times 4 - 40x - 40 \times 4\\&= 4(- 8) \times x - 160 - 8x^{2} - 40x\\&= - 32x - 160 - 8x^{2} - 40x\\&= - 8x^{2} - 32x - 40x - 160\\&= - 8x^{2} + (- 32 - 40) \times x - 160\\&= - 8x^{2} - 72x - 160
\end{align*}
C'est un polynôme de degré 2 avec $a = - 8$, $b = - 72$ et $c = - 160$.
\item
\begin{align*}
k(x) &= 2(x + 8)(x + 3)\\&= (2x + 2 \times 8)(x + 3)\\&= (2x + 16)(x + 3)\\&= 2x \times x + 2x \times 3 + 16x + 16 \times 3\\&= 3 \times 2 \times x + 48 + 2x^{2} + 16x\\&= 6x + 48 + 2x^{2} + 16x\\&= 2x^{2} + 6x + 16x + 48\\&= 2x^{2} + (6 + 16) \times x + 48\\&= 2x^{2} + 22x + 48
\end{align*}
C'est un polynôme de degré 2 avec $a = 2$, $b = 22$ et $c = 48$.
\end{enumerate}
\end{solution}
\begin{exercise}[subtitle={Étude de fonctions}]
Soit $f(x) = 5x^{2} - 15x - 140$ une fonction définie sur $\R$.
\begin{enumerate}
\item Calculer les valeurs suivantes
\[
f(1) \qquad f(-2)
\]
\item Dériver la fonction $f$
\item Étudier le signe de $f'$ puis en déduire les variations de $f$.
\item Est-ce que $f$ admet un maximum? un minimum? Calculer sa valeur.
\end{enumerate}
\end{exercise}
\begin{solution}
\begin{enumerate}
\item On remplace $x$ par les valeurs demandées
\[
f(1) = 5 \times 1^{2} - 15 \times 1 - 140=5 \times 1 - 15 - 140=5 - 155=- 150
\]
\[
f(-1) = 5 \times - 1^{2} - 15(- 1) - 140=5 \times 1 + 15 - 140=5 - 125=- 120
\]
\item Dérivation
\[
f'(x) = 10x - 15
\]
\item Pas de solutions automatiques.
\item Pas de solutions automatiques.
\end{enumerate}
\end{solution}
\begin{exercise}[subtitle={Enclos}]
Dans son garage, Jean a trouvé 21m de grillage. \\
Il décide de l'utiliser pour faire un enclos rectangulaire. Afin d'obtenir un enclos encore plus grand, il veut utiliser le mur du jardin qui formera un côté. Le grillage formera les 3 autres.
\begin{center}
\includegraphics[scale=0.8]{./fig/enclos}
\end{center}
Comment placer les poteaux pour avoir un enclos le plus grand possible ?
\textit{Cet exercice est un exercice de recherche. Vous êtes invité à utiliser les outils que vous voulez. Une part importante de la note sera dédiée à la rédaction et aux explications de ce que vous faites. Vous êtes encouragé à faire des schémas.}
\textit{Si vous utilisez le tableur, vous devrez le mentionner et m'envoyer le tableur à l'adresse \url{benjamin.bertrand@ac-lyon.fr}}
\end{exercise}
\begin{solution}
Formule de l'aire (en notant $x$ la longueur du côté de l'enclos à côté du mur)
\[
A(x) = x(21 - 2x) = - 2x^{2} + 21x
\]
On va donc étudier les variations de la fonction $A(x) = - 2x^{2} + 21x$
\begin{itemize}
\item Fonction dérivée : $A'(x) = - 4x + 21$
\item On résout l'inéquation $A'(x) \geq 0$ pour déterminer quand la fonction $A'$ est positive.
\begin{align*}
A(x) & \geq 0 \\
- 4x + 21 & \geq 0 \\
- 4x + 21 + - 21 &\geq 0 + - 21 \\
- 4x &\geq - 21 \\
\frac{- 4x}{- 4} &\leq \frac{- 21}{- 4} \\
x &\leq \dfrac{21}{4} \\
\end{align*}
Donc $A(x)$ est positif quand $x$ est plus \textbf{petit} que $\dfrac{21}{4}$
\item
\begin{center}
\begin{tikzpicture}
\tkzTabInit[lgt=3,espcl=4]{$x$/1,Signe de $f'(x)$/2, Variations de $f(x)$/2}{, $\dfrac{21}{4}$ ,}%
\tkzTabLine{, +, z, -, }
\tkzTabVar{-/ ,+/$f(\dfrac{21}{4}) = \dfrac{882}{16}$ , -/}%
\end{tikzpicture}
\end{center}
\end{itemize}
Pour avoir le plus grand enclos, il faut placer les poteaux à 6m du mur et on aura alors un enclos de $72m^2$.
\end{solution}
\end{document}
%%% Local Variables:
%%% mode: latex
%%% TeX-master: "master"
%%% End:

Binary file not shown.

View File

@ -0,0 +1,158 @@
\documentclass[a4paper,12pt]{article}
\usepackage{myXsim}
\usepackage{pgfplots}
\usetikzlibrary{decorations.markings}
\pgfplotsset{compat=1.18}
\title{ DM1 \hfill DA COSTA QUEIROGA Délinda}
\tribe{1ST}
\date{A rendre pour le lundi 27 mars 2023}
\duree{}
\xsimsetup{
solution/print = true
}
\pagestyle{empty}
\begin{document}
\maketitle
Le barème est donné à titre indicatif, il pourra être modifié.
\begin{exercise}[subtitle={Polynôme de degré 2}]
Développer les expressions suivantes pour vérifier que ce sont des polynômes de degré 2. Vous préciserez les valeurs de $a$, $b$ et $c$.
\begin{multicols}{2}
\begin{enumerate}
\item $f(x) = (2x - 2)(3x - 2)$
\item $g(x) = (- 6x + 5)^{2}$
\item $h(x) = 10 + x(4x + 9)$
\item $i(x) = 4x^{2} + x(- 10x - 8)$
\item $j(x) = 7(x + 3)(x - 6)$
\item $k(x) = - 5(x + 3)(x - 3)$
\end{enumerate}
\end{multicols}
\end{exercise}
\begin{solution}
\begin{enumerate}
\item
\begin{align*}
f(x) &= (2x - 2)(3x - 2)\\&= 2x \times 3x + 2x(- 2) - 2 \times 3x - 2(- 2)\\&= 2 \times 3 \times x^{1 + 1} - 2 \times 2 \times x - 2 \times 3 \times x + 4\\&= - 4x - 6x + 6x^{2} + 4\\&= (- 4 - 6) \times x + 6x^{2} + 4\\&= 6x^{2} - 10x + 4
\end{align*}
C'est un polynôme de degré 2 avec $a = 6$, $b = - 10$ et $c = 4$.
\item
\begin{align*}
g(x) &= (- 6x + 5)^{2}\\&= (- 6x + 5)(- 6x + 5)\\&= - 6x \times - 6x - 6x \times 5 + 5 \times - 6x + 5 \times 5\\&= - 6(- 6) \times x^{1 + 1} + 5(- 6) \times x + 5(- 6) \times x + 25\\&= - 30x - 30x + 36x^{2} + 25\\&= (- 30 - 30) \times x + 36x^{2} + 25\\&= 36x^{2} - 60x + 25
\end{align*}
C'est un polynôme de degré 2 avec $a = 36$, $b = - 60$ et $c = 25$.
\item
\begin{align*}
h(x) &= 10 + x(4x + 9)\\&= 10 + x \times 4x + x \times 9\\&= 4x^{2} + 9x + 10
\end{align*}
C'est un polynôme de degré 2 avec $a = 4$, $b = 9$ et $c = 10$.
\item
\begin{align*}
i(x) &= 4x^{2} + x(- 10x - 8)\\&= 4x^{2} + x \times - 10x + x(- 8)\\&= 4x^{2} - 10x^{2} - 8x\\&= 4x^{2} - 10x^{2} - 8x\\&= (4 - 10) \times x^{2} - 8x\\&= - 6x^{2} - 8x
\end{align*}
C'est un polynôme de degré 2 avec $a = - 6$, $b = - 8$ et $c = 0$.
\item
\begin{align*}
j(x) &= 7(x + 3)(x - 6)\\&= (7x + 7 \times 3)(x - 6)\\&= (7x + 21)(x - 6)\\&= 7x \times x + 7x(- 6) + 21x + 21(- 6)\\&= - 6 \times 7 \times x - 126 + 7x^{2} + 21x\\&= - 42x - 126 + 7x^{2} + 21x\\&= 7x^{2} - 42x + 21x - 126\\&= 7x^{2} + (- 42 + 21) \times x - 126\\&= 7x^{2} - 21x - 126
\end{align*}
C'est un polynôme de degré 2 avec $a = 7$, $b = - 21$ et $c = - 126$.
\item
\begin{align*}
k(x) &= - 5(x + 3)(x - 3)\\&= (- 5x - 5 \times 3)(x - 3)\\&= (- 5x - 15)(x - 3)\\&= - 5x \times x - 5x(- 3) - 15x - 15(- 3)\\&= - 3(- 5) \times x + 45 - 5x^{2} - 15x\\&= 15x + 45 - 5x^{2} - 15x\\&= - 5x^{2} + 15x - 15x + 45\\&= - 5x^{2} + (15 - 15) \times x + 45\\&= - 5x^{2} + 45
\end{align*}
C'est un polynôme de degré 2 avec $a = - 5$, $b = 0$ et $c = 45$.
\end{enumerate}
\end{solution}
\begin{exercise}[subtitle={Étude de fonctions}]
Soit $f(x) = - 7x^{2} + 91x - 280$ une fonction définie sur $\R$.
\begin{enumerate}
\item Calculer les valeurs suivantes
\[
f(1) \qquad f(-2)
\]
\item Dériver la fonction $f$
\item Étudier le signe de $f'$ puis en déduire les variations de $f$.
\item Est-ce que $f$ admet un maximum? un minimum? Calculer sa valeur.
\end{enumerate}
\end{exercise}
\begin{solution}
\begin{enumerate}
\item On remplace $x$ par les valeurs demandées
\[
f(1) = - 7 \times 1^{2} + 91 \times 1 - 280=- 7 \times 1 + 91 - 280=- 7 - 189=- 196
\]
\[
f(-1) = - 7 \times - 1^{2} + 91(- 1) - 280=- 7 \times 1 - 91 - 280=- 7 - 371=- 378
\]
\item Dérivation
\[
f'(x) = - 14x + 91
\]
\item Pas de solutions automatiques.
\item Pas de solutions automatiques.
\end{enumerate}
\end{solution}
\begin{exercise}[subtitle={Enclos}]
Dans son garage, Jean a trouvé 23m de grillage. \\
Il décide de l'utiliser pour faire un enclos rectangulaire. Afin d'obtenir un enclos encore plus grand, il veut utiliser le mur du jardin qui formera un côté. Le grillage formera les 3 autres.
\begin{center}
\includegraphics[scale=0.8]{./fig/enclos}
\end{center}
Comment placer les poteaux pour avoir un enclos le plus grand possible ?
\textit{Cet exercice est un exercice de recherche. Vous êtes invité à utiliser les outils que vous voulez. Une part importante de la note sera dédiée à la rédaction et aux explications de ce que vous faites. Vous êtes encouragé à faire des schémas.}
\textit{Si vous utilisez le tableur, vous devrez le mentionner et m'envoyer le tableur à l'adresse \url{benjamin.bertrand@ac-lyon.fr}}
\end{exercise}
\begin{solution}
Formule de l'aire (en notant $x$ la longueur du côté de l'enclos à côté du mur)
\[
A(x) = x(23 - 2x) = - 2x^{2} + 23x
\]
On va donc étudier les variations de la fonction $A(x) = - 2x^{2} + 23x$
\begin{itemize}
\item Fonction dérivée : $A'(x) = - 4x + 23$
\item On résout l'inéquation $A'(x) \geq 0$ pour déterminer quand la fonction $A'$ est positive.
\begin{align*}
A(x) & \geq 0 \\
- 4x + 23 & \geq 0 \\
- 4x + 23 + - 23 &\geq 0 + - 23 \\
- 4x &\geq - 23 \\
\frac{- 4x}{- 4} &\leq \frac{- 23}{- 4} \\
x &\leq \dfrac{23}{4} \\
\end{align*}
Donc $A(x)$ est positif quand $x$ est plus \textbf{petit} que $\dfrac{23}{4}$
\item
\begin{center}
\begin{tikzpicture}
\tkzTabInit[lgt=3,espcl=4]{$x$/1,Signe de $f'(x)$/2, Variations de $f(x)$/2}{, $\dfrac{23}{4}$ ,}%
\tkzTabLine{, +, z, -, }
\tkzTabVar{-/ ,+/$f(\dfrac{23}{4}) = \dfrac{1058}{16}$ , -/}%
\end{tikzpicture}
\end{center}
\end{itemize}
Pour avoir le plus grand enclos, il faut placer les poteaux à 6m du mur et on aura alors un enclos de $72m^2$.
\end{solution}
\end{document}
%%% Local Variables:
%%% mode: latex
%%% TeX-master: "master"
%%% End:

Binary file not shown.

View File

@ -0,0 +1,158 @@
\documentclass[a4paper,12pt]{article}
\usepackage{myXsim}
\usepackage{pgfplots}
\usetikzlibrary{decorations.markings}
\pgfplotsset{compat=1.18}
\title{ DM1 \hfill GNUI Kadia}
\tribe{1ST}
\date{A rendre pour le lundi 27 mars 2023}
\duree{}
\xsimsetup{
solution/print = true
}
\pagestyle{empty}
\begin{document}
\maketitle
Le barème est donné à titre indicatif, il pourra être modifié.
\begin{exercise}[subtitle={Polynôme de degré 2}]
Développer les expressions suivantes pour vérifier que ce sont des polynômes de degré 2. Vous préciserez les valeurs de $a$, $b$ et $c$.
\begin{multicols}{2}
\begin{enumerate}
\item $f(x) = (- 4x - 1)(- 8x - 1)$
\item $g(x) = (7x + 8)^{2}$
\item $h(x) = 3 + x(7x + 7)$
\item $i(x) = - 9x^{2} + x(- 3x + 10)$
\item $j(x) = - 5(x + 7)(x + 5)$
\item $k(x) = - 4(x - 6)(x + 2)$
\end{enumerate}
\end{multicols}
\end{exercise}
\begin{solution}
\begin{enumerate}
\item
\begin{align*}
f(x) &= (- 4x - 1)(- 8x - 1)\\&= - 4x \times - 8x - 4x(- 1) - 1 \times - 8x - 1(- 1)\\&= - 4(- 8) \times x^{1 + 1} - 1(- 4) \times x - 1(- 8) \times x + 1\\&= 4x + 8x + 32x^{2} + 1\\&= (4 + 8) \times x + 32x^{2} + 1\\&= 32x^{2} + 12x + 1
\end{align*}
C'est un polynôme de degré 2 avec $a = 32$, $b = 12$ et $c = 1$.
\item
\begin{align*}
g(x) &= (7x + 8)^{2}\\&= (7x + 8)(7x + 8)\\&= 7x \times 7x + 7x \times 8 + 8 \times 7x + 8 \times 8\\&= 7 \times 7 \times x^{1 + 1} + 8 \times 7 \times x + 8 \times 7 \times x + 64\\&= 56x + 56x + 49x^{2} + 64\\&= (56 + 56) \times x + 49x^{2} + 64\\&= 49x^{2} + 112x + 64
\end{align*}
C'est un polynôme de degré 2 avec $a = 49$, $b = 112$ et $c = 64$.
\item
\begin{align*}
h(x) &= 3 + x(7x + 7)\\&= 3 + x \times 7x + x \times 7\\&= 7x^{2} + 7x + 3
\end{align*}
C'est un polynôme de degré 2 avec $a = 7$, $b = 7$ et $c = 3$.
\item
\begin{align*}
i(x) &= - 9x^{2} + x(- 3x + 10)\\&= - 9x^{2} + x \times - 3x + x \times 10\\&= - 9x^{2} - 3x^{2} + 10x\\&= - 9x^{2} - 3x^{2} + 10x\\&= (- 9 - 3) \times x^{2} + 10x\\&= - 12x^{2} + 10x
\end{align*}
C'est un polynôme de degré 2 avec $a = - 12$, $b = 10$ et $c = 0$.
\item
\begin{align*}
j(x) &= - 5(x + 7)(x + 5)\\&= (- 5x - 5 \times 7)(x + 5)\\&= (- 5x - 35)(x + 5)\\&= - 5x \times x - 5x \times 5 - 35x - 35 \times 5\\&= 5(- 5) \times x - 175 - 5x^{2} - 35x\\&= - 25x - 175 - 5x^{2} - 35x\\&= - 5x^{2} - 25x - 35x - 175\\&= - 5x^{2} + (- 25 - 35) \times x - 175\\&= - 5x^{2} - 60x - 175
\end{align*}
C'est un polynôme de degré 2 avec $a = - 5$, $b = - 60$ et $c = - 175$.
\item
\begin{align*}
k(x) &= - 4(x - 6)(x + 2)\\&= (- 4x - 4(- 6))(x + 2)\\&= (- 4x + 24)(x + 2)\\&= - 4x \times x - 4x \times 2 + 24x + 24 \times 2\\&= 2(- 4) \times x + 48 - 4x^{2} + 24x\\&= - 8x + 48 - 4x^{2} + 24x\\&= - 4x^{2} - 8x + 24x + 48\\&= - 4x^{2} + (- 8 + 24) \times x + 48\\&= - 4x^{2} + 16x + 48
\end{align*}
C'est un polynôme de degré 2 avec $a = - 4$, $b = 16$ et $c = 48$.
\end{enumerate}
\end{solution}
\begin{exercise}[subtitle={Étude de fonctions}]
Soit $f(x) = - 3x^{2} - 33x - 30$ une fonction définie sur $\R$.
\begin{enumerate}
\item Calculer les valeurs suivantes
\[
f(1) \qquad f(-2)
\]
\item Dériver la fonction $f$
\item Étudier le signe de $f'$ puis en déduire les variations de $f$.
\item Est-ce que $f$ admet un maximum? un minimum? Calculer sa valeur.
\end{enumerate}
\end{exercise}
\begin{solution}
\begin{enumerate}
\item On remplace $x$ par les valeurs demandées
\[
f(1) = - 3 \times 1^{2} - 33 \times 1 - 30=- 3 \times 1 - 33 - 30=- 3 - 63=- 66
\]
\[
f(-1) = - 3 \times - 1^{2} - 33(- 1) - 30=- 3 \times 1 + 33 - 30=- 3 + 3=0
\]
\item Dérivation
\[
f'(x) = - 6x - 33
\]
\item Pas de solutions automatiques.
\item Pas de solutions automatiques.
\end{enumerate}
\end{solution}
\begin{exercise}[subtitle={Enclos}]
Dans son garage, Jean a trouvé 35m de grillage. \\
Il décide de l'utiliser pour faire un enclos rectangulaire. Afin d'obtenir un enclos encore plus grand, il veut utiliser le mur du jardin qui formera un côté. Le grillage formera les 3 autres.
\begin{center}
\includegraphics[scale=0.8]{./fig/enclos}
\end{center}
Comment placer les poteaux pour avoir un enclos le plus grand possible ?
\textit{Cet exercice est un exercice de recherche. Vous êtes invité à utiliser les outils que vous voulez. Une part importante de la note sera dédiée à la rédaction et aux explications de ce que vous faites. Vous êtes encouragé à faire des schémas.}
\textit{Si vous utilisez le tableur, vous devrez le mentionner et m'envoyer le tableur à l'adresse \url{benjamin.bertrand@ac-lyon.fr}}
\end{exercise}
\begin{solution}
Formule de l'aire (en notant $x$ la longueur du côté de l'enclos à côté du mur)
\[
A(x) = x(35 - 2x) = - 2x^{2} + 35x
\]
On va donc étudier les variations de la fonction $A(x) = - 2x^{2} + 35x$
\begin{itemize}
\item Fonction dérivée : $A'(x) = - 4x + 35$
\item On résout l'inéquation $A'(x) \geq 0$ pour déterminer quand la fonction $A'$ est positive.
\begin{align*}
A(x) & \geq 0 \\
- 4x + 35 & \geq 0 \\
- 4x + 35 + - 35 &\geq 0 + - 35 \\
- 4x &\geq - 35 \\
\frac{- 4x}{- 4} &\leq \frac{- 35}{- 4} \\
x &\leq \dfrac{35}{4} \\
\end{align*}
Donc $A(x)$ est positif quand $x$ est plus \textbf{petit} que $\dfrac{35}{4}$
\item
\begin{center}
\begin{tikzpicture}
\tkzTabInit[lgt=3,espcl=4]{$x$/1,Signe de $f'(x)$/2, Variations de $f(x)$/2}{, $\dfrac{35}{4}$ ,}%
\tkzTabLine{, +, z, -, }
\tkzTabVar{-/ ,+/$f(\dfrac{35}{4}) = \dfrac{2450}{16}$ , -/}%
\end{tikzpicture}
\end{center}
\end{itemize}
Pour avoir le plus grand enclos, il faut placer les poteaux à 6m du mur et on aura alors un enclos de $72m^2$.
\end{solution}
\end{document}
%%% Local Variables:
%%% mode: latex
%%% TeX-master: "master"
%%% End:

Binary file not shown.

View File

@ -0,0 +1,158 @@
\documentclass[a4paper,12pt]{article}
\usepackage{myXsim}
\usepackage{pgfplots}
\usetikzlibrary{decorations.markings}
\pgfplotsset{compat=1.18}
\title{ DM1 \hfill HAMMOUDI Lyna}
\tribe{1ST}
\date{A rendre pour le lundi 27 mars 2023}
\duree{}
\xsimsetup{
solution/print = true
}
\pagestyle{empty}
\begin{document}
\maketitle
Le barème est donné à titre indicatif, il pourra être modifié.
\begin{exercise}[subtitle={Polynôme de degré 2}]
Développer les expressions suivantes pour vérifier que ce sont des polynômes de degré 2. Vous préciserez les valeurs de $a$, $b$ et $c$.
\begin{multicols}{2}
\begin{enumerate}
\item $f(x) = (- 7x + 10)(- 8x + 10)$
\item $g(x) = (6x - 2)^{2}$
\item $h(x) = 2 + x(- 5x + 4)$
\item $i(x) = - 9x^{2} + x(3x - 2)$
\item $j(x) = 6(x + 9)(x + 10)$
\item $k(x) = - 8(x - 10)(x - 4)$
\end{enumerate}
\end{multicols}
\end{exercise}
\begin{solution}
\begin{enumerate}
\item
\begin{align*}
f(x) &= (- 7x + 10)(- 8x + 10)\\&= - 7x \times - 8x - 7x \times 10 + 10 \times - 8x + 10 \times 10\\&= - 7(- 8) \times x^{1 + 1} + 10(- 7) \times x + 10(- 8) \times x + 100\\&= - 70x - 80x + 56x^{2} + 100\\&= (- 70 - 80) \times x + 56x^{2} + 100\\&= 56x^{2} - 150x + 100
\end{align*}
C'est un polynôme de degré 2 avec $a = 56$, $b = - 150$ et $c = 100$.
\item
\begin{align*}
g(x) &= (6x - 2)^{2}\\&= (6x - 2)(6x - 2)\\&= 6x \times 6x + 6x(- 2) - 2 \times 6x - 2(- 2)\\&= 6 \times 6 \times x^{1 + 1} - 2 \times 6 \times x - 2 \times 6 \times x + 4\\&= - 12x - 12x + 36x^{2} + 4\\&= (- 12 - 12) \times x + 36x^{2} + 4\\&= 36x^{2} - 24x + 4
\end{align*}
C'est un polynôme de degré 2 avec $a = 36$, $b = - 24$ et $c = 4$.
\item
\begin{align*}
h(x) &= 2 + x(- 5x + 4)\\&= 2 + x \times - 5x + x \times 4\\&= - 5x^{2} + 4x + 2
\end{align*}
C'est un polynôme de degré 2 avec $a = - 5$, $b = 4$ et $c = 2$.
\item
\begin{align*}
i(x) &= - 9x^{2} + x(3x - 2)\\&= - 9x^{2} + x \times 3x + x(- 2)\\&= - 9x^{2} + 3x^{2} - 2x\\&= - 9x^{2} + 3x^{2} - 2x\\&= (- 9 + 3) \times x^{2} - 2x\\&= - 6x^{2} - 2x
\end{align*}
C'est un polynôme de degré 2 avec $a = - 6$, $b = - 2$ et $c = 0$.
\item
\begin{align*}
j(x) &= 6(x + 9)(x + 10)\\&= (6x + 6 \times 9)(x + 10)\\&= (6x + 54)(x + 10)\\&= 6x \times x + 6x \times 10 + 54x + 54 \times 10\\&= 10 \times 6 \times x + 540 + 6x^{2} + 54x\\&= 60x + 540 + 6x^{2} + 54x\\&= 6x^{2} + 60x + 54x + 540\\&= 6x^{2} + (60 + 54) \times x + 540\\&= 6x^{2} + 114x + 540
\end{align*}
C'est un polynôme de degré 2 avec $a = 6$, $b = 114$ et $c = 540$.
\item
\begin{align*}
k(x) &= - 8(x - 10)(x - 4)\\&= (- 8x - 8(- 10))(x - 4)\\&= (- 8x + 80)(x - 4)\\&= - 8x \times x - 8x(- 4) + 80x + 80(- 4)\\&= - 4(- 8) \times x - 320 - 8x^{2} + 80x\\&= 32x - 320 - 8x^{2} + 80x\\&= - 8x^{2} + 32x + 80x - 320\\&= - 8x^{2} + (32 + 80) \times x - 320\\&= - 8x^{2} + 112x - 320
\end{align*}
C'est un polynôme de degré 2 avec $a = - 8$, $b = 112$ et $c = - 320$.
\end{enumerate}
\end{solution}
\begin{exercise}[subtitle={Étude de fonctions}]
Soit $f(x) = - x^{2} - 2x + 48$ une fonction définie sur $\R$.
\begin{enumerate}
\item Calculer les valeurs suivantes
\[
f(1) \qquad f(-2)
\]
\item Dériver la fonction $f$
\item Étudier le signe de $f'$ puis en déduire les variations de $f$.
\item Est-ce que $f$ admet un maximum? un minimum? Calculer sa valeur.
\end{enumerate}
\end{exercise}
\begin{solution}
\begin{enumerate}
\item On remplace $x$ par les valeurs demandées
\[
f(1) = - 1^{2} - 2 \times 1 + 48=- 1 - 2 + 48=- 1 + 46=45
\]
\[
f(-1) = - - 1^{2} - 2(- 1) + 48=- 1 + 2 + 48=- 1 + 50=49
\]
\item Dérivation
\[
f'(x) = - 2x - 2
\]
\item Pas de solutions automatiques.
\item Pas de solutions automatiques.
\end{enumerate}
\end{solution}
\begin{exercise}[subtitle={Enclos}]
Dans son garage, Jean a trouvé 37m de grillage. \\
Il décide de l'utiliser pour faire un enclos rectangulaire. Afin d'obtenir un enclos encore plus grand, il veut utiliser le mur du jardin qui formera un côté. Le grillage formera les 3 autres.
\begin{center}
\includegraphics[scale=0.8]{./fig/enclos}
\end{center}
Comment placer les poteaux pour avoir un enclos le plus grand possible ?
\textit{Cet exercice est un exercice de recherche. Vous êtes invité à utiliser les outils que vous voulez. Une part importante de la note sera dédiée à la rédaction et aux explications de ce que vous faites. Vous êtes encouragé à faire des schémas.}
\textit{Si vous utilisez le tableur, vous devrez le mentionner et m'envoyer le tableur à l'adresse \url{benjamin.bertrand@ac-lyon.fr}}
\end{exercise}
\begin{solution}
Formule de l'aire (en notant $x$ la longueur du côté de l'enclos à côté du mur)
\[
A(x) = x(37 - 2x) = - 2x^{2} + 37x
\]
On va donc étudier les variations de la fonction $A(x) = - 2x^{2} + 37x$
\begin{itemize}
\item Fonction dérivée : $A'(x) = - 4x + 37$
\item On résout l'inéquation $A'(x) \geq 0$ pour déterminer quand la fonction $A'$ est positive.
\begin{align*}
A(x) & \geq 0 \\
- 4x + 37 & \geq 0 \\
- 4x + 37 + - 37 &\geq 0 + - 37 \\
- 4x &\geq - 37 \\
\frac{- 4x}{- 4} &\leq \frac{- 37}{- 4} \\
x &\leq \dfrac{37}{4} \\
\end{align*}
Donc $A(x)$ est positif quand $x$ est plus \textbf{petit} que $\dfrac{37}{4}$
\item
\begin{center}
\begin{tikzpicture}
\tkzTabInit[lgt=3,espcl=4]{$x$/1,Signe de $f'(x)$/2, Variations de $f(x)$/2}{, $\dfrac{37}{4}$ ,}%
\tkzTabLine{, +, z, -, }
\tkzTabVar{-/ ,+/$f(\dfrac{37}{4}) = \dfrac{2738}{16}$ , -/}%
\end{tikzpicture}
\end{center}
\end{itemize}
Pour avoir le plus grand enclos, il faut placer les poteaux à 6m du mur et on aura alors un enclos de $72m^2$.
\end{solution}
\end{document}
%%% Local Variables:
%%% mode: latex
%%% TeX-master: "master"
%%% End:

Binary file not shown.

View File

@ -0,0 +1,158 @@
\documentclass[a4paper,12pt]{article}
\usepackage{myXsim}
\usepackage{pgfplots}
\usetikzlibrary{decorations.markings}
\pgfplotsset{compat=1.18}
\title{ DM1 \hfill KITOUNI Zakaria}
\tribe{1ST}
\date{A rendre pour le lundi 27 mars 2023}
\duree{}
\xsimsetup{
solution/print = true
}
\pagestyle{empty}
\begin{document}
\maketitle
Le barème est donné à titre indicatif, il pourra être modifié.
\begin{exercise}[subtitle={Polynôme de degré 2}]
Développer les expressions suivantes pour vérifier que ce sont des polynômes de degré 2. Vous préciserez les valeurs de $a$, $b$ et $c$.
\begin{multicols}{2}
\begin{enumerate}
\item $f(x) = (- 7x - 6)(- 10x - 6)$
\item $g(x) = (4x + 2)^{2}$
\item $h(x) = 8 + x(- 9x + 7)$
\item $i(x) = 9x^{2} + x(9x - 5)$
\item $j(x) = - 1(x - 6)(x - 7)$
\item $k(x) = - 7(x - 1)(x + 3)$
\end{enumerate}
\end{multicols}
\end{exercise}
\begin{solution}
\begin{enumerate}
\item
\begin{align*}
f(x) &= (- 7x - 6)(- 10x - 6)\\&= - 7x \times - 10x - 7x(- 6) - 6 \times - 10x - 6(- 6)\\&= - 7(- 10) \times x^{1 + 1} - 6(- 7) \times x - 6(- 10) \times x + 36\\&= 42x + 60x + 70x^{2} + 36\\&= (42 + 60) \times x + 70x^{2} + 36\\&= 70x^{2} + 102x + 36
\end{align*}
C'est un polynôme de degré 2 avec $a = 70$, $b = 102$ et $c = 36$.
\item
\begin{align*}
g(x) &= (4x + 2)^{2}\\&= (4x + 2)(4x + 2)\\&= 4x \times 4x + 4x \times 2 + 2 \times 4x + 2 \times 2\\&= 4 \times 4 \times x^{1 + 1} + 2 \times 4 \times x + 2 \times 4 \times x + 4\\&= 8x + 8x + 16x^{2} + 4\\&= (8 + 8) \times x + 16x^{2} + 4\\&= 16x^{2} + 16x + 4
\end{align*}
C'est un polynôme de degré 2 avec $a = 16$, $b = 16$ et $c = 4$.
\item
\begin{align*}
h(x) &= 8 + x(- 9x + 7)\\&= 8 + x \times - 9x + x \times 7\\&= - 9x^{2} + 7x + 8
\end{align*}
C'est un polynôme de degré 2 avec $a = - 9$, $b = 7$ et $c = 8$.
\item
\begin{align*}
i(x) &= 9x^{2} + x(9x - 5)\\&= 9x^{2} + x \times 9x + x(- 5)\\&= 9x^{2} + 9x^{2} - 5x\\&= 9x^{2} + 9x^{2} - 5x\\&= (9 + 9) \times x^{2} - 5x\\&= 18x^{2} - 5x
\end{align*}
C'est un polynôme de degré 2 avec $a = 18$, $b = - 5$ et $c = 0$.
\item
\begin{align*}
j(x) &= - 1(x - 6)(x - 7)\\&= (- 1x - 1(- 6))(x - 7)\\&= (- x + 6)(x - 7)\\&= (- x) \times x + (- x)(- 7) + 6x + 6(- 7)\\&= - 7(- 1) \times x - 42 - x^{2} + 6x\\&= 7x - 42 - x^{2} + 6x\\&= - x^{2} + 7x + 6x - 42\\&= - x^{2} + (7 + 6) \times x - 42\\&= - x^{2} + 13x - 42
\end{align*}
C'est un polynôme de degré 2 avec $a = - 1$, $b = 13$ et $c = - 42$.
\item
\begin{align*}
k(x) &= - 7(x - 1)(x + 3)\\&= (- 7x - 7(- 1))(x + 3)\\&= (- 7x + 7)(x + 3)\\&= - 7x \times x - 7x \times 3 + 7x + 7 \times 3\\&= 3(- 7) \times x + 21 - 7x^{2} + 7x\\&= - 21x + 21 - 7x^{2} + 7x\\&= - 7x^{2} - 21x + 7x + 21\\&= - 7x^{2} + (- 21 + 7) \times x + 21\\&= - 7x^{2} - 14x + 21
\end{align*}
C'est un polynôme de degré 2 avec $a = - 7$, $b = - 14$ et $c = 21$.
\end{enumerate}
\end{solution}
\begin{exercise}[subtitle={Étude de fonctions}]
Soit $f(x) = 3x^{2} - 3x - 126$ une fonction définie sur $\R$.
\begin{enumerate}
\item Calculer les valeurs suivantes
\[
f(1) \qquad f(-2)
\]
\item Dériver la fonction $f$
\item Étudier le signe de $f'$ puis en déduire les variations de $f$.
\item Est-ce que $f$ admet un maximum? un minimum? Calculer sa valeur.
\end{enumerate}
\end{exercise}
\begin{solution}
\begin{enumerate}
\item On remplace $x$ par les valeurs demandées
\[
f(1) = 3 \times 1^{2} - 3 \times 1 - 126=3 \times 1 - 3 - 126=3 - 129=- 126
\]
\[
f(-1) = 3 \times - 1^{2} - 3(- 1) - 126=3 \times 1 + 3 - 126=3 - 123=- 120
\]
\item Dérivation
\[
f'(x) = 6x - 3
\]
\item Pas de solutions automatiques.
\item Pas de solutions automatiques.
\end{enumerate}
\end{solution}
\begin{exercise}[subtitle={Enclos}]
Dans son garage, Jean a trouvé 29m de grillage. \\
Il décide de l'utiliser pour faire un enclos rectangulaire. Afin d'obtenir un enclos encore plus grand, il veut utiliser le mur du jardin qui formera un côté. Le grillage formera les 3 autres.
\begin{center}
\includegraphics[scale=0.8]{./fig/enclos}
\end{center}
Comment placer les poteaux pour avoir un enclos le plus grand possible ?
\textit{Cet exercice est un exercice de recherche. Vous êtes invité à utiliser les outils que vous voulez. Une part importante de la note sera dédiée à la rédaction et aux explications de ce que vous faites. Vous êtes encouragé à faire des schémas.}
\textit{Si vous utilisez le tableur, vous devrez le mentionner et m'envoyer le tableur à l'adresse \url{benjamin.bertrand@ac-lyon.fr}}
\end{exercise}
\begin{solution}
Formule de l'aire (en notant $x$ la longueur du côté de l'enclos à côté du mur)
\[
A(x) = x(29 - 2x) = - 2x^{2} + 29x
\]
On va donc étudier les variations de la fonction $A(x) = - 2x^{2} + 29x$
\begin{itemize}
\item Fonction dérivée : $A'(x) = - 4x + 29$
\item On résout l'inéquation $A'(x) \geq 0$ pour déterminer quand la fonction $A'$ est positive.
\begin{align*}
A(x) & \geq 0 \\
- 4x + 29 & \geq 0 \\
- 4x + 29 + - 29 &\geq 0 + - 29 \\
- 4x &\geq - 29 \\
\frac{- 4x}{- 4} &\leq \frac{- 29}{- 4} \\
x &\leq \dfrac{29}{4} \\
\end{align*}
Donc $A(x)$ est positif quand $x$ est plus \textbf{petit} que $\dfrac{29}{4}$
\item
\begin{center}
\begin{tikzpicture}
\tkzTabInit[lgt=3,espcl=4]{$x$/1,Signe de $f'(x)$/2, Variations de $f(x)$/2}{, $\dfrac{29}{4}$ ,}%
\tkzTabLine{, +, z, -, }
\tkzTabVar{-/ ,+/$f(\dfrac{29}{4}) = \dfrac{1682}{16}$ , -/}%
\end{tikzpicture}
\end{center}
\end{itemize}
Pour avoir le plus grand enclos, il faut placer les poteaux à 6m du mur et on aura alors un enclos de $72m^2$.
\end{solution}
\end{document}
%%% Local Variables:
%%% mode: latex
%%% TeX-master: "master"
%%% End:

Binary file not shown.

View File

@ -0,0 +1,158 @@
\documentclass[a4paper,12pt]{article}
\usepackage{myXsim}
\usepackage{pgfplots}
\usetikzlibrary{decorations.markings}
\pgfplotsset{compat=1.18}
\title{ DM1 \hfill LAFAVERGES Joana}
\tribe{1ST}
\date{A rendre pour le lundi 27 mars 2023}
\duree{}
\xsimsetup{
solution/print = true
}
\pagestyle{empty}
\begin{document}
\maketitle
Le barème est donné à titre indicatif, il pourra être modifié.
\begin{exercise}[subtitle={Polynôme de degré 2}]
Développer les expressions suivantes pour vérifier que ce sont des polynômes de degré 2. Vous préciserez les valeurs de $a$, $b$ et $c$.
\begin{multicols}{2}
\begin{enumerate}
\item $f(x) = (10x + 7)(4x + 7)$
\item $g(x) = (- 5x - 3)^{2}$
\item $h(x) = - 4 + x(- 2x + 2)$
\item $i(x) = 3x^{2} + x(- 4x - 9)$
\item $j(x) = - 5(x + 3)(x - 10)$
\item $k(x) = 8(x - 2)(x - 6)$
\end{enumerate}
\end{multicols}
\end{exercise}
\begin{solution}
\begin{enumerate}
\item
\begin{align*}
f(x) &= (10x + 7)(4x + 7)\\&= 10x \times 4x + 10x \times 7 + 7 \times 4x + 7 \times 7\\&= 10 \times 4 \times x^{1 + 1} + 7 \times 10 \times x + 7 \times 4 \times x + 49\\&= 70x + 28x + 40x^{2} + 49\\&= (70 + 28) \times x + 40x^{2} + 49\\&= 40x^{2} + 98x + 49
\end{align*}
C'est un polynôme de degré 2 avec $a = 40$, $b = 98$ et $c = 49$.
\item
\begin{align*}
g(x) &= (- 5x - 3)^{2}\\&= (- 5x - 3)(- 5x - 3)\\&= - 5x \times - 5x - 5x(- 3) - 3 \times - 5x - 3(- 3)\\&= - 5(- 5) \times x^{1 + 1} - 3(- 5) \times x - 3(- 5) \times x + 9\\&= 15x + 15x + 25x^{2} + 9\\&= (15 + 15) \times x + 25x^{2} + 9\\&= 25x^{2} + 30x + 9
\end{align*}
C'est un polynôme de degré 2 avec $a = 25$, $b = 30$ et $c = 9$.
\item
\begin{align*}
h(x) &= - 4 + x(- 2x + 2)\\&= - 4 + x \times - 2x + x \times 2\\&= - 2x^{2} + 2x - 4
\end{align*}
C'est un polynôme de degré 2 avec $a = - 2$, $b = 2$ et $c = - 4$.
\item
\begin{align*}
i(x) &= 3x^{2} + x(- 4x - 9)\\&= 3x^{2} + x \times - 4x + x(- 9)\\&= 3x^{2} - 4x^{2} - 9x\\&= 3x^{2} - 4x^{2} - 9x\\&= (3 - 4) \times x^{2} - 9x\\&= - x^{2} - 9x
\end{align*}
C'est un polynôme de degré 2 avec $a = - 1$, $b = - 9$ et $c = 0$.
\item
\begin{align*}
j(x) &= - 5(x + 3)(x - 10)\\&= (- 5x - 5 \times 3)(x - 10)\\&= (- 5x - 15)(x - 10)\\&= - 5x \times x - 5x(- 10) - 15x - 15(- 10)\\&= - 10(- 5) \times x + 150 - 5x^{2} - 15x\\&= 50x + 150 - 5x^{2} - 15x\\&= - 5x^{2} + 50x - 15x + 150\\&= - 5x^{2} + (50 - 15) \times x + 150\\&= - 5x^{2} + 35x + 150
\end{align*}
C'est un polynôme de degré 2 avec $a = - 5$, $b = 35$ et $c = 150$.
\item
\begin{align*}
k(x) &= 8(x - 2)(x - 6)\\&= (8x + 8(- 2))(x - 6)\\&= (8x - 16)(x - 6)\\&= 8x \times x + 8x(- 6) - 16x - 16(- 6)\\&= - 6 \times 8 \times x + 96 + 8x^{2} - 16x\\&= - 48x + 96 + 8x^{2} - 16x\\&= 8x^{2} - 48x - 16x + 96\\&= 8x^{2} + (- 48 - 16) \times x + 96\\&= 8x^{2} - 64x + 96
\end{align*}
C'est un polynôme de degré 2 avec $a = 8$, $b = - 64$ et $c = 96$.
\end{enumerate}
\end{solution}
\begin{exercise}[subtitle={Étude de fonctions}]
Soit $f(x) = 6x^{2} - 36x + 48$ une fonction définie sur $\R$.
\begin{enumerate}
\item Calculer les valeurs suivantes
\[
f(1) \qquad f(-2)
\]
\item Dériver la fonction $f$
\item Étudier le signe de $f'$ puis en déduire les variations de $f$.
\item Est-ce que $f$ admet un maximum? un minimum? Calculer sa valeur.
\end{enumerate}
\end{exercise}
\begin{solution}
\begin{enumerate}
\item On remplace $x$ par les valeurs demandées
\[
f(1) = 6 \times 1^{2} - 36 \times 1 + 48=6 \times 1 - 36 + 48=6 + 12=18
\]
\[
f(-1) = 6 \times - 1^{2} - 36(- 1) + 48=6 \times 1 + 36 + 48=6 + 84=90
\]
\item Dérivation
\[
f'(x) = 12x - 36
\]
\item Pas de solutions automatiques.
\item Pas de solutions automatiques.
\end{enumerate}
\end{solution}
\begin{exercise}[subtitle={Enclos}]
Dans son garage, Jean a trouvé 27m de grillage. \\
Il décide de l'utiliser pour faire un enclos rectangulaire. Afin d'obtenir un enclos encore plus grand, il veut utiliser le mur du jardin qui formera un côté. Le grillage formera les 3 autres.
\begin{center}
\includegraphics[scale=0.8]{./fig/enclos}
\end{center}
Comment placer les poteaux pour avoir un enclos le plus grand possible ?
\textit{Cet exercice est un exercice de recherche. Vous êtes invité à utiliser les outils que vous voulez. Une part importante de la note sera dédiée à la rédaction et aux explications de ce que vous faites. Vous êtes encouragé à faire des schémas.}
\textit{Si vous utilisez le tableur, vous devrez le mentionner et m'envoyer le tableur à l'adresse \url{benjamin.bertrand@ac-lyon.fr}}
\end{exercise}
\begin{solution}
Formule de l'aire (en notant $x$ la longueur du côté de l'enclos à côté du mur)
\[
A(x) = x(27 - 2x) = - 2x^{2} + 27x
\]
On va donc étudier les variations de la fonction $A(x) = - 2x^{2} + 27x$
\begin{itemize}
\item Fonction dérivée : $A'(x) = - 4x + 27$
\item On résout l'inéquation $A'(x) \geq 0$ pour déterminer quand la fonction $A'$ est positive.
\begin{align*}
A(x) & \geq 0 \\
- 4x + 27 & \geq 0 \\
- 4x + 27 + - 27 &\geq 0 + - 27 \\
- 4x &\geq - 27 \\
\frac{- 4x}{- 4} &\leq \frac{- 27}{- 4} \\
x &\leq \dfrac{27}{4} \\
\end{align*}
Donc $A(x)$ est positif quand $x$ est plus \textbf{petit} que $\dfrac{27}{4}$
\item
\begin{center}
\begin{tikzpicture}
\tkzTabInit[lgt=3,espcl=4]{$x$/1,Signe de $f'(x)$/2, Variations de $f(x)$/2}{, $\dfrac{27}{4}$ ,}%
\tkzTabLine{, +, z, -, }
\tkzTabVar{-/ ,+/$f(\dfrac{27}{4}) = \dfrac{1458}{16}$ , -/}%
\end{tikzpicture}
\end{center}
\end{itemize}
Pour avoir le plus grand enclos, il faut placer les poteaux à 6m du mur et on aura alors un enclos de $72m^2$.
\end{solution}
\end{document}
%%% Local Variables:
%%% mode: latex
%%% TeX-master: "master"
%%% End:

Binary file not shown.

View File

@ -0,0 +1,158 @@
\documentclass[a4paper,12pt]{article}
\usepackage{myXsim}
\usepackage{pgfplots}
\usetikzlibrary{decorations.markings}
\pgfplotsset{compat=1.18}
\title{ DM1 \hfill NAKIR SAILAH Mohamed}
\tribe{1ST}
\date{A rendre pour le lundi 27 mars 2023}
\duree{}
\xsimsetup{
solution/print = true
}
\pagestyle{empty}
\begin{document}
\maketitle
Le barème est donné à titre indicatif, il pourra être modifié.
\begin{exercise}[subtitle={Polynôme de degré 2}]
Développer les expressions suivantes pour vérifier que ce sont des polynômes de degré 2. Vous préciserez les valeurs de $a$, $b$ et $c$.
\begin{multicols}{2}
\begin{enumerate}
\item $f(x) = (- 9x + 2)(- 6x + 2)$
\item $g(x) = (8x + 2)^{2}$
\item $h(x) = 2 + x(- 3x + 5)$
\item $i(x) = 8x^{2} + x(5x + 8)$
\item $j(x) = 10(x + 7)(x - 10)$
\item $k(x) = - 4(x - 7)(x + 5)$
\end{enumerate}
\end{multicols}
\end{exercise}
\begin{solution}
\begin{enumerate}
\item
\begin{align*}
f(x) &= (- 9x + 2)(- 6x + 2)\\&= - 9x \times - 6x - 9x \times 2 + 2 \times - 6x + 2 \times 2\\&= - 9(- 6) \times x^{1 + 1} + 2(- 9) \times x + 2(- 6) \times x + 4\\&= - 18x - 12x + 54x^{2} + 4\\&= (- 18 - 12) \times x + 54x^{2} + 4\\&= 54x^{2} - 30x + 4
\end{align*}
C'est un polynôme de degré 2 avec $a = 54$, $b = - 30$ et $c = 4$.
\item
\begin{align*}
g(x) &= (8x + 2)^{2}\\&= (8x + 2)(8x + 2)\\&= 8x \times 8x + 8x \times 2 + 2 \times 8x + 2 \times 2\\&= 8 \times 8 \times x^{1 + 1} + 2 \times 8 \times x + 2 \times 8 \times x + 4\\&= 16x + 16x + 64x^{2} + 4\\&= (16 + 16) \times x + 64x^{2} + 4\\&= 64x^{2} + 32x + 4
\end{align*}
C'est un polynôme de degré 2 avec $a = 64$, $b = 32$ et $c = 4$.
\item
\begin{align*}
h(x) &= 2 + x(- 3x + 5)\\&= 2 + x \times - 3x + x \times 5\\&= - 3x^{2} + 5x + 2
\end{align*}
C'est un polynôme de degré 2 avec $a = - 3$, $b = 5$ et $c = 2$.
\item
\begin{align*}
i(x) &= 8x^{2} + x(5x + 8)\\&= 8x^{2} + x \times 5x + x \times 8\\&= 8x^{2} + 5x^{2} + 8x\\&= 8x^{2} + 5x^{2} + 8x\\&= (8 + 5) \times x^{2} + 8x\\&= 13x^{2} + 8x
\end{align*}
C'est un polynôme de degré 2 avec $a = 13$, $b = 8$ et $c = 0$.
\item
\begin{align*}
j(x) &= 10(x + 7)(x - 10)\\&= (10x + 10 \times 7)(x - 10)\\&= (10x + 70)(x - 10)\\&= 10x \times x + 10x(- 10) + 70x + 70(- 10)\\&= - 10 \times 10 \times x - 700 + 10x^{2} + 70x\\&= - 100x - 700 + 10x^{2} + 70x\\&= 10x^{2} - 100x + 70x - 700\\&= 10x^{2} + (- 100 + 70) \times x - 700\\&= 10x^{2} - 30x - 700
\end{align*}
C'est un polynôme de degré 2 avec $a = 10$, $b = - 30$ et $c = - 700$.
\item
\begin{align*}
k(x) &= - 4(x - 7)(x + 5)\\&= (- 4x - 4(- 7))(x + 5)\\&= (- 4x + 28)(x + 5)\\&= - 4x \times x - 4x \times 5 + 28x + 28 \times 5\\&= 5(- 4) \times x + 140 - 4x^{2} + 28x\\&= - 20x + 140 - 4x^{2} + 28x\\&= - 4x^{2} - 20x + 28x + 140\\&= - 4x^{2} + (- 20 + 28) \times x + 140\\&= - 4x^{2} + 8x + 140
\end{align*}
C'est un polynôme de degré 2 avec $a = - 4$, $b = 8$ et $c = 140$.
\end{enumerate}
\end{solution}
\begin{exercise}[subtitle={Étude de fonctions}]
Soit $f(x) = 2x^{2} - 28x + 80$ une fonction définie sur $\R$.
\begin{enumerate}
\item Calculer les valeurs suivantes
\[
f(1) \qquad f(-2)
\]
\item Dériver la fonction $f$
\item Étudier le signe de $f'$ puis en déduire les variations de $f$.
\item Est-ce que $f$ admet un maximum? un minimum? Calculer sa valeur.
\end{enumerate}
\end{exercise}
\begin{solution}
\begin{enumerate}
\item On remplace $x$ par les valeurs demandées
\[
f(1) = 2 \times 1^{2} - 28 \times 1 + 80=2 \times 1 - 28 + 80=2 + 52=54
\]
\[
f(-1) = 2 \times - 1^{2} - 28(- 1) + 80=2 \times 1 + 28 + 80=2 + 108=110
\]
\item Dérivation
\[
f'(x) = 4x - 28
\]
\item Pas de solutions automatiques.
\item Pas de solutions automatiques.
\end{enumerate}
\end{solution}
\begin{exercise}[subtitle={Enclos}]
Dans son garage, Jean a trouvé 29m de grillage. \\
Il décide de l'utiliser pour faire un enclos rectangulaire. Afin d'obtenir un enclos encore plus grand, il veut utiliser le mur du jardin qui formera un côté. Le grillage formera les 3 autres.
\begin{center}
\includegraphics[scale=0.8]{./fig/enclos}
\end{center}
Comment placer les poteaux pour avoir un enclos le plus grand possible ?
\textit{Cet exercice est un exercice de recherche. Vous êtes invité à utiliser les outils que vous voulez. Une part importante de la note sera dédiée à la rédaction et aux explications de ce que vous faites. Vous êtes encouragé à faire des schémas.}
\textit{Si vous utilisez le tableur, vous devrez le mentionner et m'envoyer le tableur à l'adresse \url{benjamin.bertrand@ac-lyon.fr}}
\end{exercise}
\begin{solution}
Formule de l'aire (en notant $x$ la longueur du côté de l'enclos à côté du mur)
\[
A(x) = x(29 - 2x) = - 2x^{2} + 29x
\]
On va donc étudier les variations de la fonction $A(x) = - 2x^{2} + 29x$
\begin{itemize}
\item Fonction dérivée : $A'(x) = - 4x + 29$
\item On résout l'inéquation $A'(x) \geq 0$ pour déterminer quand la fonction $A'$ est positive.
\begin{align*}
A(x) & \geq 0 \\
- 4x + 29 & \geq 0 \\
- 4x + 29 + - 29 &\geq 0 + - 29 \\
- 4x &\geq - 29 \\
\frac{- 4x}{- 4} &\leq \frac{- 29}{- 4} \\
x &\leq \dfrac{29}{4} \\
\end{align*}
Donc $A(x)$ est positif quand $x$ est plus \textbf{petit} que $\dfrac{29}{4}$
\item
\begin{center}
\begin{tikzpicture}
\tkzTabInit[lgt=3,espcl=4]{$x$/1,Signe de $f'(x)$/2, Variations de $f(x)$/2}{, $\dfrac{29}{4}$ ,}%
\tkzTabLine{, +, z, -, }
\tkzTabVar{-/ ,+/$f(\dfrac{29}{4}) = \dfrac{1682}{16}$ , -/}%
\end{tikzpicture}
\end{center}
\end{itemize}
Pour avoir le plus grand enclos, il faut placer les poteaux à 6m du mur et on aura alors un enclos de $72m^2$.
\end{solution}
\end{document}
%%% Local Variables:
%%% mode: latex
%%% TeX-master: "master"
%%% End:

Binary file not shown.

View File

@ -0,0 +1,158 @@
\documentclass[a4paper,12pt]{article}
\usepackage{myXsim}
\usepackage{pgfplots}
\usetikzlibrary{decorations.markings}
\pgfplotsset{compat=1.18}
\title{ DM1 \hfill NEIVA Diego}
\tribe{1ST}
\date{A rendre pour le lundi 27 mars 2023}
\duree{}
\xsimsetup{
solution/print = true
}
\pagestyle{empty}
\begin{document}
\maketitle
Le barème est donné à titre indicatif, il pourra être modifié.
\begin{exercise}[subtitle={Polynôme de degré 2}]
Développer les expressions suivantes pour vérifier que ce sont des polynômes de degré 2. Vous préciserez les valeurs de $a$, $b$ et $c$.
\begin{multicols}{2}
\begin{enumerate}
\item $f(x) = (10x + 6)(5x + 6)$
\item $g(x) = (- 3x - 8)^{2}$
\item $h(x) = 5 + x(6x + 10)$
\item $i(x) = 2x^{2} + x(3x + 3)$
\item $j(x) = 10(x - 2)(x + 8)$
\item $k(x) = - 8(x - 1)(x - 9)$
\end{enumerate}
\end{multicols}
\end{exercise}
\begin{solution}
\begin{enumerate}
\item
\begin{align*}
f(x) &= (10x + 6)(5x + 6)\\&= 10x \times 5x + 10x \times 6 + 6 \times 5x + 6 \times 6\\&= 10 \times 5 \times x^{1 + 1} + 6 \times 10 \times x + 6 \times 5 \times x + 36\\&= 60x + 30x + 50x^{2} + 36\\&= (60 + 30) \times x + 50x^{2} + 36\\&= 50x^{2} + 90x + 36
\end{align*}
C'est un polynôme de degré 2 avec $a = 50$, $b = 90$ et $c = 36$.
\item
\begin{align*}
g(x) &= (- 3x - 8)^{2}\\&= (- 3x - 8)(- 3x - 8)\\&= - 3x \times - 3x - 3x(- 8) - 8 \times - 3x - 8(- 8)\\&= - 3(- 3) \times x^{1 + 1} - 8(- 3) \times x - 8(- 3) \times x + 64\\&= 24x + 24x + 9x^{2} + 64\\&= (24 + 24) \times x + 9x^{2} + 64\\&= 9x^{2} + 48x + 64
\end{align*}
C'est un polynôme de degré 2 avec $a = 9$, $b = 48$ et $c = 64$.
\item
\begin{align*}
h(x) &= 5 + x(6x + 10)\\&= 5 + x \times 6x + x \times 10\\&= 6x^{2} + 10x + 5
\end{align*}
C'est un polynôme de degré 2 avec $a = 6$, $b = 10$ et $c = 5$.
\item
\begin{align*}
i(x) &= 2x^{2} + x(3x + 3)\\&= 2x^{2} + x \times 3x + x \times 3\\&= 2x^{2} + 3x^{2} + 3x\\&= 2x^{2} + 3x^{2} + 3x\\&= (2 + 3) \times x^{2} + 3x\\&= 5x^{2} + 3x
\end{align*}
C'est un polynôme de degré 2 avec $a = 5$, $b = 3$ et $c = 0$.
\item
\begin{align*}
j(x) &= 10(x - 2)(x + 8)\\&= (10x + 10(- 2))(x + 8)\\&= (10x - 20)(x + 8)\\&= 10x \times x + 10x \times 8 - 20x - 20 \times 8\\&= 8 \times 10 \times x - 160 + 10x^{2} - 20x\\&= 80x - 160 + 10x^{2} - 20x\\&= 10x^{2} + 80x - 20x - 160\\&= 10x^{2} + (80 - 20) \times x - 160\\&= 10x^{2} + 60x - 160
\end{align*}
C'est un polynôme de degré 2 avec $a = 10$, $b = 60$ et $c = - 160$.
\item
\begin{align*}
k(x) &= - 8(x - 1)(x - 9)\\&= (- 8x - 8(- 1))(x - 9)\\&= (- 8x + 8)(x - 9)\\&= - 8x \times x - 8x(- 9) + 8x + 8(- 9)\\&= - 9(- 8) \times x - 72 - 8x^{2} + 8x\\&= 72x - 72 - 8x^{2} + 8x\\&= - 8x^{2} + 72x + 8x - 72\\&= - 8x^{2} + (72 + 8) \times x - 72\\&= - 8x^{2} + 80x - 72
\end{align*}
C'est un polynôme de degré 2 avec $a = - 8$, $b = 80$ et $c = - 72$.
\end{enumerate}
\end{solution}
\begin{exercise}[subtitle={Étude de fonctions}]
Soit $f(x) = - 4x^{2} - 8x + 60$ une fonction définie sur $\R$.
\begin{enumerate}
\item Calculer les valeurs suivantes
\[
f(1) \qquad f(-2)
\]
\item Dériver la fonction $f$
\item Étudier le signe de $f'$ puis en déduire les variations de $f$.
\item Est-ce que $f$ admet un maximum? un minimum? Calculer sa valeur.
\end{enumerate}
\end{exercise}
\begin{solution}
\begin{enumerate}
\item On remplace $x$ par les valeurs demandées
\[
f(1) = - 4 \times 1^{2} - 8 \times 1 + 60=- 4 \times 1 - 8 + 60=- 4 + 52=48
\]
\[
f(-1) = - 4 \times - 1^{2} - 8(- 1) + 60=- 4 \times 1 + 8 + 60=- 4 + 68=64
\]
\item Dérivation
\[
f'(x) = - 8x - 8
\]
\item Pas de solutions automatiques.
\item Pas de solutions automatiques.
\end{enumerate}
\end{solution}
\begin{exercise}[subtitle={Enclos}]
Dans son garage, Jean a trouvé 35m de grillage. \\
Il décide de l'utiliser pour faire un enclos rectangulaire. Afin d'obtenir un enclos encore plus grand, il veut utiliser le mur du jardin qui formera un côté. Le grillage formera les 3 autres.
\begin{center}
\includegraphics[scale=0.8]{./fig/enclos}
\end{center}
Comment placer les poteaux pour avoir un enclos le plus grand possible ?
\textit{Cet exercice est un exercice de recherche. Vous êtes invité à utiliser les outils que vous voulez. Une part importante de la note sera dédiée à la rédaction et aux explications de ce que vous faites. Vous êtes encouragé à faire des schémas.}
\textit{Si vous utilisez le tableur, vous devrez le mentionner et m'envoyer le tableur à l'adresse \url{benjamin.bertrand@ac-lyon.fr}}
\end{exercise}
\begin{solution}
Formule de l'aire (en notant $x$ la longueur du côté de l'enclos à côté du mur)
\[
A(x) = x(35 - 2x) = - 2x^{2} + 35x
\]
On va donc étudier les variations de la fonction $A(x) = - 2x^{2} + 35x$
\begin{itemize}
\item Fonction dérivée : $A'(x) = - 4x + 35$
\item On résout l'inéquation $A'(x) \geq 0$ pour déterminer quand la fonction $A'$ est positive.
\begin{align*}
A(x) & \geq 0 \\
- 4x + 35 & \geq 0 \\
- 4x + 35 + - 35 &\geq 0 + - 35 \\
- 4x &\geq - 35 \\
\frac{- 4x}{- 4} &\leq \frac{- 35}{- 4} \\
x &\leq \dfrac{35}{4} \\
\end{align*}
Donc $A(x)$ est positif quand $x$ est plus \textbf{petit} que $\dfrac{35}{4}$
\item
\begin{center}
\begin{tikzpicture}
\tkzTabInit[lgt=3,espcl=4]{$x$/1,Signe de $f'(x)$/2, Variations de $f(x)$/2}{, $\dfrac{35}{4}$ ,}%
\tkzTabLine{, +, z, -, }
\tkzTabVar{-/ ,+/$f(\dfrac{35}{4}) = \dfrac{2450}{16}$ , -/}%
\end{tikzpicture}
\end{center}
\end{itemize}
Pour avoir le plus grand enclos, il faut placer les poteaux à 6m du mur et on aura alors un enclos de $72m^2$.
\end{solution}
\end{document}
%%% Local Variables:
%%% mode: latex
%%% TeX-master: "master"
%%% End:

Binary file not shown.

View File

@ -0,0 +1,158 @@
\documentclass[a4paper,12pt]{article}
\usepackage{myXsim}
\usepackage{pgfplots}
\usetikzlibrary{decorations.markings}
\pgfplotsset{compat=1.18}
\title{ DM1 \hfill SAADI Yazid}
\tribe{1ST}
\date{A rendre pour le lundi 27 mars 2023}
\duree{}
\xsimsetup{
solution/print = true
}
\pagestyle{empty}
\begin{document}
\maketitle
Le barème est donné à titre indicatif, il pourra être modifié.
\begin{exercise}[subtitle={Polynôme de degré 2}]
Développer les expressions suivantes pour vérifier que ce sont des polynômes de degré 2. Vous préciserez les valeurs de $a$, $b$ et $c$.
\begin{multicols}{2}
\begin{enumerate}
\item $f(x) = (- 8x - 2)(- 6x - 2)$
\item $g(x) = (7x + 8)^{2}$
\item $h(x) = - 7 + x(- 1x - 10)$
\item $i(x) = 2x^{2} + x(6x + 6)$
\item $j(x) = - 9(x - 9)(x + 10)$
\item $k(x) = 4(x - 9)(x - 8)$
\end{enumerate}
\end{multicols}
\end{exercise}
\begin{solution}
\begin{enumerate}
\item
\begin{align*}
f(x) &= (- 8x - 2)(- 6x - 2)\\&= - 8x \times - 6x - 8x(- 2) - 2 \times - 6x - 2(- 2)\\&= - 8(- 6) \times x^{1 + 1} - 2(- 8) \times x - 2(- 6) \times x + 4\\&= 16x + 12x + 48x^{2} + 4\\&= (16 + 12) \times x + 48x^{2} + 4\\&= 48x^{2} + 28x + 4
\end{align*}
C'est un polynôme de degré 2 avec $a = 48$, $b = 28$ et $c = 4$.
\item
\begin{align*}
g(x) &= (7x + 8)^{2}\\&= (7x + 8)(7x + 8)\\&= 7x \times 7x + 7x \times 8 + 8 \times 7x + 8 \times 8\\&= 7 \times 7 \times x^{1 + 1} + 8 \times 7 \times x + 8 \times 7 \times x + 64\\&= 56x + 56x + 49x^{2} + 64\\&= (56 + 56) \times x + 49x^{2} + 64\\&= 49x^{2} + 112x + 64
\end{align*}
C'est un polynôme de degré 2 avec $a = 49$, $b = 112$ et $c = 64$.
\item
\begin{align*}
h(x) &= - 7 + x(- 1x - 10)\\&= - 7 + x(- x) + x(- 10)\\&= - x^{2} - 10x - 7
\end{align*}
C'est un polynôme de degré 2 avec $a = - 1$, $b = - 10$ et $c = - 7$.
\item
\begin{align*}
i(x) &= 2x^{2} + x(6x + 6)\\&= 2x^{2} + x \times 6x + x \times 6\\&= 2x^{2} + 6x^{2} + 6x\\&= 2x^{2} + 6x^{2} + 6x\\&= (2 + 6) \times x^{2} + 6x\\&= 8x^{2} + 6x
\end{align*}
C'est un polynôme de degré 2 avec $a = 8$, $b = 6$ et $c = 0$.
\item
\begin{align*}
j(x) &= - 9(x - 9)(x + 10)\\&= (- 9x - 9(- 9))(x + 10)\\&= (- 9x + 81)(x + 10)\\&= - 9x \times x - 9x \times 10 + 81x + 81 \times 10\\&= 10(- 9) \times x + 810 - 9x^{2} + 81x\\&= - 90x + 810 - 9x^{2} + 81x\\&= - 9x^{2} - 90x + 81x + 810\\&= - 9x^{2} + (- 90 + 81) \times x + 810\\&= - 9x^{2} - 9x + 810
\end{align*}
C'est un polynôme de degré 2 avec $a = - 9$, $b = - 9$ et $c = 810$.
\item
\begin{align*}
k(x) &= 4(x - 9)(x - 8)\\&= (4x + 4(- 9))(x - 8)\\&= (4x - 36)(x - 8)\\&= 4x \times x + 4x(- 8) - 36x - 36(- 8)\\&= - 8 \times 4 \times x + 288 + 4x^{2} - 36x\\&= - 32x + 288 + 4x^{2} - 36x\\&= 4x^{2} - 32x - 36x + 288\\&= 4x^{2} + (- 32 - 36) \times x + 288\\&= 4x^{2} - 68x + 288
\end{align*}
C'est un polynôme de degré 2 avec $a = 4$, $b = - 68$ et $c = 288$.
\end{enumerate}
\end{solution}
\begin{exercise}[subtitle={Étude de fonctions}]
Soit $f(x) = 3x^{2} - 39x + 90$ une fonction définie sur $\R$.
\begin{enumerate}
\item Calculer les valeurs suivantes
\[
f(1) \qquad f(-2)
\]
\item Dériver la fonction $f$
\item Étudier le signe de $f'$ puis en déduire les variations de $f$.
\item Est-ce que $f$ admet un maximum? un minimum? Calculer sa valeur.
\end{enumerate}
\end{exercise}
\begin{solution}
\begin{enumerate}
\item On remplace $x$ par les valeurs demandées
\[
f(1) = 3 \times 1^{2} - 39 \times 1 + 90=3 \times 1 - 39 + 90=3 + 51=54
\]
\[
f(-1) = 3 \times - 1^{2} - 39(- 1) + 90=3 \times 1 + 39 + 90=3 + 129=132
\]
\item Dérivation
\[
f'(x) = 6x - 39
\]
\item Pas de solutions automatiques.
\item Pas de solutions automatiques.
\end{enumerate}
\end{solution}
\begin{exercise}[subtitle={Enclos}]
Dans son garage, Jean a trouvé 20m de grillage. \\
Il décide de l'utiliser pour faire un enclos rectangulaire. Afin d'obtenir un enclos encore plus grand, il veut utiliser le mur du jardin qui formera un côté. Le grillage formera les 3 autres.
\begin{center}
\includegraphics[scale=0.8]{./fig/enclos}
\end{center}
Comment placer les poteaux pour avoir un enclos le plus grand possible ?
\textit{Cet exercice est un exercice de recherche. Vous êtes invité à utiliser les outils que vous voulez. Une part importante de la note sera dédiée à la rédaction et aux explications de ce que vous faites. Vous êtes encouragé à faire des schémas.}
\textit{Si vous utilisez le tableur, vous devrez le mentionner et m'envoyer le tableur à l'adresse \url{benjamin.bertrand@ac-lyon.fr}}
\end{exercise}
\begin{solution}
Formule de l'aire (en notant $x$ la longueur du côté de l'enclos à côté du mur)
\[
A(x) = x(20 - 2x) = - 2x^{2} + 20x
\]
On va donc étudier les variations de la fonction $A(x) = - 2x^{2} + 20x$
\begin{itemize}
\item Fonction dérivée : $A'(x) = - 4x + 20$
\item On résout l'inéquation $A'(x) \geq 0$ pour déterminer quand la fonction $A'$ est positive.
\begin{align*}
A(x) & \geq 0 \\
- 4x + 20 & \geq 0 \\
- 4x + 20 + - 20 &\geq 0 + - 20 \\
- 4x &\geq - 20 \\
\frac{- 4x}{- 4} &\leq \frac{- 20}{- 4} \\
x &\leq 5 \\
\end{align*}
Donc $A(x)$ est positif quand $x$ est plus \textbf{petit} que $5$
\item
\begin{center}
\begin{tikzpicture}
\tkzTabInit[lgt=3,espcl=4]{$x$/1,Signe de $f'(x)$/2, Variations de $f(x)$/2}{, $5$ ,}%
\tkzTabLine{, +, z, -, }
\tkzTabVar{-/ ,+/$f(5) = 50$ , -/}%
\end{tikzpicture}
\end{center}
\end{itemize}
Pour avoir le plus grand enclos, il faut placer les poteaux à 6m du mur et on aura alors un enclos de $72m^2$.
\end{solution}
\end{document}
%%% Local Variables:
%%% mode: latex
%%% TeX-master: "master"
%%% End:

Binary file not shown.

View File

@ -0,0 +1,158 @@
\documentclass[a4paper,12pt]{article}
\usepackage{myXsim}
\usepackage{pgfplots}
\usetikzlibrary{decorations.markings}
\pgfplotsset{compat=1.18}
\title{ DM1 \hfill SCOPELLITI Martina}
\tribe{1ST}
\date{A rendre pour le lundi 27 mars 2023}
\duree{}
\xsimsetup{
solution/print = true
}
\pagestyle{empty}
\begin{document}
\maketitle
Le barème est donné à titre indicatif, il pourra être modifié.
\begin{exercise}[subtitle={Polynôme de degré 2}]
Développer les expressions suivantes pour vérifier que ce sont des polynômes de degré 2. Vous préciserez les valeurs de $a$, $b$ et $c$.
\begin{multicols}{2}
\begin{enumerate}
\item $f(x) = (- 1x - 10)(6x - 10)$
\item $g(x) = (- 8x - 7)^{2}$
\item $h(x) = - 2 + x(6x - 10)$
\item $i(x) = 5x^{2} + x(- 4x + 9)$
\item $j(x) = - 10(x + 7)(x + 3)$
\item $k(x) = 4(x - 10)(x - 3)$
\end{enumerate}
\end{multicols}
\end{exercise}
\begin{solution}
\begin{enumerate}
\item
\begin{align*}
f(x) &= (- 1x - 10)(6x - 10)\\&= (- x) \times 6x + (- x)(- 10) - 10 \times 6x - 10(- 10)\\&= - 1 \times 6 \times x^{1 + 1} - 10(- 1) \times x - 10 \times 6 \times x + 100\\&= 10x - 60x - 6x^{2} + 100\\&= (10 - 60) \times x - 6x^{2} + 100\\&= - 6x^{2} - 50x + 100
\end{align*}
C'est un polynôme de degré 2 avec $a = - 6$, $b = - 50$ et $c = 100$.
\item
\begin{align*}
g(x) &= (- 8x - 7)^{2}\\&= (- 8x - 7)(- 8x - 7)\\&= - 8x \times - 8x - 8x(- 7) - 7 \times - 8x - 7(- 7)\\&= - 8(- 8) \times x^{1 + 1} - 7(- 8) \times x - 7(- 8) \times x + 49\\&= 56x + 56x + 64x^{2} + 49\\&= (56 + 56) \times x + 64x^{2} + 49\\&= 64x^{2} + 112x + 49
\end{align*}
C'est un polynôme de degré 2 avec $a = 64$, $b = 112$ et $c = 49$.
\item
\begin{align*}
h(x) &= - 2 + x(6x - 10)\\&= - 2 + x \times 6x + x(- 10)\\&= 6x^{2} - 10x - 2
\end{align*}
C'est un polynôme de degré 2 avec $a = 6$, $b = - 10$ et $c = - 2$.
\item
\begin{align*}
i(x) &= 5x^{2} + x(- 4x + 9)\\&= 5x^{2} + x \times - 4x + x \times 9\\&= 5x^{2} - 4x^{2} + 9x\\&= 5x^{2} - 4x^{2} + 9x\\&= (5 - 4) \times x^{2} + 9x\\&= x^{2} + 9x
\end{align*}
C'est un polynôme de degré 2 avec $a = 1$, $b = 9$ et $c = 0$.
\item
\begin{align*}
j(x) &= - 10(x + 7)(x + 3)\\&= (- 10x - 10 \times 7)(x + 3)\\&= (- 10x - 70)(x + 3)\\&= - 10x \times x - 10x \times 3 - 70x - 70 \times 3\\&= 3(- 10) \times x - 210 - 10x^{2} - 70x\\&= - 30x - 210 - 10x^{2} - 70x\\&= - 10x^{2} - 30x - 70x - 210\\&= - 10x^{2} + (- 30 - 70) \times x - 210\\&= - 10x^{2} - 100x - 210
\end{align*}
C'est un polynôme de degré 2 avec $a = - 10$, $b = - 100$ et $c = - 210$.
\item
\begin{align*}
k(x) &= 4(x - 10)(x - 3)\\&= (4x + 4(- 10))(x - 3)\\&= (4x - 40)(x - 3)\\&= 4x \times x + 4x(- 3) - 40x - 40(- 3)\\&= - 3 \times 4 \times x + 120 + 4x^{2} - 40x\\&= - 12x + 120 + 4x^{2} - 40x\\&= 4x^{2} - 12x - 40x + 120\\&= 4x^{2} + (- 12 - 40) \times x + 120\\&= 4x^{2} - 52x + 120
\end{align*}
C'est un polynôme de degré 2 avec $a = 4$, $b = - 52$ et $c = 120$.
\end{enumerate}
\end{solution}
\begin{exercise}[subtitle={Étude de fonctions}]
Soit $f(x) = - 9x^{2} - 153x - 630$ une fonction définie sur $\R$.
\begin{enumerate}
\item Calculer les valeurs suivantes
\[
f(1) \qquad f(-2)
\]
\item Dériver la fonction $f$
\item Étudier le signe de $f'$ puis en déduire les variations de $f$.
\item Est-ce que $f$ admet un maximum? un minimum? Calculer sa valeur.
\end{enumerate}
\end{exercise}
\begin{solution}
\begin{enumerate}
\item On remplace $x$ par les valeurs demandées
\[
f(1) = - 9 \times 1^{2} - 153 \times 1 - 630=- 9 \times 1 - 153 - 630=- 9 - 783=- 792
\]
\[
f(-1) = - 9 \times - 1^{2} - 153(- 1) - 630=- 9 \times 1 + 153 - 630=- 9 - 477=- 486
\]
\item Dérivation
\[
f'(x) = - 18x - 153
\]
\item Pas de solutions automatiques.
\item Pas de solutions automatiques.
\end{enumerate}
\end{solution}
\begin{exercise}[subtitle={Enclos}]
Dans son garage, Jean a trouvé 28m de grillage. \\
Il décide de l'utiliser pour faire un enclos rectangulaire. Afin d'obtenir un enclos encore plus grand, il veut utiliser le mur du jardin qui formera un côté. Le grillage formera les 3 autres.
\begin{center}
\includegraphics[scale=0.8]{./fig/enclos}
\end{center}
Comment placer les poteaux pour avoir un enclos le plus grand possible ?
\textit{Cet exercice est un exercice de recherche. Vous êtes invité à utiliser les outils que vous voulez. Une part importante de la note sera dédiée à la rédaction et aux explications de ce que vous faites. Vous êtes encouragé à faire des schémas.}
\textit{Si vous utilisez le tableur, vous devrez le mentionner et m'envoyer le tableur à l'adresse \url{benjamin.bertrand@ac-lyon.fr}}
\end{exercise}
\begin{solution}
Formule de l'aire (en notant $x$ la longueur du côté de l'enclos à côté du mur)
\[
A(x) = x(28 - 2x) = - 2x^{2} + 28x
\]
On va donc étudier les variations de la fonction $A(x) = - 2x^{2} + 28x$
\begin{itemize}
\item Fonction dérivée : $A'(x) = - 4x + 28$
\item On résout l'inéquation $A'(x) \geq 0$ pour déterminer quand la fonction $A'$ est positive.
\begin{align*}
A(x) & \geq 0 \\
- 4x + 28 & \geq 0 \\
- 4x + 28 + - 28 &\geq 0 + - 28 \\
- 4x &\geq - 28 \\
\frac{- 4x}{- 4} &\leq \frac{- 28}{- 4} \\
x &\leq 7 \\
\end{align*}
Donc $A(x)$ est positif quand $x$ est plus \textbf{petit} que $7$
\item
\begin{center}
\begin{tikzpicture}
\tkzTabInit[lgt=3,espcl=4]{$x$/1,Signe de $f'(x)$/2, Variations de $f(x)$/2}{, $7$ ,}%
\tkzTabLine{, +, z, -, }
\tkzTabVar{-/ ,+/$f(7) = 98$ , -/}%
\end{tikzpicture}
\end{center}
\end{itemize}
Pour avoir le plus grand enclos, il faut placer les poteaux à 6m du mur et on aura alors un enclos de $72m^2$.
\end{solution}
\end{document}
%%% Local Variables:
%%% mode: latex
%%% TeX-master: "master"
%%% End:

Binary file not shown.

View File

@ -0,0 +1,158 @@
\documentclass[a4paper,12pt]{article}
\usepackage{myXsim}
\usepackage{pgfplots}
\usetikzlibrary{decorations.markings}
\pgfplotsset{compat=1.18}
\title{ DM1 \hfill SORIANO Johan}
\tribe{1ST}
\date{A rendre pour le lundi 27 mars 2023}
\duree{}
\xsimsetup{
solution/print = true
}
\pagestyle{empty}
\begin{document}
\maketitle
Le barème est donné à titre indicatif, il pourra être modifié.
\begin{exercise}[subtitle={Polynôme de degré 2}]
Développer les expressions suivantes pour vérifier que ce sont des polynômes de degré 2. Vous préciserez les valeurs de $a$, $b$ et $c$.
\begin{multicols}{2}
\begin{enumerate}
\item $f(x) = (5x - 2)(5x - 2)$
\item $g(x) = (- 4x - 4)^{2}$
\item $h(x) = - 9 + x(- 9x + 5)$
\item $i(x) = - 4x^{2} + x(5x - 4)$
\item $j(x) = 3(x - 10)(x - 6)$
\item $k(x) = 6(x + 7)(x - 9)$
\end{enumerate}
\end{multicols}
\end{exercise}
\begin{solution}
\begin{enumerate}
\item
\begin{align*}
f(x) &= (5x - 2)(5x - 2)\\&= 5x \times 5x + 5x(- 2) - 2 \times 5x - 2(- 2)\\&= 5 \times 5 \times x^{1 + 1} - 2 \times 5 \times x - 2 \times 5 \times x + 4\\&= - 10x - 10x + 25x^{2} + 4\\&= (- 10 - 10) \times x + 25x^{2} + 4\\&= 25x^{2} - 20x + 4
\end{align*}
C'est un polynôme de degré 2 avec $a = 25$, $b = - 20$ et $c = 4$.
\item
\begin{align*}
g(x) &= (- 4x - 4)^{2}\\&= (- 4x - 4)(- 4x - 4)\\&= - 4x \times - 4x - 4x(- 4) - 4 \times - 4x - 4(- 4)\\&= - 4(- 4) \times x^{1 + 1} - 4(- 4) \times x - 4(- 4) \times x + 16\\&= 16x + 16x + 16x^{2} + 16\\&= (16 + 16) \times x + 16x^{2} + 16\\&= 16x^{2} + 32x + 16
\end{align*}
C'est un polynôme de degré 2 avec $a = 16$, $b = 32$ et $c = 16$.
\item
\begin{align*}
h(x) &= - 9 + x(- 9x + 5)\\&= - 9 + x \times - 9x + x \times 5\\&= - 9x^{2} + 5x - 9
\end{align*}
C'est un polynôme de degré 2 avec $a = - 9$, $b = 5$ et $c = - 9$.
\item
\begin{align*}
i(x) &= - 4x^{2} + x(5x - 4)\\&= - 4x^{2} + x \times 5x + x(- 4)\\&= - 4x^{2} + 5x^{2} - 4x\\&= - 4x^{2} + 5x^{2} - 4x\\&= (- 4 + 5) \times x^{2} - 4x\\&= x^{2} - 4x
\end{align*}
C'est un polynôme de degré 2 avec $a = 1$, $b = - 4$ et $c = 0$.
\item
\begin{align*}
j(x) &= 3(x - 10)(x - 6)\\&= (3x + 3(- 10))(x - 6)\\&= (3x - 30)(x - 6)\\&= 3x \times x + 3x(- 6) - 30x - 30(- 6)\\&= - 6 \times 3 \times x + 180 + 3x^{2} - 30x\\&= - 18x + 180 + 3x^{2} - 30x\\&= 3x^{2} - 18x - 30x + 180\\&= 3x^{2} + (- 18 - 30) \times x + 180\\&= 3x^{2} - 48x + 180
\end{align*}
C'est un polynôme de degré 2 avec $a = 3$, $b = - 48$ et $c = 180$.
\item
\begin{align*}
k(x) &= 6(x + 7)(x - 9)\\&= (6x + 6 \times 7)(x - 9)\\&= (6x + 42)(x - 9)\\&= 6x \times x + 6x(- 9) + 42x + 42(- 9)\\&= - 9 \times 6 \times x - 378 + 6x^{2} + 42x\\&= - 54x - 378 + 6x^{2} + 42x\\&= 6x^{2} - 54x + 42x - 378\\&= 6x^{2} + (- 54 + 42) \times x - 378\\&= 6x^{2} - 12x - 378
\end{align*}
C'est un polynôme de degré 2 avec $a = 6$, $b = - 12$ et $c = - 378$.
\end{enumerate}
\end{solution}
\begin{exercise}[subtitle={Étude de fonctions}]
Soit $f(x) = - 7x^{2} + 21x + 28$ une fonction définie sur $\R$.
\begin{enumerate}
\item Calculer les valeurs suivantes
\[
f(1) \qquad f(-2)
\]
\item Dériver la fonction $f$
\item Étudier le signe de $f'$ puis en déduire les variations de $f$.
\item Est-ce que $f$ admet un maximum? un minimum? Calculer sa valeur.
\end{enumerate}
\end{exercise}
\begin{solution}
\begin{enumerate}
\item On remplace $x$ par les valeurs demandées
\[
f(1) = - 7 \times 1^{2} + 21 \times 1 + 28=- 7 \times 1 + 21 + 28=- 7 + 49=42
\]
\[
f(-1) = - 7 \times - 1^{2} + 21(- 1) + 28=- 7 \times 1 - 21 + 28=- 7 + 7=0
\]
\item Dérivation
\[
f'(x) = - 14x + 21
\]
\item Pas de solutions automatiques.
\item Pas de solutions automatiques.
\end{enumerate}
\end{solution}
\begin{exercise}[subtitle={Enclos}]
Dans son garage, Jean a trouvé 17m de grillage. \\
Il décide de l'utiliser pour faire un enclos rectangulaire. Afin d'obtenir un enclos encore plus grand, il veut utiliser le mur du jardin qui formera un côté. Le grillage formera les 3 autres.
\begin{center}
\includegraphics[scale=0.8]{./fig/enclos}
\end{center}
Comment placer les poteaux pour avoir un enclos le plus grand possible ?
\textit{Cet exercice est un exercice de recherche. Vous êtes invité à utiliser les outils que vous voulez. Une part importante de la note sera dédiée à la rédaction et aux explications de ce que vous faites. Vous êtes encouragé à faire des schémas.}
\textit{Si vous utilisez le tableur, vous devrez le mentionner et m'envoyer le tableur à l'adresse \url{benjamin.bertrand@ac-lyon.fr}}
\end{exercise}
\begin{solution}
Formule de l'aire (en notant $x$ la longueur du côté de l'enclos à côté du mur)
\[
A(x) = x(17 - 2x) = - 2x^{2} + 17x
\]
On va donc étudier les variations de la fonction $A(x) = - 2x^{2} + 17x$
\begin{itemize}
\item Fonction dérivée : $A'(x) = - 4x + 17$
\item On résout l'inéquation $A'(x) \geq 0$ pour déterminer quand la fonction $A'$ est positive.
\begin{align*}
A(x) & \geq 0 \\
- 4x + 17 & \geq 0 \\
- 4x + 17 + - 17 &\geq 0 + - 17 \\
- 4x &\geq - 17 \\
\frac{- 4x}{- 4} &\leq \frac{- 17}{- 4} \\
x &\leq \dfrac{17}{4} \\
\end{align*}
Donc $A(x)$ est positif quand $x$ est plus \textbf{petit} que $\dfrac{17}{4}$
\item
\begin{center}
\begin{tikzpicture}
\tkzTabInit[lgt=3,espcl=4]{$x$/1,Signe de $f'(x)$/2, Variations de $f(x)$/2}{, $\dfrac{17}{4}$ ,}%
\tkzTabLine{, +, z, -, }
\tkzTabVar{-/ ,+/$f(\dfrac{17}{4}) = \dfrac{578}{16}$ , -/}%
\end{tikzpicture}
\end{center}
\end{itemize}
Pour avoir le plus grand enclos, il faut placer les poteaux à 6m du mur et on aura alors un enclos de $72m^2$.
\end{solution}
\end{document}
%%% Local Variables:
%%% mode: latex
%%% TeX-master: "master"
%%% End:

Binary file not shown.

View File

@ -0,0 +1,158 @@
\documentclass[a4paper,12pt]{article}
\usepackage{myXsim}
\usepackage{pgfplots}
\usetikzlibrary{decorations.markings}
\pgfplotsset{compat=1.18}
\title{ DM1 \hfill STRUKELJ Charles}
\tribe{1ST}
\date{A rendre pour le lundi 27 mars 2023}
\duree{}
\xsimsetup{
solution/print = true
}
\pagestyle{empty}
\begin{document}
\maketitle
Le barème est donné à titre indicatif, il pourra être modifié.
\begin{exercise}[subtitle={Polynôme de degré 2}]
Développer les expressions suivantes pour vérifier que ce sont des polynômes de degré 2. Vous préciserez les valeurs de $a$, $b$ et $c$.
\begin{multicols}{2}
\begin{enumerate}
\item $f(x) = (8x + 9)(4x + 9)$
\item $g(x) = (3x - 7)^{2}$
\item $h(x) = 2 + x(5x - 7)$
\item $i(x) = - 2x^{2} + x(- 3x - 10)$
\item $j(x) = - 4(x - 1)(x - 5)$
\item $k(x) = - 6(x + 4)(x + 8)$
\end{enumerate}
\end{multicols}
\end{exercise}
\begin{solution}
\begin{enumerate}
\item
\begin{align*}
f(x) &= (8x + 9)(4x + 9)\\&= 8x \times 4x + 8x \times 9 + 9 \times 4x + 9 \times 9\\&= 8 \times 4 \times x^{1 + 1} + 9 \times 8 \times x + 9 \times 4 \times x + 81\\&= 72x + 36x + 32x^{2} + 81\\&= (72 + 36) \times x + 32x^{2} + 81\\&= 32x^{2} + 108x + 81
\end{align*}
C'est un polynôme de degré 2 avec $a = 32$, $b = 108$ et $c = 81$.
\item
\begin{align*}
g(x) &= (3x - 7)^{2}\\&= (3x - 7)(3x - 7)\\&= 3x \times 3x + 3x(- 7) - 7 \times 3x - 7(- 7)\\&= 3 \times 3 \times x^{1 + 1} - 7 \times 3 \times x - 7 \times 3 \times x + 49\\&= - 21x - 21x + 9x^{2} + 49\\&= (- 21 - 21) \times x + 9x^{2} + 49\\&= 9x^{2} - 42x + 49
\end{align*}
C'est un polynôme de degré 2 avec $a = 9$, $b = - 42$ et $c = 49$.
\item
\begin{align*}
h(x) &= 2 + x(5x - 7)\\&= 2 + x \times 5x + x(- 7)\\&= 5x^{2} - 7x + 2
\end{align*}
C'est un polynôme de degré 2 avec $a = 5$, $b = - 7$ et $c = 2$.
\item
\begin{align*}
i(x) &= - 2x^{2} + x(- 3x - 10)\\&= - 2x^{2} + x \times - 3x + x(- 10)\\&= - 2x^{2} - 3x^{2} - 10x\\&= - 2x^{2} - 3x^{2} - 10x\\&= (- 2 - 3) \times x^{2} - 10x\\&= - 5x^{2} - 10x
\end{align*}
C'est un polynôme de degré 2 avec $a = - 5$, $b = - 10$ et $c = 0$.
\item
\begin{align*}
j(x) &= - 4(x - 1)(x - 5)\\&= (- 4x - 4(- 1))(x - 5)\\&= (- 4x + 4)(x - 5)\\&= - 4x \times x - 4x(- 5) + 4x + 4(- 5)\\&= - 5(- 4) \times x - 20 - 4x^{2} + 4x\\&= 20x - 20 - 4x^{2} + 4x\\&= - 4x^{2} + 20x + 4x - 20\\&= - 4x^{2} + (20 + 4) \times x - 20\\&= - 4x^{2} + 24x - 20
\end{align*}
C'est un polynôme de degré 2 avec $a = - 4$, $b = 24$ et $c = - 20$.
\item
\begin{align*}
k(x) &= - 6(x + 4)(x + 8)\\&= (- 6x - 6 \times 4)(x + 8)\\&= (- 6x - 24)(x + 8)\\&= - 6x \times x - 6x \times 8 - 24x - 24 \times 8\\&= 8(- 6) \times x - 192 - 6x^{2} - 24x\\&= - 48x - 192 - 6x^{2} - 24x\\&= - 6x^{2} - 48x - 24x - 192\\&= - 6x^{2} + (- 48 - 24) \times x - 192\\&= - 6x^{2} - 72x - 192
\end{align*}
C'est un polynôme de degré 2 avec $a = - 6$, $b = - 72$ et $c = - 192$.
\end{enumerate}
\end{solution}
\begin{exercise}[subtitle={Étude de fonctions}]
Soit $f(x) = 9x^{2} - 18x - 720$ une fonction définie sur $\R$.
\begin{enumerate}
\item Calculer les valeurs suivantes
\[
f(1) \qquad f(-2)
\]
\item Dériver la fonction $f$
\item Étudier le signe de $f'$ puis en déduire les variations de $f$.
\item Est-ce que $f$ admet un maximum? un minimum? Calculer sa valeur.
\end{enumerate}
\end{exercise}
\begin{solution}
\begin{enumerate}
\item On remplace $x$ par les valeurs demandées
\[
f(1) = 9 \times 1^{2} - 18 \times 1 - 720=9 \times 1 - 18 - 720=9 - 738=- 729
\]
\[
f(-1) = 9 \times - 1^{2} - 18(- 1) - 720=9 \times 1 + 18 - 720=9 - 702=- 693
\]
\item Dérivation
\[
f'(x) = 18x - 18
\]
\item Pas de solutions automatiques.
\item Pas de solutions automatiques.
\end{enumerate}
\end{solution}
\begin{exercise}[subtitle={Enclos}]
Dans son garage, Jean a trouvé 33m de grillage. \\
Il décide de l'utiliser pour faire un enclos rectangulaire. Afin d'obtenir un enclos encore plus grand, il veut utiliser le mur du jardin qui formera un côté. Le grillage formera les 3 autres.
\begin{center}
\includegraphics[scale=0.8]{./fig/enclos}
\end{center}
Comment placer les poteaux pour avoir un enclos le plus grand possible ?
\textit{Cet exercice est un exercice de recherche. Vous êtes invité à utiliser les outils que vous voulez. Une part importante de la note sera dédiée à la rédaction et aux explications de ce que vous faites. Vous êtes encouragé à faire des schémas.}
\textit{Si vous utilisez le tableur, vous devrez le mentionner et m'envoyer le tableur à l'adresse \url{benjamin.bertrand@ac-lyon.fr}}
\end{exercise}
\begin{solution}
Formule de l'aire (en notant $x$ la longueur du côté de l'enclos à côté du mur)
\[
A(x) = x(33 - 2x) = - 2x^{2} + 33x
\]
On va donc étudier les variations de la fonction $A(x) = - 2x^{2} + 33x$
\begin{itemize}
\item Fonction dérivée : $A'(x) = - 4x + 33$
\item On résout l'inéquation $A'(x) \geq 0$ pour déterminer quand la fonction $A'$ est positive.
\begin{align*}
A(x) & \geq 0 \\
- 4x + 33 & \geq 0 \\
- 4x + 33 + - 33 &\geq 0 + - 33 \\
- 4x &\geq - 33 \\
\frac{- 4x}{- 4} &\leq \frac{- 33}{- 4} \\
x &\leq \dfrac{33}{4} \\
\end{align*}
Donc $A(x)$ est positif quand $x$ est plus \textbf{petit} que $\dfrac{33}{4}$
\item
\begin{center}
\begin{tikzpicture}
\tkzTabInit[lgt=3,espcl=4]{$x$/1,Signe de $f'(x)$/2, Variations de $f(x)$/2}{, $\dfrac{33}{4}$ ,}%
\tkzTabLine{, +, z, -, }
\tkzTabVar{-/ ,+/$f(\dfrac{33}{4}) = \dfrac{2178}{16}$ , -/}%
\end{tikzpicture}
\end{center}
\end{itemize}
Pour avoir le plus grand enclos, il faut placer les poteaux à 6m du mur et on aura alors un enclos de $72m^2$.
\end{solution}
\end{document}
%%% Local Variables:
%%% mode: latex
%%% TeX-master: "master"
%%% End:

Binary file not shown.

View File

@ -0,0 +1,158 @@
\documentclass[a4paper,12pt]{article}
\usepackage{myXsim}
\usepackage{pgfplots}
\usetikzlibrary{decorations.markings}
\pgfplotsset{compat=1.18}
\title{ DM1 \hfill THERET Olympe}
\tribe{1ST}
\date{A rendre pour le lundi 27 mars 2023}
\duree{}
\xsimsetup{
solution/print = true
}
\pagestyle{empty}
\begin{document}
\maketitle
Le barème est donné à titre indicatif, il pourra être modifié.
\begin{exercise}[subtitle={Polynôme de degré 2}]
Développer les expressions suivantes pour vérifier que ce sont des polynômes de degré 2. Vous préciserez les valeurs de $a$, $b$ et $c$.
\begin{multicols}{2}
\begin{enumerate}
\item $f(x) = (- 6x + 8)(7x + 8)$
\item $g(x) = (- 8x + 5)^{2}$
\item $h(x) = 4 + x(2x - 3)$
\item $i(x) = - 5x^{2} + x(8x + 5)$
\item $j(x) = - 7(x + 10)(x - 4)$
\item $k(x) = 3(x + 8)(x + 8)$
\end{enumerate}
\end{multicols}
\end{exercise}
\begin{solution}
\begin{enumerate}
\item
\begin{align*}
f(x) &= (- 6x + 8)(7x + 8)\\&= - 6x \times 7x - 6x \times 8 + 8 \times 7x + 8 \times 8\\&= - 6 \times 7 \times x^{1 + 1} + 8(- 6) \times x + 8 \times 7 \times x + 64\\&= - 48x + 56x - 42x^{2} + 64\\&= (- 48 + 56) \times x - 42x^{2} + 64\\&= - 42x^{2} + 8x + 64
\end{align*}
C'est un polynôme de degré 2 avec $a = - 42$, $b = 8$ et $c = 64$.
\item
\begin{align*}
g(x) &= (- 8x + 5)^{2}\\&= (- 8x + 5)(- 8x + 5)\\&= - 8x \times - 8x - 8x \times 5 + 5 \times - 8x + 5 \times 5\\&= - 8(- 8) \times x^{1 + 1} + 5(- 8) \times x + 5(- 8) \times x + 25\\&= - 40x - 40x + 64x^{2} + 25\\&= (- 40 - 40) \times x + 64x^{2} + 25\\&= 64x^{2} - 80x + 25
\end{align*}
C'est un polynôme de degré 2 avec $a = 64$, $b = - 80$ et $c = 25$.
\item
\begin{align*}
h(x) &= 4 + x(2x - 3)\\&= 4 + x \times 2x + x(- 3)\\&= 2x^{2} - 3x + 4
\end{align*}
C'est un polynôme de degré 2 avec $a = 2$, $b = - 3$ et $c = 4$.
\item
\begin{align*}
i(x) &= - 5x^{2} + x(8x + 5)\\&= - 5x^{2} + x \times 8x + x \times 5\\&= - 5x^{2} + 8x^{2} + 5x\\&= - 5x^{2} + 8x^{2} + 5x\\&= (- 5 + 8) \times x^{2} + 5x\\&= 3x^{2} + 5x
\end{align*}
C'est un polynôme de degré 2 avec $a = 3$, $b = 5$ et $c = 0$.
\item
\begin{align*}
j(x) &= - 7(x + 10)(x - 4)\\&= (- 7x - 7 \times 10)(x - 4)\\&= (- 7x - 70)(x - 4)\\&= - 7x \times x - 7x(- 4) - 70x - 70(- 4)\\&= - 4(- 7) \times x + 280 - 7x^{2} - 70x\\&= 28x + 280 - 7x^{2} - 70x\\&= - 7x^{2} + 28x - 70x + 280\\&= - 7x^{2} + (28 - 70) \times x + 280\\&= - 7x^{2} - 42x + 280
\end{align*}
C'est un polynôme de degré 2 avec $a = - 7$, $b = - 42$ et $c = 280$.
\item
\begin{align*}
k(x) &= 3(x + 8)(x + 8)\\&= (3x + 3 \times 8)(x + 8)\\&= (3x + 24)(x + 8)\\&= 3x \times x + 3x \times 8 + 24x + 24 \times 8\\&= 8 \times 3 \times x + 192 + 3x^{2} + 24x\\&= 24x + 192 + 3x^{2} + 24x\\&= 3x^{2} + 24x + 24x + 192\\&= 3x^{2} + (24 + 24) \times x + 192\\&= 3x^{2} + 48x + 192
\end{align*}
C'est un polynôme de degré 2 avec $a = 3$, $b = 48$ et $c = 192$.
\end{enumerate}
\end{solution}
\begin{exercise}[subtitle={Étude de fonctions}]
Soit $f(x) = 5x^{2} - 70x + 240$ une fonction définie sur $\R$.
\begin{enumerate}
\item Calculer les valeurs suivantes
\[
f(1) \qquad f(-2)
\]
\item Dériver la fonction $f$
\item Étudier le signe de $f'$ puis en déduire les variations de $f$.
\item Est-ce que $f$ admet un maximum? un minimum? Calculer sa valeur.
\end{enumerate}
\end{exercise}
\begin{solution}
\begin{enumerate}
\item On remplace $x$ par les valeurs demandées
\[
f(1) = 5 \times 1^{2} - 70 \times 1 + 240=5 \times 1 - 70 + 240=5 + 170=175
\]
\[
f(-1) = 5 \times - 1^{2} - 70(- 1) + 240=5 \times 1 + 70 + 240=5 + 310=315
\]
\item Dérivation
\[
f'(x) = 10x - 70
\]
\item Pas de solutions automatiques.
\item Pas de solutions automatiques.
\end{enumerate}
\end{solution}
\begin{exercise}[subtitle={Enclos}]
Dans son garage, Jean a trouvé 29m de grillage. \\
Il décide de l'utiliser pour faire un enclos rectangulaire. Afin d'obtenir un enclos encore plus grand, il veut utiliser le mur du jardin qui formera un côté. Le grillage formera les 3 autres.
\begin{center}
\includegraphics[scale=0.8]{./fig/enclos}
\end{center}
Comment placer les poteaux pour avoir un enclos le plus grand possible ?
\textit{Cet exercice est un exercice de recherche. Vous êtes invité à utiliser les outils que vous voulez. Une part importante de la note sera dédiée à la rédaction et aux explications de ce que vous faites. Vous êtes encouragé à faire des schémas.}
\textit{Si vous utilisez le tableur, vous devrez le mentionner et m'envoyer le tableur à l'adresse \url{benjamin.bertrand@ac-lyon.fr}}
\end{exercise}
\begin{solution}
Formule de l'aire (en notant $x$ la longueur du côté de l'enclos à côté du mur)
\[
A(x) = x(29 - 2x) = - 2x^{2} + 29x
\]
On va donc étudier les variations de la fonction $A(x) = - 2x^{2} + 29x$
\begin{itemize}
\item Fonction dérivée : $A'(x) = - 4x + 29$
\item On résout l'inéquation $A'(x) \geq 0$ pour déterminer quand la fonction $A'$ est positive.
\begin{align*}
A(x) & \geq 0 \\
- 4x + 29 & \geq 0 \\
- 4x + 29 + - 29 &\geq 0 + - 29 \\
- 4x &\geq - 29 \\
\frac{- 4x}{- 4} &\leq \frac{- 29}{- 4} \\
x &\leq \dfrac{29}{4} \\
\end{align*}
Donc $A(x)$ est positif quand $x$ est plus \textbf{petit} que $\dfrac{29}{4}$
\item
\begin{center}
\begin{tikzpicture}
\tkzTabInit[lgt=3,espcl=4]{$x$/1,Signe de $f'(x)$/2, Variations de $f(x)$/2}{, $\dfrac{29}{4}$ ,}%
\tkzTabLine{, +, z, -, }
\tkzTabVar{-/ ,+/$f(\dfrac{29}{4}) = \dfrac{1682}{16}$ , -/}%
\end{tikzpicture}
\end{center}
\end{itemize}
Pour avoir le plus grand enclos, il faut placer les poteaux à 6m du mur et on aura alors un enclos de $72m^2$.
\end{solution}
\end{document}
%%% Local Variables:
%%% mode: latex
%%% TeX-master: "master"
%%% End:

Binary file not shown.

View File

@ -0,0 +1,158 @@
\documentclass[a4paper,12pt]{article}
\usepackage{myXsim}
\usepackage{pgfplots}
\usetikzlibrary{decorations.markings}
\pgfplotsset{compat=1.18}
\title{ DM1 \hfill }
\tribe{1ST}
\date{A rendre pour le lundi 27 mars 2023}
\duree{}
\xsimsetup{
solution/print = true
}
\pagestyle{empty}
\begin{document}
\maketitle
Le barème est donné à titre indicatif, il pourra être modifié.
\begin{exercise}[subtitle={Polynôme de degré 2}]
Développer les expressions suivantes pour vérifier que ce sont des polynômes de degré 2. Vous préciserez les valeurs de $a$, $b$ et $c$.
\begin{multicols}{2}
\begin{enumerate}
\item $f(x) = (3x - 6)(- 7x - 6)$
\item $g(x) = (- 3x + 10)^{2}$
\item $h(x) = - 10 + x(- 7x - 4)$
\item $i(x) = - 1x^{2} + x(- 8x - 4)$
\item $j(x) = 6(x - 3)(x + 8)$
\item $k(x) = 5(x + 6)(x - 10)$
\end{enumerate}
\end{multicols}
\end{exercise}
\begin{solution}
\begin{enumerate}
\item
\begin{align*}
f(x) &= (3x - 6)(- 7x - 6)\\&= 3x \times - 7x + 3x(- 6) - 6 \times - 7x - 6(- 6)\\&= 3(- 7) \times x^{1 + 1} - 6 \times 3 \times x - 6(- 7) \times x + 36\\&= - 18x + 42x - 21x^{2} + 36\\&= (- 18 + 42) \times x - 21x^{2} + 36\\&= - 21x^{2} + 24x + 36
\end{align*}
C'est un polynôme de degré 2 avec $a = - 21$, $b = 24$ et $c = 36$.
\item
\begin{align*}
g(x) &= (- 3x + 10)^{2}\\&= (- 3x + 10)(- 3x + 10)\\&= - 3x \times - 3x - 3x \times 10 + 10 \times - 3x + 10 \times 10\\&= - 3(- 3) \times x^{1 + 1} + 10(- 3) \times x + 10(- 3) \times x + 100\\&= - 30x - 30x + 9x^{2} + 100\\&= (- 30 - 30) \times x + 9x^{2} + 100\\&= 9x^{2} - 60x + 100
\end{align*}
C'est un polynôme de degré 2 avec $a = 9$, $b = - 60$ et $c = 100$.
\item
\begin{align*}
h(x) &= - 10 + x(- 7x - 4)\\&= - 10 + x \times - 7x + x(- 4)\\&= - 7x^{2} - 4x - 10
\end{align*}
C'est un polynôme de degré 2 avec $a = - 7$, $b = - 4$ et $c = - 10$.
\item
\begin{align*}
i(x) &= - 1x^{2} + x(- 8x - 4)\\&= - x^{2} + x \times - 8x + x(- 4)\\&= - x^{2} - 8x^{2} - 4x\\&= - x^{2} - 8x^{2} - 4x\\&= (- 1 - 8) \times x^{2} - 4x\\&= - 9x^{2} - 4x
\end{align*}
C'est un polynôme de degré 2 avec $a = - 9$, $b = - 4$ et $c = 0$.
\item
\begin{align*}
j(x) &= 6(x - 3)(x + 8)\\&= (6x + 6(- 3))(x + 8)\\&= (6x - 18)(x + 8)\\&= 6x \times x + 6x \times 8 - 18x - 18 \times 8\\&= 8 \times 6 \times x - 144 + 6x^{2} - 18x\\&= 48x - 144 + 6x^{2} - 18x\\&= 6x^{2} + 48x - 18x - 144\\&= 6x^{2} + (48 - 18) \times x - 144\\&= 6x^{2} + 30x - 144
\end{align*}
C'est un polynôme de degré 2 avec $a = 6$, $b = 30$ et $c = - 144$.
\item
\begin{align*}
k(x) &= 5(x + 6)(x - 10)\\&= (5x + 5 \times 6)(x - 10)\\&= (5x + 30)(x - 10)\\&= 5x \times x + 5x(- 10) + 30x + 30(- 10)\\&= - 10 \times 5 \times x - 300 + 5x^{2} + 30x\\&= - 50x - 300 + 5x^{2} + 30x\\&= 5x^{2} - 50x + 30x - 300\\&= 5x^{2} + (- 50 + 30) \times x - 300\\&= 5x^{2} - 20x - 300
\end{align*}
C'est un polynôme de degré 2 avec $a = 5$, $b = - 20$ et $c = - 300$.
\end{enumerate}
\end{solution}
\begin{exercise}[subtitle={Étude de fonctions}]
Soit $f(x) = 6x^{2} - 48x - 120$ une fonction définie sur $\R$.
\begin{enumerate}
\item Calculer les valeurs suivantes
\[
f(1) \qquad f(-2)
\]
\item Dériver la fonction $f$
\item Étudier le signe de $f'$ puis en déduire les variations de $f$.
\item Est-ce que $f$ admet un maximum? un minimum? Calculer sa valeur.
\end{enumerate}
\end{exercise}
\begin{solution}
\begin{enumerate}
\item On remplace $x$ par les valeurs demandées
\[
f(1) = 6 \times 1^{2} - 48 \times 1 - 120=6 \times 1 - 48 - 120=6 - 168=- 162
\]
\[
f(-1) = 6 \times - 1^{2} - 48(- 1) - 120=6 \times 1 + 48 - 120=6 - 72=- 66
\]
\item Dérivation
\[
f'(x) = 12x - 48
\]
\item Pas de solutions automatiques.
\item Pas de solutions automatiques.
\end{enumerate}
\end{solution}
\begin{exercise}[subtitle={Enclos}]
Dans son garage, Jean a trouvé 24m de grillage. \\
Il décide de l'utiliser pour faire un enclos rectangulaire. Afin d'obtenir un enclos encore plus grand, il veut utiliser le mur du jardin qui formera un côté. Le grillage formera les 3 autres.
\begin{center}
\includegraphics[scale=0.8]{./fig/enclos}
\end{center}
Comment placer les poteaux pour avoir un enclos le plus grand possible ?
\textit{Cet exercice est un exercice de recherche. Vous êtes invité à utiliser les outils que vous voulez. Une part importante de la note sera dédiée à la rédaction et aux explications de ce que vous faites. Vous êtes encouragé à faire des schémas.}
\textit{Si vous utilisez le tableur, vous devrez le mentionner et m'envoyer le tableur à l'adresse \url{benjamin.bertrand@ac-lyon.fr}}
\end{exercise}
\begin{solution}
Formule de l'aire (en notant $x$ la longueur du côté de l'enclos à côté du mur)
\[
A(x) = x(24 - 2x) = - 2x^{2} + 24x
\]
On va donc étudier les variations de la fonction $A(x) = - 2x^{2} + 24x$
\begin{itemize}
\item Fonction dérivée : $A'(x) = - 4x + 24$
\item On résout l'inéquation $A'(x) \geq 0$ pour déterminer quand la fonction $A'$ est positive.
\begin{align*}
A(x) & \geq 0 \\
- 4x + 24 & \geq 0 \\
- 4x + 24 + - 24 &\geq 0 + - 24 \\
- 4x &\geq - 24 \\
\frac{- 4x}{- 4} &\leq \frac{- 24}{- 4} \\
x &\leq 6 \\
\end{align*}
Donc $A(x)$ est positif quand $x$ est plus \textbf{petit} que $6$
\item
\begin{center}
\begin{tikzpicture}
\tkzTabInit[lgt=3,espcl=4]{$x$/1,Signe de $f'(x)$/2, Variations de $f(x)$/2}{, $6$ ,}%
\tkzTabLine{, +, z, -, }
\tkzTabVar{-/ ,+/$f(6) = 72$ , -/}%
\end{tikzpicture}
\end{center}
\end{itemize}
Pour avoir le plus grand enclos, il faut placer les poteaux à 6m du mur et on aura alors un enclos de $72m^2$.
\end{solution}
\end{document}
%%% Local Variables:
%%% mode: latex
%%% TeX-master: "master"
%%% End:

Binary file not shown.

View File

@ -0,0 +1,158 @@
\documentclass[a4paper,12pt]{article}
\usepackage{myXsim}
\usepackage{pgfplots}
\usetikzlibrary{decorations.markings}
\pgfplotsset{compat=1.18}
\title{ DM1 \hfill TODESCHINI Alissa}
\tribe{1ST}
\date{A rendre pour le lundi 27 mars 2023}
\duree{}
\xsimsetup{
solution/print = true
}
\pagestyle{empty}
\begin{document}
\maketitle
Le barème est donné à titre indicatif, il pourra être modifié.
\begin{exercise}[subtitle={Polynôme de degré 2}]
Développer les expressions suivantes pour vérifier que ce sont des polynômes de degré 2. Vous préciserez les valeurs de $a$, $b$ et $c$.
\begin{multicols}{2}
\begin{enumerate}
\item $f(x) = (- 8x - 2)(- 7x - 2)$
\item $g(x) = (6x + 5)^{2}$
\item $h(x) = - 4 + x(6x - 1)$
\item $i(x) = - 7x^{2} + x(- 6x - 3)$
\item $j(x) = - 9(x - 4)(x - 4)$
\item $k(x) = 8(x - 2)(x - 4)$
\end{enumerate}
\end{multicols}
\end{exercise}
\begin{solution}
\begin{enumerate}
\item
\begin{align*}
f(x) &= (- 8x - 2)(- 7x - 2)\\&= - 8x \times - 7x - 8x(- 2) - 2 \times - 7x - 2(- 2)\\&= - 8(- 7) \times x^{1 + 1} - 2(- 8) \times x - 2(- 7) \times x + 4\\&= 16x + 14x + 56x^{2} + 4\\&= (16 + 14) \times x + 56x^{2} + 4\\&= 56x^{2} + 30x + 4
\end{align*}
C'est un polynôme de degré 2 avec $a = 56$, $b = 30$ et $c = 4$.
\item
\begin{align*}
g(x) &= (6x + 5)^{2}\\&= (6x + 5)(6x + 5)\\&= 6x \times 6x + 6x \times 5 + 5 \times 6x + 5 \times 5\\&= 6 \times 6 \times x^{1 + 1} + 5 \times 6 \times x + 5 \times 6 \times x + 25\\&= 30x + 30x + 36x^{2} + 25\\&= (30 + 30) \times x + 36x^{2} + 25\\&= 36x^{2} + 60x + 25
\end{align*}
C'est un polynôme de degré 2 avec $a = 36$, $b = 60$ et $c = 25$.
\item
\begin{align*}
h(x) &= - 4 + x(6x - 1)\\&= - 4 + x \times 6x + x(- 1)\\&= 6x^{2} - x - 4
\end{align*}
C'est un polynôme de degré 2 avec $a = 6$, $b = - 1$ et $c = - 4$.
\item
\begin{align*}
i(x) &= - 7x^{2} + x(- 6x - 3)\\&= - 7x^{2} + x \times - 6x + x(- 3)\\&= - 7x^{2} - 6x^{2} - 3x\\&= - 7x^{2} - 6x^{2} - 3x\\&= (- 7 - 6) \times x^{2} - 3x\\&= - 13x^{2} - 3x
\end{align*}
C'est un polynôme de degré 2 avec $a = - 13$, $b = - 3$ et $c = 0$.
\item
\begin{align*}
j(x) &= - 9(x - 4)(x - 4)\\&= (- 9x - 9(- 4))(x - 4)\\&= (- 9x + 36)(x - 4)\\&= - 9x \times x - 9x(- 4) + 36x + 36(- 4)\\&= - 4(- 9) \times x - 144 - 9x^{2} + 36x\\&= 36x - 144 - 9x^{2} + 36x\\&= - 9x^{2} + 36x + 36x - 144\\&= - 9x^{2} + (36 + 36) \times x - 144\\&= - 9x^{2} + 72x - 144
\end{align*}
C'est un polynôme de degré 2 avec $a = - 9$, $b = 72$ et $c = - 144$.
\item
\begin{align*}
k(x) &= 8(x - 2)(x - 4)\\&= (8x + 8(- 2))(x - 4)\\&= (8x - 16)(x - 4)\\&= 8x \times x + 8x(- 4) - 16x - 16(- 4)\\&= - 4 \times 8 \times x + 64 + 8x^{2} - 16x\\&= - 32x + 64 + 8x^{2} - 16x\\&= 8x^{2} - 32x - 16x + 64\\&= 8x^{2} + (- 32 - 16) \times x + 64\\&= 8x^{2} - 48x + 64
\end{align*}
C'est un polynôme de degré 2 avec $a = 8$, $b = - 48$ et $c = 64$.
\end{enumerate}
\end{solution}
\begin{exercise}[subtitle={Étude de fonctions}]
Soit $f(x) = 2x^{2} - 162$ une fonction définie sur $\R$.
\begin{enumerate}
\item Calculer les valeurs suivantes
\[
f(1) \qquad f(-2)
\]
\item Dériver la fonction $f$
\item Étudier le signe de $f'$ puis en déduire les variations de $f$.
\item Est-ce que $f$ admet un maximum? un minimum? Calculer sa valeur.
\end{enumerate}
\end{exercise}
\begin{solution}
\begin{enumerate}
\item On remplace $x$ par les valeurs demandées
\[
f(1) = 2 \times 1^{2} - 162=2 \times 1 - 162=2 - 162=- 160
\]
\[
f(-1) = 2 \times - 1^{2} - 162=2 \times 1 - 162=2 - 162=- 160
\]
\item Dérivation
\[
f'(x) = 4x
\]
\item Pas de solutions automatiques.
\item Pas de solutions automatiques.
\end{enumerate}
\end{solution}
\begin{exercise}[subtitle={Enclos}]
Dans son garage, Jean a trouvé 37m de grillage. \\
Il décide de l'utiliser pour faire un enclos rectangulaire. Afin d'obtenir un enclos encore plus grand, il veut utiliser le mur du jardin qui formera un côté. Le grillage formera les 3 autres.
\begin{center}
\includegraphics[scale=0.8]{./fig/enclos}
\end{center}
Comment placer les poteaux pour avoir un enclos le plus grand possible ?
\textit{Cet exercice est un exercice de recherche. Vous êtes invité à utiliser les outils que vous voulez. Une part importante de la note sera dédiée à la rédaction et aux explications de ce que vous faites. Vous êtes encouragé à faire des schémas.}
\textit{Si vous utilisez le tableur, vous devrez le mentionner et m'envoyer le tableur à l'adresse \url{benjamin.bertrand@ac-lyon.fr}}
\end{exercise}
\begin{solution}
Formule de l'aire (en notant $x$ la longueur du côté de l'enclos à côté du mur)
\[
A(x) = x(37 - 2x) = - 2x^{2} + 37x
\]
On va donc étudier les variations de la fonction $A(x) = - 2x^{2} + 37x$
\begin{itemize}
\item Fonction dérivée : $A'(x) = - 4x + 37$
\item On résout l'inéquation $A'(x) \geq 0$ pour déterminer quand la fonction $A'$ est positive.
\begin{align*}
A(x) & \geq 0 \\
- 4x + 37 & \geq 0 \\
- 4x + 37 + - 37 &\geq 0 + - 37 \\
- 4x &\geq - 37 \\
\frac{- 4x}{- 4} &\leq \frac{- 37}{- 4} \\
x &\leq \dfrac{37}{4} \\
\end{align*}
Donc $A(x)$ est positif quand $x$ est plus \textbf{petit} que $\dfrac{37}{4}$
\item
\begin{center}
\begin{tikzpicture}
\tkzTabInit[lgt=3,espcl=4]{$x$/1,Signe de $f'(x)$/2, Variations de $f(x)$/2}{, $\dfrac{37}{4}$ ,}%
\tkzTabLine{, +, z, -, }
\tkzTabVar{-/ ,+/$f(\dfrac{37}{4}) = \dfrac{2738}{16}$ , -/}%
\end{tikzpicture}
\end{center}
\end{itemize}
Pour avoir le plus grand enclos, il faut placer les poteaux à 6m du mur et on aura alors un enclos de $72m^2$.
\end{solution}
\end{document}
%%% Local Variables:
%%% mode: latex
%%% TeX-master: "master"
%%% End:

Binary file not shown.

View File

@ -0,0 +1,158 @@
\documentclass[a4paper,12pt]{article}
\usepackage{myXsim}
\usepackage{pgfplots}
\usetikzlibrary{decorations.markings}
\pgfplotsset{compat=1.18}
\title{ DM1 \hfill VAN ES Tristan}
\tribe{1ST}
\date{A rendre pour le lundi 27 mars 2023}
\duree{}
\xsimsetup{
solution/print = true
}
\pagestyle{empty}
\begin{document}
\maketitle
Le barème est donné à titre indicatif, il pourra être modifié.
\begin{exercise}[subtitle={Polynôme de degré 2}]
Développer les expressions suivantes pour vérifier que ce sont des polynômes de degré 2. Vous préciserez les valeurs de $a$, $b$ et $c$.
\begin{multicols}{2}
\begin{enumerate}
\item $f(x) = (- 9x + 3)(- 9x + 3)$
\item $g(x) = (- 3x + 6)^{2}$
\item $h(x) = 10 + x(6x - 7)$
\item $i(x) = - 1x^{2} + x(- 10x - 4)$
\item $j(x) = 4(x - 3)(x + 4)$
\item $k(x) = 4(x + 9)(x + 3)$
\end{enumerate}
\end{multicols}
\end{exercise}
\begin{solution}
\begin{enumerate}
\item
\begin{align*}
f(x) &= (- 9x + 3)(- 9x + 3)\\&= - 9x \times - 9x - 9x \times 3 + 3 \times - 9x + 3 \times 3\\&= - 9(- 9) \times x^{1 + 1} + 3(- 9) \times x + 3(- 9) \times x + 9\\&= - 27x - 27x + 81x^{2} + 9\\&= (- 27 - 27) \times x + 81x^{2} + 9\\&= 81x^{2} - 54x + 9
\end{align*}
C'est un polynôme de degré 2 avec $a = 81$, $b = - 54$ et $c = 9$.
\item
\begin{align*}
g(x) &= (- 3x + 6)^{2}\\&= (- 3x + 6)(- 3x + 6)\\&= - 3x \times - 3x - 3x \times 6 + 6 \times - 3x + 6 \times 6\\&= - 3(- 3) \times x^{1 + 1} + 6(- 3) \times x + 6(- 3) \times x + 36\\&= - 18x - 18x + 9x^{2} + 36\\&= (- 18 - 18) \times x + 9x^{2} + 36\\&= 9x^{2} - 36x + 36
\end{align*}
C'est un polynôme de degré 2 avec $a = 9$, $b = - 36$ et $c = 36$.
\item
\begin{align*}
h(x) &= 10 + x(6x - 7)\\&= 10 + x \times 6x + x(- 7)\\&= 6x^{2} - 7x + 10
\end{align*}
C'est un polynôme de degré 2 avec $a = 6$, $b = - 7$ et $c = 10$.
\item
\begin{align*}
i(x) &= - 1x^{2} + x(- 10x - 4)\\&= - x^{2} + x \times - 10x + x(- 4)\\&= - x^{2} - 10x^{2} - 4x\\&= - x^{2} - 10x^{2} - 4x\\&= (- 1 - 10) \times x^{2} - 4x\\&= - 11x^{2} - 4x
\end{align*}
C'est un polynôme de degré 2 avec $a = - 11$, $b = - 4$ et $c = 0$.
\item
\begin{align*}
j(x) &= 4(x - 3)(x + 4)\\&= (4x + 4(- 3))(x + 4)\\&= (4x - 12)(x + 4)\\&= 4x \times x + 4x \times 4 - 12x - 12 \times 4\\&= 4 \times 4 \times x - 48 + 4x^{2} - 12x\\&= 16x - 48 + 4x^{2} - 12x\\&= 4x^{2} + 16x - 12x - 48\\&= 4x^{2} + (16 - 12) \times x - 48\\&= 4x^{2} + 4x - 48
\end{align*}
C'est un polynôme de degré 2 avec $a = 4$, $b = 4$ et $c = - 48$.
\item
\begin{align*}
k(x) &= 4(x + 9)(x + 3)\\&= (4x + 4 \times 9)(x + 3)\\&= (4x + 36)(x + 3)\\&= 4x \times x + 4x \times 3 + 36x + 36 \times 3\\&= 3 \times 4 \times x + 108 + 4x^{2} + 36x\\&= 12x + 108 + 4x^{2} + 36x\\&= 4x^{2} + 12x + 36x + 108\\&= 4x^{2} + (12 + 36) \times x + 108\\&= 4x^{2} + 48x + 108
\end{align*}
C'est un polynôme de degré 2 avec $a = 4$, $b = 48$ et $c = 108$.
\end{enumerate}
\end{solution}
\begin{exercise}[subtitle={Étude de fonctions}]
Soit $f(x) = 3x^{2} + 12x - 180$ une fonction définie sur $\R$.
\begin{enumerate}
\item Calculer les valeurs suivantes
\[
f(1) \qquad f(-2)
\]
\item Dériver la fonction $f$
\item Étudier le signe de $f'$ puis en déduire les variations de $f$.
\item Est-ce que $f$ admet un maximum? un minimum? Calculer sa valeur.
\end{enumerate}
\end{exercise}
\begin{solution}
\begin{enumerate}
\item On remplace $x$ par les valeurs demandées
\[
f(1) = 3 \times 1^{2} + 12 \times 1 - 180=3 \times 1 + 12 - 180=3 - 168=- 165
\]
\[
f(-1) = 3 \times - 1^{2} + 12(- 1) - 180=3 \times 1 - 12 - 180=3 - 192=- 189
\]
\item Dérivation
\[
f'(x) = 6x + 12
\]
\item Pas de solutions automatiques.
\item Pas de solutions automatiques.
\end{enumerate}
\end{solution}
\begin{exercise}[subtitle={Enclos}]
Dans son garage, Jean a trouvé 15m de grillage. \\
Il décide de l'utiliser pour faire un enclos rectangulaire. Afin d'obtenir un enclos encore plus grand, il veut utiliser le mur du jardin qui formera un côté. Le grillage formera les 3 autres.
\begin{center}
\includegraphics[scale=0.8]{./fig/enclos}
\end{center}
Comment placer les poteaux pour avoir un enclos le plus grand possible ?
\textit{Cet exercice est un exercice de recherche. Vous êtes invité à utiliser les outils que vous voulez. Une part importante de la note sera dédiée à la rédaction et aux explications de ce que vous faites. Vous êtes encouragé à faire des schémas.}
\textit{Si vous utilisez le tableur, vous devrez le mentionner et m'envoyer le tableur à l'adresse \url{benjamin.bertrand@ac-lyon.fr}}
\end{exercise}
\begin{solution}
Formule de l'aire (en notant $x$ la longueur du côté de l'enclos à côté du mur)
\[
A(x) = x(15 - 2x) = - 2x^{2} + 15x
\]
On va donc étudier les variations de la fonction $A(x) = - 2x^{2} + 15x$
\begin{itemize}
\item Fonction dérivée : $A'(x) = - 4x + 15$
\item On résout l'inéquation $A'(x) \geq 0$ pour déterminer quand la fonction $A'$ est positive.
\begin{align*}
A(x) & \geq 0 \\
- 4x + 15 & \geq 0 \\
- 4x + 15 + - 15 &\geq 0 + - 15 \\
- 4x &\geq - 15 \\
\frac{- 4x}{- 4} &\leq \frac{- 15}{- 4} \\
x &\leq \dfrac{15}{4} \\
\end{align*}
Donc $A(x)$ est positif quand $x$ est plus \textbf{petit} que $\dfrac{15}{4}$
\item
\begin{center}
\begin{tikzpicture}
\tkzTabInit[lgt=3,espcl=4]{$x$/1,Signe de $f'(x)$/2, Variations de $f(x)$/2}{, $\dfrac{15}{4}$ ,}%
\tkzTabLine{, +, z, -, }
\tkzTabVar{-/ ,+/$f(\dfrac{15}{4}) = \dfrac{450}{16}$ , -/}%
\end{tikzpicture}
\end{center}
\end{itemize}
Pour avoir le plus grand enclos, il faut placer les poteaux à 6m du mur et on aura alors un enclos de $72m^2$.
\end{solution}
\end{document}
%%% Local Variables:
%%% mode: latex
%%% TeX-master: "master"
%%% End:

Binary file not shown.

View File

@ -0,0 +1,158 @@
\documentclass[a4paper,12pt]{article}
\usepackage{myXsim}
\usepackage{pgfplots}
\usetikzlibrary{decorations.markings}
\pgfplotsset{compat=1.18}
\title{ DM1 \hfill YANIK Azra}
\tribe{1ST}
\date{A rendre pour le lundi 27 mars 2023}
\duree{}
\xsimsetup{
solution/print = true
}
\pagestyle{empty}
\begin{document}
\maketitle
Le barème est donné à titre indicatif, il pourra être modifié.
\begin{exercise}[subtitle={Polynôme de degré 2}]
Développer les expressions suivantes pour vérifier que ce sont des polynômes de degré 2. Vous préciserez les valeurs de $a$, $b$ et $c$.
\begin{multicols}{2}
\begin{enumerate}
\item $f(x) = (- 9x + 7)(- 5x + 7)$
\item $g(x) = (6x - 5)^{2}$
\item $h(x) = - 8 + x(- 3x - 7)$
\item $i(x) = - 3x^{2} + x(- 3x + 3)$
\item $j(x) = 4(x - 3)(x - 1)$
\item $k(x) = - 6(x + 8)(x + 4)$
\end{enumerate}
\end{multicols}
\end{exercise}
\begin{solution}
\begin{enumerate}
\item
\begin{align*}
f(x) &= (- 9x + 7)(- 5x + 7)\\&= - 9x \times - 5x - 9x \times 7 + 7 \times - 5x + 7 \times 7\\&= - 9(- 5) \times x^{1 + 1} + 7(- 9) \times x + 7(- 5) \times x + 49\\&= - 63x - 35x + 45x^{2} + 49\\&= (- 63 - 35) \times x + 45x^{2} + 49\\&= 45x^{2} - 98x + 49
\end{align*}
C'est un polynôme de degré 2 avec $a = 45$, $b = - 98$ et $c = 49$.
\item
\begin{align*}
g(x) &= (6x - 5)^{2}\\&= (6x - 5)(6x - 5)\\&= 6x \times 6x + 6x(- 5) - 5 \times 6x - 5(- 5)\\&= 6 \times 6 \times x^{1 + 1} - 5 \times 6 \times x - 5 \times 6 \times x + 25\\&= - 30x - 30x + 36x^{2} + 25\\&= (- 30 - 30) \times x + 36x^{2} + 25\\&= 36x^{2} - 60x + 25
\end{align*}
C'est un polynôme de degré 2 avec $a = 36$, $b = - 60$ et $c = 25$.
\item
\begin{align*}
h(x) &= - 8 + x(- 3x - 7)\\&= - 8 + x \times - 3x + x(- 7)\\&= - 3x^{2} - 7x - 8
\end{align*}
C'est un polynôme de degré 2 avec $a = - 3$, $b = - 7$ et $c = - 8$.
\item
\begin{align*}
i(x) &= - 3x^{2} + x(- 3x + 3)\\&= - 3x^{2} + x \times - 3x + x \times 3\\&= - 3x^{2} - 3x^{2} + 3x\\&= - 3x^{2} - 3x^{2} + 3x\\&= (- 3 - 3) \times x^{2} + 3x\\&= - 6x^{2} + 3x
\end{align*}
C'est un polynôme de degré 2 avec $a = - 6$, $b = 3$ et $c = 0$.
\item
\begin{align*}
j(x) &= 4(x - 3)(x - 1)\\&= (4x + 4(- 3))(x - 1)\\&= (4x - 12)(x - 1)\\&= 4x \times x + 4x(- 1) - 12x - 12(- 1)\\&= - 1 \times 4 \times x + 12 + 4x^{2} - 12x\\&= - 4x + 12 + 4x^{2} - 12x\\&= 4x^{2} - 4x - 12x + 12\\&= 4x^{2} + (- 4 - 12) \times x + 12\\&= 4x^{2} - 16x + 12
\end{align*}
C'est un polynôme de degré 2 avec $a = 4$, $b = - 16$ et $c = 12$.
\item
\begin{align*}
k(x) &= - 6(x + 8)(x + 4)\\&= (- 6x - 6 \times 8)(x + 4)\\&= (- 6x - 48)(x + 4)\\&= - 6x \times x - 6x \times 4 - 48x - 48 \times 4\\&= 4(- 6) \times x - 192 - 6x^{2} - 48x\\&= - 24x - 192 - 6x^{2} - 48x\\&= - 6x^{2} - 24x - 48x - 192\\&= - 6x^{2} + (- 24 - 48) \times x - 192\\&= - 6x^{2} - 72x - 192
\end{align*}
C'est un polynôme de degré 2 avec $a = - 6$, $b = - 72$ et $c = - 192$.
\end{enumerate}
\end{solution}
\begin{exercise}[subtitle={Étude de fonctions}]
Soit $f(x) = - 2x^{2} - 32x - 120$ une fonction définie sur $\R$.
\begin{enumerate}
\item Calculer les valeurs suivantes
\[
f(1) \qquad f(-2)
\]
\item Dériver la fonction $f$
\item Étudier le signe de $f'$ puis en déduire les variations de $f$.
\item Est-ce que $f$ admet un maximum? un minimum? Calculer sa valeur.
\end{enumerate}
\end{exercise}
\begin{solution}
\begin{enumerate}
\item On remplace $x$ par les valeurs demandées
\[
f(1) = - 2 \times 1^{2} - 32 \times 1 - 120=- 2 \times 1 - 32 - 120=- 2 - 152=- 154
\]
\[
f(-1) = - 2 \times - 1^{2} - 32(- 1) - 120=- 2 \times 1 + 32 - 120=- 2 - 88=- 90
\]
\item Dérivation
\[
f'(x) = - 4x - 32
\]
\item Pas de solutions automatiques.
\item Pas de solutions automatiques.
\end{enumerate}
\end{solution}
\begin{exercise}[subtitle={Enclos}]
Dans son garage, Jean a trouvé 30m de grillage. \\
Il décide de l'utiliser pour faire un enclos rectangulaire. Afin d'obtenir un enclos encore plus grand, il veut utiliser le mur du jardin qui formera un côté. Le grillage formera les 3 autres.
\begin{center}
\includegraphics[scale=0.8]{./fig/enclos}
\end{center}
Comment placer les poteaux pour avoir un enclos le plus grand possible ?
\textit{Cet exercice est un exercice de recherche. Vous êtes invité à utiliser les outils que vous voulez. Une part importante de la note sera dédiée à la rédaction et aux explications de ce que vous faites. Vous êtes encouragé à faire des schémas.}
\textit{Si vous utilisez le tableur, vous devrez le mentionner et m'envoyer le tableur à l'adresse \url{benjamin.bertrand@ac-lyon.fr}}
\end{exercise}
\begin{solution}
Formule de l'aire (en notant $x$ la longueur du côté de l'enclos à côté du mur)
\[
A(x) = x(30 - 2x) = - 2x^{2} + 30x
\]
On va donc étudier les variations de la fonction $A(x) = - 2x^{2} + 30x$
\begin{itemize}
\item Fonction dérivée : $A'(x) = - 4x + 30$
\item On résout l'inéquation $A'(x) \geq 0$ pour déterminer quand la fonction $A'$ est positive.
\begin{align*}
A(x) & \geq 0 \\
- 4x + 30 & \geq 0 \\
- 4x + 30 + - 30 &\geq 0 + - 30 \\
- 4x &\geq - 30 \\
\frac{- 4x}{- 4} &\leq \frac{- 30}{- 4} \\
x &\leq \dfrac{15}{2} \\
\end{align*}
Donc $A(x)$ est positif quand $x$ est plus \textbf{petit} que $\dfrac{15}{2}$
\item
\begin{center}
\begin{tikzpicture}
\tkzTabInit[lgt=3,espcl=4]{$x$/1,Signe de $f'(x)$/2, Variations de $f(x)$/2}{, $\dfrac{15}{2}$ ,}%
\tkzTabLine{, +, z, -, }
\tkzTabVar{-/ ,+/$f(\dfrac{15}{2}) = \dfrac{450}{4}$ , -/}%
\end{tikzpicture}
\end{center}
\end{itemize}
Pour avoir le plus grand enclos, il faut placer les poteaux à 6m du mur et on aura alors un enclos de $72m^2$.
\end{solution}
\end{document}
%%% Local Variables:
%%% mode: latex
%%% TeX-master: "master"
%%% End:

Binary file not shown.

View File

@ -0,0 +1,158 @@
\documentclass[a4paper,12pt]{article}
\usepackage{myXsim}
\usepackage{pgfplots}
\usetikzlibrary{decorations.markings}
\pgfplotsset{compat=1.18}
\title{ DM1 \hfill ZINBI Myriem}
\tribe{1ST}
\date{A rendre pour le lundi 27 mars 2023}
\duree{}
\xsimsetup{
solution/print = true
}
\pagestyle{empty}
\begin{document}
\maketitle
Le barème est donné à titre indicatif, il pourra être modifié.
\begin{exercise}[subtitle={Polynôme de degré 2}]
Développer les expressions suivantes pour vérifier que ce sont des polynômes de degré 2. Vous préciserez les valeurs de $a$, $b$ et $c$.
\begin{multicols}{2}
\begin{enumerate}
\item $f(x) = (- 1x + 9)(- 7x + 9)$
\item $g(x) = (2x + 2)^{2}$
\item $h(x) = 7 + x(- 10x - 3)$
\item $i(x) = - 1x^{2} + x(10x - 9)$
\item $j(x) = - 4(x + 10)(x + 4)$
\item $k(x) = - 6(x - 7)(x - 10)$
\end{enumerate}
\end{multicols}
\end{exercise}
\begin{solution}
\begin{enumerate}
\item
\begin{align*}
f(x) &= (- 1x + 9)(- 7x + 9)\\&= (- x) \times - 7x + (- x) \times 9 + 9 \times - 7x + 9 \times 9\\&= - 1(- 7) \times x^{1 + 1} + 9(- 1) \times x + 9(- 7) \times x + 81\\&= - 9x - 63x + 7x^{2} + 81\\&= (- 9 - 63) \times x + 7x^{2} + 81\\&= 7x^{2} - 72x + 81
\end{align*}
C'est un polynôme de degré 2 avec $a = 7$, $b = - 72$ et $c = 81$.
\item
\begin{align*}
g(x) &= (2x + 2)^{2}\\&= (2x + 2)(2x + 2)\\&= 2x \times 2x + 2x \times 2 + 2 \times 2x + 2 \times 2\\&= 2 \times 2 \times x^{1 + 1} + 2 \times 2 \times x + 2 \times 2 \times x + 4\\&= 4x + 4x + 4x^{2} + 4\\&= (4 + 4) \times x + 4x^{2} + 4\\&= 4x^{2} + 8x + 4
\end{align*}
C'est un polynôme de degré 2 avec $a = 4$, $b = 8$ et $c = 4$.
\item
\begin{align*}
h(x) &= 7 + x(- 10x - 3)\\&= 7 + x \times - 10x + x(- 3)\\&= - 10x^{2} - 3x + 7
\end{align*}
C'est un polynôme de degré 2 avec $a = - 10$, $b = - 3$ et $c = 7$.
\item
\begin{align*}
i(x) &= - 1x^{2} + x(10x - 9)\\&= - x^{2} + x \times 10x + x(- 9)\\&= - x^{2} + 10x^{2} - 9x\\&= - x^{2} + 10x^{2} - 9x\\&= (- 1 + 10) \times x^{2} - 9x\\&= 9x^{2} - 9x
\end{align*}
C'est un polynôme de degré 2 avec $a = 9$, $b = - 9$ et $c = 0$.
\item
\begin{align*}
j(x) &= - 4(x + 10)(x + 4)\\&= (- 4x - 4 \times 10)(x + 4)\\&= (- 4x - 40)(x + 4)\\&= - 4x \times x - 4x \times 4 - 40x - 40 \times 4\\&= 4(- 4) \times x - 160 - 4x^{2} - 40x\\&= - 16x - 160 - 4x^{2} - 40x\\&= - 4x^{2} - 16x - 40x - 160\\&= - 4x^{2} + (- 16 - 40) \times x - 160\\&= - 4x^{2} - 56x - 160
\end{align*}
C'est un polynôme de degré 2 avec $a = - 4$, $b = - 56$ et $c = - 160$.
\item
\begin{align*}
k(x) &= - 6(x - 7)(x - 10)\\&= (- 6x - 6(- 7))(x - 10)\\&= (- 6x + 42)(x - 10)\\&= - 6x \times x - 6x(- 10) + 42x + 42(- 10)\\&= - 10(- 6) \times x - 420 - 6x^{2} + 42x\\&= 60x - 420 - 6x^{2} + 42x\\&= - 6x^{2} + 60x + 42x - 420\\&= - 6x^{2} + (60 + 42) \times x - 420\\&= - 6x^{2} + 102x - 420
\end{align*}
C'est un polynôme de degré 2 avec $a = - 6$, $b = 102$ et $c = - 420$.
\end{enumerate}
\end{solution}
\begin{exercise}[subtitle={Étude de fonctions}]
Soit $f(x) = 3x^{2} - 147$ une fonction définie sur $\R$.
\begin{enumerate}
\item Calculer les valeurs suivantes
\[
f(1) \qquad f(-2)
\]
\item Dériver la fonction $f$
\item Étudier le signe de $f'$ puis en déduire les variations de $f$.
\item Est-ce que $f$ admet un maximum? un minimum? Calculer sa valeur.
\end{enumerate}
\end{exercise}
\begin{solution}
\begin{enumerate}
\item On remplace $x$ par les valeurs demandées
\[
f(1) = 3 \times 1^{2} - 147=3 \times 1 - 147=3 - 147=- 144
\]
\[
f(-1) = 3 \times - 1^{2} - 147=3 \times 1 - 147=3 - 147=- 144
\]
\item Dérivation
\[
f'(x) = 6x
\]
\item Pas de solutions automatiques.
\item Pas de solutions automatiques.
\end{enumerate}
\end{solution}
\begin{exercise}[subtitle={Enclos}]
Dans son garage, Jean a trouvé 35m de grillage. \\
Il décide de l'utiliser pour faire un enclos rectangulaire. Afin d'obtenir un enclos encore plus grand, il veut utiliser le mur du jardin qui formera un côté. Le grillage formera les 3 autres.
\begin{center}
\includegraphics[scale=0.8]{./fig/enclos}
\end{center}
Comment placer les poteaux pour avoir un enclos le plus grand possible ?
\textit{Cet exercice est un exercice de recherche. Vous êtes invité à utiliser les outils que vous voulez. Une part importante de la note sera dédiée à la rédaction et aux explications de ce que vous faites. Vous êtes encouragé à faire des schémas.}
\textit{Si vous utilisez le tableur, vous devrez le mentionner et m'envoyer le tableur à l'adresse \url{benjamin.bertrand@ac-lyon.fr}}
\end{exercise}
\begin{solution}
Formule de l'aire (en notant $x$ la longueur du côté de l'enclos à côté du mur)
\[
A(x) = x(35 - 2x) = - 2x^{2} + 35x
\]
On va donc étudier les variations de la fonction $A(x) = - 2x^{2} + 35x$
\begin{itemize}
\item Fonction dérivée : $A'(x) = - 4x + 35$
\item On résout l'inéquation $A'(x) \geq 0$ pour déterminer quand la fonction $A'$ est positive.
\begin{align*}
A(x) & \geq 0 \\
- 4x + 35 & \geq 0 \\
- 4x + 35 + - 35 &\geq 0 + - 35 \\
- 4x &\geq - 35 \\
\frac{- 4x}{- 4} &\leq \frac{- 35}{- 4} \\
x &\leq \dfrac{35}{4} \\
\end{align*}
Donc $A(x)$ est positif quand $x$ est plus \textbf{petit} que $\dfrac{35}{4}$
\item
\begin{center}
\begin{tikzpicture}
\tkzTabInit[lgt=3,espcl=4]{$x$/1,Signe de $f'(x)$/2, Variations de $f(x)$/2}{, $\dfrac{35}{4}$ ,}%
\tkzTabLine{, +, z, -, }
\tkzTabVar{-/ ,+/$f(\dfrac{35}{4}) = \dfrac{2450}{16}$ , -/}%
\end{tikzpicture}
\end{center}
\end{itemize}
Pour avoir le plus grand enclos, il faut placer les poteaux à 6m du mur et on aura alors un enclos de $72m^2$.
\end{solution}
\end{document}
%%% Local Variables:
%%% mode: latex
%%% TeX-master: "master"
%%% End:

Binary file not shown.

Binary file not shown.

After

Width:  |  Height:  |  Size: 34 KiB

Binary file not shown.

Some files were not shown because too many files have changed in this diff Show More